Avaruus.fi - keskustelualue

Tähtiharrastus ja tähtitiede => Kosmologia => Aiheen aloitti: velihopea - 11.06.2023, 17:01:53

Otsikko: Epäselvyys suhteellisuusteorian aika ja etäisyys käsityksissä
Kirjoitti: velihopea - 11.06.2023, 17:01:53
Päivää. Koetin saada saada kysymyksen Ta-lehden kysymyksiä & vastauksia palstalle, mutta tekstini osoittautui liian pitkäksi. Sain toimitukselta vinkin, että tällä foorumilla voisi esittää pidempiäkin kysymyksiä ja keskustelunaiheita.

Kysymys on (suppean) suhteellisuusteorian ymmärtämisestä, todellisuuskuvasta eri liiketiloissa. Tähtitiede/Kosmologia tuntui oikealta ryhmältä. Ensin luin koko ryhmän jutut suurinpiirtien läpi (huh, oli iso urakka), mutta täsmälleen kysymykseeni en löytänyt  vastausta, vaikka aiheen ympärillä muutamassa säikeessä pyörittiinkin.

Mielenkiinnolla odotan, onko tällä aihealueella enää aktiivisia seuraajia ja viisaita, jotka osaisivat kommentoida. Kiitos etukäteen. Itse kysymys on seuraavassa postauksessani.
Otsikko: Vs: Epäselvyys suhteellisuusteorian aika ja etäisyys käsityksissä
Kirjoitti: velihopea - 11.06.2023, 17:03:43
Otetaan taas suhteellisuusteorian (ST) yhteydessä käytetty hypoteettinen kaksosparadoksi-esimerkki. Siinä kaksosista A jää Maahan ja B käy raketilla Siriuksessa 8vv päässä. Raketti kulkee 0.8c vauhtia. Gravitaatiota, kiihdytyksiä, jarrutuksia ei huomioida. A laskee matkan kestävän 10v suuntaansa, yhteensä 20v.

Lisäksi A lähettää raketin lähtöhetkestä laskien joka vuoden lopussa radioviestin B:lle.

Einsteinin esittämän kaavan (1-sqrt(1-(v/c)^2)) mukaan aika hidastuu  v = 0.8c-nopeudessa 0.6-osaan. Eli kun esim. Maan kello näyttää 10h, tuolla nopeudella liikuva kello näyttää vasta 6h. Kaavan toimivuus on todettu kokein (eli atomikellon käynti on hidastunut). Mm. GPS-paikannus perustuu osaksi tuohon kaavaan.

1-suuntainen matka kesti Maan kellon mukaan 10v, raketin kellon mukaan 6v. ST pitää 6v todellisena ja oikeasti toteutuneena. Jotta raketin nopeus 0.8c pysyisi entisellään, ST sanoo, että kuljettu matka on lyhentynyt vastaavasti, eli 8vv:sta 4.8vv:een. Paluumatka samoin. ST:n mukaan kaikkiaan raketissa kului 12v ja se lensi 9.6vv matkan.

Nyt seuraa näkökohtia, jotka ei oikein käy järkeeni:

1. B on vastaanottanut A:n radioviestit. Hetkeä ennen kun B laskeutuu takaisin Maahan, viestejä on tullut 20. Kumpaa vuosimäärää B:n pitäisi pitää oikeana lennon kestona: raketin kelloa mukaista 12v vai viestien lukumäärän tarkoittamaan 20v.

2. Oikeastaan B ei tarvitse edes "synkronoivia" radioviestejä. Sillä B tietää, että raketin vauhti Maan suhteen on 0.8c. (Tässä nopeudessa Maan oma liike on suhteessa niin pieni, että Maan voi olettaa olevan paikallaan ilman että tämän hypotettisen laskuharjoituksen demo-vaikutus tulee paljoakaan epätarkaksi.) Esimerkiksi saapumishetkellä Siriukseen raketin kello näyttää 6v. B tietää, että 0.8c nopeudessa raketin kellon käynti on hidastunut 0.6-osaan. Kun raketin kellon hidastuminen oikaistaan saadaan 10v (niin kuin pitääkin). Ja matkaa on taivallettu 8vv (niin kuin pitääkin). Pituuskontraktio-mallinnosta ei tarvita!

Varmaan joku muukin on mielessään esittänyt samanlaisen skenaarion kuin edellä. Kysynkin miten virallinen ST tyrmää esitykseni. ST:n tulkinta, että liikkeessä aika on sama kun liikkuvan kellon lukema, voisi olla jotenkin mallinnettavissa. Mutta ST:n mukainen todellisen pituuskontraktion tapahtuminen on jo "liian paksua". Etäisyyksien ei tarvitse muuttua liikkeessa edellä kuvaamallani mallinnoksella.
Otsikko: Vs: Epäselvyys suhteellisuusteorian aika ja etäisyys käsityksissä
Kirjoitti: einari - 11.06.2023, 18:37:05
En nyt vastaa kysymykseesi. Joku varmaan suoraankin osaa sen tehdä.
Mutta jos tällaiset kysymykset kiinnostaa niin suosittelen lämpimästi Jyväskylän yliopiston (avoin) suhteellisuusteorian ja kvanttimekaniikan opintoja.
On kaikenlisäksi ilmainen.
Sain juuri hiljattain itse suoritettua loppuun ja oli tosiaan mielenkiintoista.

Linkki materiaaliin:
https://onlinecourses.jyu.fi/course/view.php?id=37&section=17
Otsikko: Vs: Epäselvyys suhteellisuusteorian aika ja etäisyys käsityksissä
Kirjoitti: mistral - 11.06.2023, 20:34:54
Lainaus käyttäjältä: velihopea - 11.06.2023, 17:03:43
Mutta ST:n mukainen todellisen pituuskontraktion tapahtuminen on jo "liian paksua".

En minäkään vastaa itse kysymykseen, ainakaan vielä, mutta pituuskontraktiosta muutama sana. Miten itse ratkaisisit ongelman jos lähestyisit 0,8c nopeudella Proxima Centauria ja huomaisit että matka etenee "liian" nopeasti? Tarkoitan että mitä asiaa muuttaisit jotta nopeutesi ei ylittäisi 300 000km/s. Koska kelloa ei parane rukata, silloin aikaan ei voi kajota (kaikissa "omissa" koordinaatistoissa sekunti on sekunti). Joten jäljelle jää vain etäisyyden muuttaminen. Siksipä pituuskontraktio on otettu käyttöön, ongelma ei ratkea muuten.
Otsikko: Vs: Epäselvyys suhteellisuusteorian aika ja etäisyys käsityksissä
Kirjoitti: velihopea - 11.06.2023, 23:28:13
LainaaEn minäkään vastaa itse kysymykseen, ainakaan vielä, mutta pituuskontraktiosta muutama sana. Miten itse ratkaisisit ongelman jos lähestyisit 0,8c nopeudella Proxima Centauria ja huomaisit että matka etenee "liian" nopeasti? Tarkoitan että mitä asiaa muuttaisit jotta nopeutesi ei ylittäisi 300 000km/s. Koska kelloa ei parane rukata, silloin aikaan ei voi kajota (kaikissa "omissa" koordinaatistoissa sekunti on sekunti). Joten jäljelle jää vain etäisyyden muuttaminen. Siksipä pituuskontraktio on otettu käyttöön, ongelma ei ratkea muuten.

Jos raketin kellon lukema olisi todellinen toteutunut aika, sitä siis olisi matkaan kulunut 12v. Toisaalta muuta kautta rakettimies tietää, että aikaa matkaan kului 20v. Tämä on minusta ristiriita. Siitä pääsisi eroon, kun rakettimies korjaa matka/nopeus -laskelmissaan raketin kellon hidastumisen, minkä hän tietää siellä olevan.

Hieman sama kuvio, kun Helsingintä Turkuun ajoaika autolla on normaalisti 1:45. Jos toistaa saman ajon, samalla nopeudella, mutta käytössä sattuu olemaan kello, joka jätättää (näyttää ajoajaksi vaikka 1:10), ei auton nopeus eikä ajomatka ole mihinkään muuttuneet.
Otsikko: Vs: Epäselvyys suhteellisuusteorian aika ja etäisyys käsityksissä
Kirjoitti: mistral - 12.06.2023, 00:54:27
Itse olen vuosia ihmetellyt näitä asioita ja päätynyt lopulta seuraavaan, hyvin lyhyt kuvaus:

1. molemmat, sekä astronautti että maassa oleva liikkuu toistensa suhteen. Siksi molempien näkökulma pitää ottaa huomioon. Tämä onnistuu kun molempien maailmanviivat tunnetaan. Maailmanviiva kuvaa ajankulua, mitä lyhyempi viiva, sitä lyhyempi aika. Juuri pituuskontraktio lyhentää maailmanviivaa. Esim valon maailmanviiva on tasan nolla koska kontraktio on litistänyt avaruuden nollaan (etenemissuunnassa).
2. Kun maailmanviivat tunnetaan kummaltakin, niiden erotus kertoo matkan tuottaneen ikäeron.
3. Jotta maailmanviivat saadaan määritettyä, teen sen vain näiden kahden osapuolen välillä, unohdan koko muun universumin. Astronautin viiva määritetään suhteessa maan koordinaatistoon ja taas maan viiva määritetään suhteessa astronautin koordinaatistoon. Tilanne näyttää äkkiseltään symmetriseltä mutta ei ole sitä. Epäsymmetrisyys tulee siitä että vain astronautti kiihdyttää ja jarruttaa. Siksi astronautti on jatkuvasti (ilman viivettä) "tietoinen" maan koordinaatiston liiketilasta kun taas maa on etäisyydestä johtuen viiveellä "tietoinen" astronautin koordinaatistosta. Tämä viive juuri vaikuttaa maailmanviivan muodostukseen ja siksi astronautin saavuttua maahan, sen viiva on lyhyempi. Se on siis matkustanut voimakkaammassa kontraktiossa keskimäärin.

En väitä että välttämättä olisin oikeassa, ompahan yksi käsitys muiden joukossa.
Otsikko: Vs: Epäselvyys suhteellisuusteorian aika ja etäisyys käsityksissä
Kirjoitti: mistral - 12.06.2023, 13:37:29
Googletin maailmanviivan eikä löytynyt kuin Wikin selitys  https://fi.wikipedia.org/wiki/Maailmanviiva
Ei auennut minulle mutta tiedefoorumin keskusteluissa eräs fyysikko sanoi että maailmanviiva korreloi ikääntymisen kanssa. Oli erikoistunut suhteellisuusteoriaan joten varmaan tiesi tuon mutta itse maailmanviivan idea jäi minulle vain puheen varaan.

Löytyi enlanniksi käännös, worldline ja sillä kattavampi selitys   https://en.wikipedia.org/wiki/World_line
Otsikko: Vs: Epäselvyys suhteellisuusteorian aika ja etäisyys käsityksissä
Kirjoitti: velihopea - 12.06.2023, 23:48:56
LainaaItse olen vuosia ihmetellyt näitä asioita ja päätynyt lopulta seuraavaan, hyvin lyhyt kuvaus:

Kiitos mistral yrityksestäsi selventää minulle kysymystäni. Valitettavasti en saanut siitä helpotusta ymmärtämisen tuskaani.

Mainitsit termin "maailmanviiva". En muista törmänneeni siihen niissä muutamassa aiheen kirjassa, jotka olen lukenut. Siispä minäkin googlasin, ja todella se on kosmologian termi (~ne hiukkasen x,y,z koordinaatit, jossa hiukkanen on ajan t myötä, eli hiukkasen rata).

Toisaalta selityksesi tuntui minusta kuin minun olisi ensin pitänyt ymmärtää se ja "ostaa se kaikkine karvoineen". Kun kysymykseni ja epäselvyys (minulle) on ST:n aika ja etäisyys käsityksissä. Ne arvelen olevan ST:n peruskiviä joiden päälle maailmanviiva mallinnokset yms. voi rakentaa.

Nyt tuli mieleeni konstruoida rakettiesimerkki uudestaan, nyt ihan maallisista elementeistä. Tämän pohja oli jossain netissä, mutta en löydä sitä enää. Muokattuna se menee näin:

On suora tasainen maantie ja sen laidassa 2 tolppaa tarkasti 1 km etäisyydellä toisistaan. Kumpaankin tolppaan kiinnitetään tarkka atomikello ja ne synronoidaan näyttämään samaa aikaa.

On kolmaskin yhtä tarkka atomikello. Se on autossa. Auton ajaessa jomman kumman tolpan ohi tolpan kello ja auton kello rekisteröivät mitä lukemaa kumpikin kello näyttää. Ehkä joku lasersäde tolpan kohdalla triggeröi ajanoton.

Kuski ajaa autoa ja koettaa pitää mahdollisimman tarkkaan auton nopeuden 60 km/h:ssa tolppien välin. Testiajon jälkeen lasketaan ajoaika a) tolppien kelloista ja b) auton kellosta.

Jos sattuisi käymään niin, että tolppien kellon mukaan auton nopeus on ollut täsmälleen 60 km/h (1 min ajoaika), liikkuvan auton hidastuneen kellon mukaan aikaa kului hieman alle 1 min. Missä on pituuskontraktio eli kuljetun matkan lyheneminen ajan kulun (auton kello) suhteessa? En näe sitä. Vaan pidän varmana, että auto kulki täsmälleen 1 km.

Toisaalta jos sattuisi käymään niin, että auton kellon mukaan nopeus oli täsmälleen 60 km/h, tolppien nopeampikäyntisten kellojen mukaan aikaa kului hieman yli 1 min. Onko tässä joku pituuskontraktio läsnä? En näe sellaista. Vaan se oli se sama 1 km matka, jonka auto kulki.
Otsikko: Vs: Epäselvyys suhteellisuusteorian aika ja etäisyys käsityksissä
Kirjoitti: Mare Nectaris - 13.06.2023, 11:45:10
Tämä saattaisi olla avuksi:

https://iep.utm.edu/proper-t/
Otsikko: Vs: Epäselvyys suhteellisuusteorian aika ja etäisyys käsityksissä
Kirjoitti: mistral - 13.06.2023, 11:52:01
Lainaus käyttäjältä: velihopea - 12.06.2023, 23:48:56

On suora tasainen maantie ja sen laidassa 2 tolppaa tarkasti 1 km etäisyydellä toisistaan. Kumpaankin tolppaan kiinnitetään tarkka atomikello ja ne synronoidaan näyttämään samaa aikaa.

On kolmaskin yhtä tarkka atomikello. Se on autossa. Auton ajaessa jomman kumman tolpan ohi tolpan kello ja auton kello rekisteröivät mitä lukemaa kumpikin kello näyttää. Ehkä joku lasersäde tolpan kohdalla triggeröi ajanoton.

Kuski ajaa autoa ja koettaa pitää mahdollisimman tarkkaan auton nopeuden 60 km/h:ssa tolppien välin. Testiajon jälkeen lasketaan ajoaika a) tolppien kelloista ja b) auton kellosta.

Se menee niin että tolppien koordinaatistossa muuttuu eri asiat kuin auton koordinaatistossa.

............................................................tolppien koord. näkökulma ..................................auton koord. näkökulma

auton kello ....................................................muuttuu .........................................................ei muutu

tolppien kello ................................................ei muutu .........................................................muuttuu

auton pituus .................................................lyhenee .......................................................... ei lyhene

tolppien välimatka ........................................ei muutu ..........................................................muuttuu, siis lyhenee


Tein pisteet siksi että taulukon asiat pysyy oikeilla paikoilla. Taulukosta näkee että eri näkökulmat muuttaa eri asioita. Esim auton kello ei muutu autossa vaan siellä on aina samat luonnonlait, kvartsikide sykkii samalla lailla kuin parkissakin. Relativistinen muutos juontaa aika-avaruudesta, ei kellosta.
Otsikko: Vs: Epäselvyys suhteellisuusteorian aika ja etäisyys käsityksissä
Kirjoitti: velihopea - 14.06.2023, 18:08:18
LainaaSe menee niin että tolppien koordinaatistossa muuttuu eri asiat kuin auton koordinaatistossa.

............................................................tolppien koord. näkökulma ..................................auton koord. näkökulma

auton kello ....................................................muuttuu .........................................................ei muutu

tolppien kello ................................................ei muutu .........................................................muuttuu

auton pituus .................................................lyhenee .......................................................... ei lyhene

tolppien välimatka ........................................ei muutu ..........................................................muuttuu, siis lyhenee

Minä tosiaan taidan olla kovakallo, ainakin kova urputtamaan. Nimittäin autonajo -testissä tehtiin vain yksi koe. Olkoon se juuri se koe, jossa tolppien kellot osoitti täsmälleen 1 min ajoajan. Koska auto liikkui nopeammin, sen kello kävi hitaammin ja se näytti hieman alle 1 min ajoajan. Näin kai on? Ne lukemat ei siitä muutu vaikka näkökulmaa kuinka muutetaan.

Jos testi olisi järjestetty siten, että maantie olisikin kuin kuntosalin juoksumatto, joka kulkee 60 km/h, ja jossa mittatoplat kulkevat maton mukana. Auton moottori käyntiin, vaihde päälle ja sopivasti kaasua. Jos tässä mittauksessa liikkuvat (ja siksi nyt hitaammin käyvät) tolppien kellot olisivat taas saaneet ajoajaksi tasan 1 min, paikallaan ollut nopeammin käyvä auton kello näyttäisi hieman yli minuutin ajoaikaa. Näin kai on? Mutta tämä olisi eri testijärjestely, ei näkökulmakysymys.

LainaaTein pisteet siksi että taulukon asiat pysyy oikeilla paikoilla. Taulukosta näkee että eri näkökulmat muuttaa eri asioita. Esim auton kello ei muutu autossa vaan siellä on aina samat luonnonlait, kvartsikide sykkii samalla lailla kuin parkissakin. Relativistinen muutos juontaa aika-avaruudesta, ei kellosta.

Mitä? Eikö liikkeessä oleva kello käykään hitaammin kuin paikallaan oleva, myös kvartsikiteen sykkeeseen perustuva. Sen kyllä ymmärrän, että auton kuljettajasta tuntuu, että "tuossahan se kello tikittää ja näyttää näyttävän järkevää aikaa".
Otsikko: Vs: Epäselvyys suhteellisuusteorian aika ja etäisyys käsityksissä
Kirjoitti: mistral - 14.06.2023, 20:27:26




Suhteellisuudessa (erityinen suhteellisuusteoria) ei ole väliä kumpi liikkuu, efektit säilyy samoina jos vaan nopeuserot säilyy samoina. Oma koordinaatisto on aina muuttumaton. Muutos tulee oman ja toisen koordinaatiston välisestä aika-avaruudesta. Siksi näyttää siltä että aina toinen on se muuttuva osapuoli.

1. Auto A ajaa suola-aavikolla toista kohti 20km/t. Toinen B ajaa vastaan 180km/t. Koordinaatistojen nopeusero on 200km/t
2. A ajaa 80km/t   ja B 120km/t  =200km/t
3. A 150km/t  ja B 50km/t =200km/t
4. A 200km/t ja B 0km/t  =200km/t
Jokaisessa tapauksessa A:n mielestä  B:n kello käy hitaammin, sanotaan vaikka 2 nanosekuntia.
Myös jokaisessa tapauksessa B:n mielestä A:n kello käy 2 nanosekuntia hitaammin.
Tässä haetaan vain sitä että nopeusero on oleellinen asia, eikä ole väliä kumpi liikkuu kovempaa.
Tuloshan on hullu, kuinka molempien kellot jätättää ja vielä yhtä paljon. Mutta näin se vaan on. Tilanne helpottuu kun katsotaan maailmanviivoja, molempien maailmanviivat lyhenee yhtä paljon jolloin ikäero on nollassa.

Tosin jos kiihdytykset otetaan huomioon, saadaan aikaseksi pieni ikäero. Tämä johtuu siitä että koordinaatistot päivittyy c nopeuden viiveellä. Eli esim A on eri suhteessa B:hen kuin B on A:han.
Otsikko: Vs: Epäselvyys suhteellisuusteorian aika ja etäisyys käsityksissä
Kirjoitti: Kaizu - 16.06.2023, 11:02:27
Toistensa suhteen liikkuvissa koordinaatistoissa ainoa vakio on valon nopeus. Kaikki muu on suhteellista, kellojen käynti ja mittakepin pituus.

Kaizu
Otsikko: Vs: Epäselvyys suhteellisuusteorian aika ja etäisyys käsityksissä
Kirjoitti: mistral - 17.06.2023, 13:31:21
Kyllä vaan. Gps:n tapausta en ymmärrä kun siinä ikäeroa syntyy maan ja satelliitin välille. En nyt tarkoita gravitaation vaikuttamaa ikäeroa vaan nopeuden vaikuttamaa.  Mahtaakohan selitys olla siinä että satelliitti on jatkuvassa kiihtyvyydessä?
Otsikko: Vs: Epäselvyys suhteellisuusteorian aika ja etäisyys käsityksissä
Kirjoitti: Mare Nectaris - 18.06.2023, 09:35:56
Lainaus käyttäjältä: mistral - 17.06.2023, 13:31:21
Kyllä vaan. Gps:n tapausta en ymmärrä kun siinä ikäeroa syntyy maan ja satelliitin välille. En nyt tarkoita gravitaation vaikuttamaa ikäeroa vaan nopeuden vaikuttamaa.  Mahtaakohan selitys olla siinä että satelliitti on jatkuvassa kiihtyvyydessä?

Täällä selitetään sekä liikenopeus- että painovoimakomponentin vaikutukset signaalin aikasiirtymään:

https://www.astronomy.ohio-state.edu/pogge.1/Ast162/Unit5/gps.html
Otsikko: Vs: Epäselvyys suhteellisuusteorian aika ja etäisyys käsityksissä
Kirjoitti: velihopea - 18.06.2023, 13:45:28
Kiitos kaikille kommentoijille ja neuvon antajille. Kuitenkin olen hieman pettynyt, että kukaan ei tunnistanut (ainakaan ei sympatiseerannut niin, että olisi kommentoinut) asiaa, jota pidin epäselvänä. Viittaan rakettimatka Siriukseen 0.8c nopeudella 8vv päähän esimerkkiin. Kun rakettimies tietää, että hänen nopeusero Maan suhteen 0.8c ja lisäksi hän tietää, että kellon käynti (ja omat elintoimintonsakin) on hidastunut siinä nopeudessa 0.6-osaan, miksei rakettimies yhtä lailla voisi seurata Maan koordinaatistoa. Maan asukashan hän on ja aikoo sinne palata.

Kun kellot on synkronoitu matkan alussa, hidastuneesta raketin kellosta raketimies voi laskea sekunnin tarkasti paljonko Maan kello näyttää. Samoin hän voi laskea metrin tarkasti missä kohtaa hän on menossa Maa-Sirius 8vv taipaleella. Näin rakettimies ei koe epäselvyydeksi kokemaani asiaa, mm. "montako vuotta matkaan kului".

Yleisesti ottaen minusta tuntuu, että ST on valinnut tulkinnan: (tarkkailijan) aika on (tarkailijan mukana olevan) kellon lukema. Ja jotta yhtälö nopeus = matka / aika pitäisi yhä paikkansa, on matkan muututtava samassa suhteessa kuin ajan. Tavallaan matematiikka sanelee todellisuuskäsitystämme.

Kokein on todettu, että kellojen käynti muuttuu liikeessä ja gravitaatiossa, aivan Einsteinin esittämien kaavojen mukaan. Mutta ST ei selitä miksi niin tapahtuu.
Otsikko: Vs: Epäselvyys suhteellisuusteorian aika ja etäisyys käsityksissä
Kirjoitti: Mare Nectaris - 18.06.2023, 14:23:22
Ellei oivalla, että kummankaan "kellojen käynti" ei hidastu tai nopeudu heille itselleen vaan s u h t e e s s a  toisiinsa, ei voi ymmärtää valitettavasti s u h t e e l l i s u u steorian ideaa. Koordinaatisto kiinnittäen voi ainoastaan ymmärtää asiaa jomman kumman näkökulmasta. Luethan linkkaamani sisällöt. Se varmasti auttaa.
Otsikko: Vs: Epäselvyys suhteellisuusteorian aika ja etäisyys käsityksissä
Kirjoitti: mistral - 18.06.2023, 15:53:50
Lainaus käyttäjältä: velihopea - 18.06.2023, 13:45:28
Kiitos kaikille kommentoijille ja neuvon antajille. Kuitenkin olen hieman pettynyt, että kukaan ei tunnistanut (ainakaan ei sympatiseerannut niin, että olisi kommentoinut) asiaa, jota pidin epäselvänä. Viittaan rakettimatka Siriukseen 0.8c nopeudella 8vv päähän esimerkkiin. Kun rakettimies tietää, että hänen nopeusero Maan suhteen 0.8c ja lisäksi hän tietää, että kellon käynti (ja omat elintoimintonsakin) on hidastunut siinä nopeudessa 0.6-osaan, miksei rakettimies yhtä lailla voisi seurata Maan koordinaatistoa. Maan asukashan hän on ja aikoo sinne palata.

En tiedä perimmäistä syytä miksei rakettimies eläisi maan aikaa. Ehkä vihje löytyisi siitä että valon nopeus on kaikille koordinaatistoille sama. Eli luonto katselee asioita aina ikäänkuin universumin keskeltä käsin? Jos tämän periaatteen mukaan mennään, oma koordinaatisto on aina määräävä. Tämä on suhteellisuuden sisään rakennettu. Eli ollaan luovuttu eetteristä mistä seuraa se että on vain suhteiden verkosto.
Otsikko: Vs: Epäselvyys suhteellisuusteorian aika ja etäisyys käsityksissä
Kirjoitti: HarriR - 18.06.2023, 16:00:46
Ehkä siksi että universumi ei tunne mitään koordinaatistoa, sillä ei ole mitään keskustaa, liike on vain suhteellista toiseen kappaleeseen nähden.
Otsikko: Vs: Epäselvyys suhteellisuusteorian aika ja etäisyys käsityksissä
Kirjoitti: velihopea - 28.06.2023, 16:51:03
Olen ollut matkoilla, autolla ajoa n. 1500km. Ajovuorollani koetin ajaa nopeasti, rajoitusten puitteissa toki, mutta en silti havainnut erityistä matkan lyhentymistä ;) Mutta asiaan:

LainaaEllei oivalla, että kummankaan "kellojen käynti" ei hidastu tai nopeudu heille itselleen vaan s u h t e e s s a  toisiinsa, ei voi ymmärtää valitettavasti s u h t e e l l i s u u steorian ideaa. Koordinaatisto kiinnittäen voi ainoastaan ymmärtää asiaa jomman kumman näkökulmasta. Luethan linkkaamani sisällöt. Se varmasti auttaa.

Mielestän ymmärrän nuo painottamasi asiat. Voin toki olla väärässä. Luin myös ehdottamasi https://iep.utm.edu/proper-t/ tekstin. Aiempi Einarin ehdottama linkki https://onlinecourses.jyu.fi/course/view.php?id=37&section=17 oli minusta hyvin tehty ja ymmärrettävämmin asiat selittävä (katsoin kohdat aikadilataatiosta ja pituuskontraktiosta). Jotain voisin molemmista vielä kommentoida, mutta ehkei nyt.

Lainaa
Lainaa
Lainaus käyttäjältä: velihopea - 18-06-2023, 13:45:28

    Kiitos kaikille kommentoijille ja neuvon antajille. Kuitenkin olen hieman pettynyt, että kukaan ei tunnistanut (ainakaan ei sympatiseerannut niin, että olisi kommentoinut) asiaa, jota pidin epäselvänä. Viittaan rakettimatka Siriukseen 0.8c nopeudella 8vv päähän esimerkkiin. Kun rakettimies tietää, että hänen nopeusero Maan suhteen 0.8c ja lisäksi hän tietää, että kellon käynti (ja omat elintoimintonsakin) on hidastunut siinä nopeudessa 0.6-osaan, miksei rakettimies yhtä lailla voisi seurata Maan koordinaatistoa. Maan asukashan hän on ja aikoo sinne palata.

En tiedä perimmäistä syytä miksei rakettimies eläisi maan aikaa.

Kiitos mistral. Siitä, että pääsit ehkä jujulle mitä ajan takaa. Sitä, että rakettimies tietää (tarkoitan: voi laskea) mitä Maan kello kulloinkin näyttää ja mikä on raketin kulkema matka Maan koordinaatistossa. En ollenkaan vänkää, että rakettimies alkaisi elämään Maan kellon vuorokausia tms. Uskon mukisematta, että rakettimies tietysti elää raketin kellon mukaisia päiviä.

Mutta kysynpä kaikilta, että onko juuri edellä esittämäni väite "rakettimies voi laskea..." mielestänne pätevä. Vastaus alkakoon Kyllä- tai Ei-sanalla. Sitten voi seurata, varsinkin jos Ei-vastaus, selitys missä kohtaa järkeilyni on väärä. Tai mitä tahansa lausumia tai linkkejä ST:stä minulle ja muillekin tiedoksi.
Otsikko: Vs: Epäselvyys suhteellisuusteorian aika ja etäisyys käsityksissä
Kirjoitti: mistral - 28.06.2023, 21:58:10
Kyllä rakettimies voi laskea jos tietää oman kiihtyvyyden, nopeuden ym. Tosin ristiriidasta ei pääse eroon jollei nyt joku nero keksi aivan uuden tavan ratkaista ristiriita. Ristiriitaa voi ehkä rinnastaa tähän:
Suurennuslasi laitetaan nurmikolle kyljelleen niin että molemmilta puolilta näkee läpi. A puolelta B:n ruoho on kontraktoitunut. B:n puolelta katsottuna A:n ruoho on kontraktoitunut. Tässä siis todellisuus ei ole muuttunut vaan ongelma ratkeaa valon optiikan kautta.

Suhteellisuudessa taas todellisuus on muuttunut, sitä on vaikeampi ratkaista.
Otsikko: Vs: Epäselvyys suhteellisuusteorian aika ja etäisyys käsityksissä
Kirjoitti: Kaizu - 28.06.2023, 22:20:42
Olisiko tuosta apua? https://fi.wikipedia.org/wiki/Lorentz-muunnos (https://fi.wikipedia.org/wiki/Lorentz-muunnos)

Kaizu
Otsikko: Vs: Epäselvyys suhteellisuusteorian aika ja etäisyys käsityksissä
Kirjoitti: mistral - 29.06.2023, 19:31:53
Lainaus käyttäjältä: Kaizu - 28.06.2023, 22:20:42
Olisiko tuosta apua? https://fi.wikipedia.org/wiki/Lorentz-muunnos (https://fi.wikipedia.org/wiki/Lorentz-muunnos)

Kaizu

Lorentz muunnos on kerroin ykkösen ja äärettömän välillä kun newtonilaisuudessa ei ole sitä puolta. Siksi Newtonin avaruus on ristiriidaton. Tällä en tarkoita että Newtonin olisi parempi, kyllä suhteellisuus on parempi mutta kuka onnistuu purkamaan sen ristiriidat? Tähän asti ihminen on epäonnistunut siinä, tai sitten ristiriidat kuuluukin olla siellä?
Otsikko: Vs: Epäselvyys suhteellisuusteorian aika ja etäisyys käsityksissä
Kirjoitti: velihopea - 02.07.2023, 22:30:30
LainaaOlisiko tuosta apua? https://fi.wikipedia.org/wiki/Lorentz-muunnos

Kiitos Kaizu linkintä, mutta sivu ei auttanut minua. Minusta sivu otti lähtökohdakseen "näin se vaan on ST:ssa" ja esitti matemaattisessa esitysmuodossa kaavat Lorentz-muunnokselle.

Minulla on ymmärrys ja laskutapa L-muunnokselle ST:ssä, että tasaisesti liikkuva kappale ikäänkuin litistyy liikkeen suunnassa ja kappaleen kulkema matka lyhenee. Kuin paljon? Samassa suhteessa kuin kappaleen mukana olevan kellon käynti hidastuu.

-----
Yleisesti ottaen minusta tuntuu, että en ole saanut (paitsi mistralilta johonkin kohtaan) vastakaikua ajatukselle, mutta en vastaväitteitäkään, että voisiko seuraava ajatuskulku olla pätevä (esimerkkinä se rakettimatka Maasta Siriukseen ja takaisin 8vv päähän 0.8c nopeudella):

Rakettimies tietää nopeuseronsa Maahan olevan 0.8c, ja että se hidastaa raketin kellon käynnin 0.6-osaan. Kaavalla raketin kelloaika/0.6 hän voi aina laskea paljonko Maaveljen kello näyttää. Ja kaavalla (raketin kelloaika/0.6)x0.8c hän voi laskea missä kohtaa hän on menossa Maaveljen mielestä.

Nyt miksei se rakettimies vain toteaisi, että hänen kellonsa käynti on hidastunut, ja hän todella on siinä tietyssä kulloinkin laskemassaan kohdassa Maa-Sirius taipaleella. Ajan kulun suhteenkin olisi minusta jotenkin parempaa realismia rakettimiehen ymmärtää, että 20v siihen matkaan meni, mitä osoittaa mm. ne 20 vastaanotettua "nyt se vuosi taas meni"-radioviestiä Maasta. Se että rakettimies ei ole vanhentunut niin paljoa kuin Maaveli, johtuu siitä, hän on elänyt hitaammin (kuten kellonsakin) kuin Maaveli.

Kellon käynnin hidastuminen on todettu kokein, mm. kelloja lennättämällä. Pituuskontraktiota ei ole kokein todettu (Heikki Oja: Einsteinin perintö, Ursa 2021, s 43).

Edellinen on kai kerettilästä ajattelua ST:tä kohtaan. Siis, että oikeasti (jos Maan koordinaatistoa pidetään "oikeana") matka kesti 20v ja matkan pituus oli 16vv, ja rakettimieskin voi olla samassa käsityksessä. Missä paikassa skenaarioni ontuu? En kai minä ole ainoa ihminen maailmassa, joka on tällaista spekuloinut.

Alan jo toistamaan itseäni, joten ehkä lopetan osaltani tämän aiheen. Mutta kun olen lueskellut eri tekstejä ja katsellut eri videoita, joitakin sivukysymyksiä saatan julkaista lukijoiden iloksi ja kommentoitavaksi.

Apropos, missä koordinaatistossa NASA laskee pitkän matkan satelliittien radat. Laskut on varmaan vaikeita kun gravitaatio otetaan mukaan ja satelliitti hakee lisävauhtia toisten planeettojen gravitaatiolingoista jne. Täysin maan koordinaatistonko suhteen vaikka raketilla olisi mikä ST-käsityksensä ajasta ja kuljetusta matkasta?
Otsikko: Vs: Epäselvyys suhteellisuusteorian aika ja etäisyys käsityksissä
Kirjoitti: mistral - 03.07.2023, 00:23:12
velihopea:
Rakettimies tietää nopeuseronsa Maahan olevan 0.8c, ja että se hidastaa raketin kellon käynnin 0.6-osaan. Kaavalla raketin kelloaika/0.6 hän voi aina laskea paljonko Maaveljen kello näyttää. Ja kaavalla (raketin kelloaika/0.6)x0.8c hän voi laskea missä kohtaa hän on menossa Maaveljen mielestä.

Luulisin että tuo toimii. Jos oletetaan että maaveljen kello näyttää vaikka 3 vuotta niin mitä näyttäisi maaveljen kello sitten jos rakettimies näkisi sen myös reaaliaikaisesti (valon nopeus olisi ääretön)? Se ei näyttäisi 3v vaan jotain 2v! Se "Lorentzin suurennuslasi" tekee ristiriidan. Eli aina lasin toisella puolella on relativistiset efektit, ei koskaan lasin omalla puolella. Siis todellisuudet menee ristiin, Einstein sanoi aika-avaruutta nilviäiseksi koska se venyy joka suuntaan.

Kaksosparadoksista on eri ratkaisuja juuri siksi kun se on niin vaikeatajuinen.
Otsikko: Vs: Epäselvyys suhteellisuusteorian aika ja etäisyys käsityksissä
Kirjoitti: velihopea - 04.07.2023, 21:59:11
LainaaLuulisin että tuo toimii. Jos oletetaan että maaveljen kello näyttää vaikka 3 vuotta niin mitä näyttäisi maaveljen kello sitten jos rakettimies näkisi sen myös reaaliaikaisesti (valon nopeus olisi ääretön)? Se ei näyttäisi 3v vaan jotain 2v! Se "Lorentzin suurennuslasi" tekee ristiriidan. Eli aina lasin toisella puolella on relativistiset efektit, ei koskaan lasin omalla puolella. Siis todellisuudet menee ristiin, Einstein sanoi aika-avaruutta nilviäiseksi koska se venyy joka suuntaan.

Kiitos mistral taas vastauksesta. Tosin konseptia "lorentzin suurennuslasi" en ymmärrä, edes googlen avulla. Ja eikös "relativistiset efektit" (relativity?), jotenkin liity yleiseen ST:hen ja gravitaatioon. Kun esimerkissäni pelataan vielä suppean ST:n puolella.

Mutta tuo 3v versus 2v, riippuen kummalta puolelta katsotaan, on minulle kuin syöttö suoraan lapaan. Ehkä olisin aloittanut siitä oman keskustelun, vaikka otsikolla "Onko kellon käynti symmetrinen eri liiketiloissa?", mutta jatkan nyt vielä tähän.

Siis meillä olisi tuollaiset ihmekiikarit (mutta sellaisia ei ole), ja:
- Maan kellon lyötyä 3v Maamies kiikaroi rakettiin ja näkee sen kellon olevan 2v.
- raketin kellon lyötyä 3v rakettimies kiikaroi Maahan ja näkee sen kellon olevan 2v.

Tämä on kai ST-käsitys kellon käynnistä Maassa (suhteessa raketin kelloon) ja raketissa (suhteessa Maan kelloon). ST-ajatus ymmärtääkseni kulkee niin, että kun on liiketilat K (paikallaan) ja K' (liikkuu) on saman arvoista pitää K:ta liikkuvana ja K':a paikallaan olevana, ja kellojen käyntinopeudet liiketiloissansa seuraavat sitä kummin päin asia katsotaan. En osta tuota järkeilyä. Perusteluni pohjaksi kaivan esiin aiemman esimerkkini:

LainaaOn suora tasainen maantie ja sen laidassa 2 tolppaa tarkasti 1 km etäisyydellä toisistaan. Kumpaankin tolppaan kiinnitetään tarkka atomikello ja ne synkronoidaan näyttämään samaa aikaa.

    On kolmaskin yhtä tarkka atomikello. Se on autossa. Auton ajaessa jomman kumman tolpan ohi tolpan kello ja auton kello rekisteröivät mitä lukemaa kumpikin kello näyttää [takoitan: oman kellon lukeman oman kellon kovalevylle]. Ehkä joku lasersäde tolpan kohdalla triggeröi ajanoton.

Auto on ottanut 60km/h vauhdin ja ajaa sillä vauhdilla tolppien välin. On maantie-liiketila (havainnoitsijana tien laidassa katseleva henkilö) ja auto-liiketila (havainnoitsijana auton kuski), joiden nopeusero on 60km/h. Testin aikana kumpikaan ei vaihda liiketilaansa. Kuvitellaan, että tuon 1km testimatkan jossain vaiheessa auton kuski toteaa, kun auto rullaa tasaisesti ja maisemat vaihtuvat, ja hänpä muuttaa katsantokantaansa. Että auto onkin paikallaan ja maantie maisemineen ja tolppineen liikkuu. Joten ST-ajattelun mukaan pitäisi (muka) liikkuvan maantie-liiketilan kellon olla jätättänyt (muka) paikallaan ollutta auto-liiketilan kelloon nähden.

Kun testiajo on ohi tolppien kellojen kovalevyltä puretaan lähtö- ja tuloajat ja saadaan ajon kesto maantie-liiketilan mukaan, olkoon se vaikka 1min. Kun auton kellosta puretaan lähtö- ja tuloaika ja saadaan ajoaika, se on hieman alle 1min. Kai tästä ollaan samaa mieltä? Ja eikös koe vastaa suurinpiirtein 1971 tehtyä kellojen lennätystä.

Mutta jatketaan. Tolppakellojen ajoaika ja autokellon ajoaika jaetaan molemmat 1000:een osaan (ikäänkuin ajoaika kunkin metrin kohdalla) ja paritetaan keskenään. Minkään metrin kohdalla ymmärtääkseni tolppamittaus ei osoita hitaampaa aikaa vaikka kuski olisi kuinka meditoinut ja muuttanut katsantokantaansa!

JYU:n videossa puhuttiin myös tästä aiheesta rakettimatka Jupiteriin ja takaisin esimerkillä.

Ensin rakettimiehen koordinaatistoa esiteltiin siten, että raketti lilluu paikallaan avaruudessa ja Maa ja muu avaruus liikkuu edestakaisin raketin suhteen. Huh, vaikea järjestää tuollainen koetilanne...

Jos rakettimies ja Maamies alkaisivat kinaamaan kumman kello kello käy toisen kelloa nopeammin/hitaammin, kuitattiin videossa pois toteamalla, että rakettimies ei pysynyt koko matkaa samassa liikekoordinaatistossa. Ok, samoin selitetään monessa muussakin tekstissä. Mutta tässä auto-esimerkissä, kuski oli koko testin ajan siinä yhdessä koordinaatistossa...

Video sanoo myös: "kelloja voi verrata toisiinsa ainoastaan samassa koordinaatistossa". Esittämässäni auto-kokeessa yhdet kelloajat ovat todellisia maantieltä, toiset todellisia autosta. Samoin kuin kellojen lennätyksessä, saatuja mittaustuloksia verrataan kaikessa rauhassa kokeen jälkeen.

----
Minun päähäni nyt voisi mennä seuraava käsitys: Avaruudesta ei voi osoittaa paikkaa, mikä olisi paikallaan. Ainehippukasaumat, jotka kokonaisuutena liikkuvat yhdessä ovat liikeympäristöjä. Avaruuden kontekstissa kaikki liikeympäristöt liikkuvat jollakin nopeudella 0-c. Jos on liikeympäristöt K ja K', ja kun tiedetään, että K' liikkuu nopeammin kuin K, kaikkien on tyytyminen siihen tosiasiaan, että nopeammassa liikeympäristössä kellon käynti on hitaampaa.

Esimerkiksi kenenkään on turha katsantokanta-teoretisoida, että raketti onkin paikallaan ja Maa loittonee siitä 0.8c nopeudella (ja esittää että raketin kello käy nopeammin kuin Maan kello). Jos joku saisi tuon asianlaidan ihan aikuisten-oikeasti voimaan, niin silloin Maan ja raketin kellojen käyntisuhteet vaihtaisivat paikkaa.

Huh. Taisinpa taas eksyä tyrkkimään yhtä ST-rakennelman pilaria. Odotan mielenkiinnolla miten käsityksiäni korjataan ja miksi autoesimerkki ei ole todistusvoimainen.

Otsikko: Vs: Epäselvyys suhteellisuusteorian aika ja etäisyys käsityksissä
Kirjoitti: Mare Nectaris - 05.07.2023, 10:57:25
Klassisen mekaniikan, fysiikan tai aikakäsityksen avulla näitä ei voi tarkastella oikein. Juuri siksi Einstein korvasi klassisen mekaniikan laskut ajan absoluuttisuuden hylkäävällä Lorenz-muunnoksella.

Yhdeksi esimerkiksi laskutapojen erosta sopii kiista, kuinka paljon auringon painovoimakenttä siirtää tähden paikkaa maasta katsoen, kun tähden valo tulee havainnoitsijalle tähden ollessa maasta katsoen auringon painovoimakentän vaikutuksessa.

Klassisen newtonilaisen mekaniikan kautta laskettuna  tähden paikan pitäisi siirtyä 0,87". Einsteinilaisen laskuopin mukaan 1,75".

Auringonpimennystä 29.5.1919 seurannut retkikunta onnistui ottamaan valokuvan Hyadien tähdistöstä pimennyksen aikana. Valokuvauslevyltä mitattujen tähtien paikkoja verrattiin tilanteeseen, jolloin tähden valo ei saavu maapallosta katsoen painovoimakentän vaikuttamana. Osoittautui, että Einsteinin laskutapa oli oikeassa. Klassisen mekniikan mukainen laskutapa ei antanut oikeaa empiiristä tulosta, koska se ei pystynyt huomioimaan avaruuden kaareutumista.

Asiasta voi lukea tarkemmin täältä:

Dyson, F.W., Eddington, A.S. & Davidson, C. (1920) A Determination of the Deflection of Light by the Sun's Gravitational Field, from Observations Made at the Total
Eclipse of May 29, 1919. Philosophical Transactions of the Royal Society of London. Series A, Containing Papers of a Mathematical or Physical Character. Vol
220 (1920), ss. 291-333.

Linkki artikkeliin:

https://royalsocietypublishing.org/doi/abs/10.1098/rsta.1920.0009

Klassinen mekaniikka ja klassiset koejärjestelyt absoluuttisine laskutapoineen eivät siis valitettavasti riitä. Juuri siinä on koko suhteellisuusteorian juju: pitää käyttää soveltuvia laskutapoja, ei klassisen mekaniikan ja ajattelutavan mukaisia.

Einsteinin teoriat ja laskutavat on varsin usein osoitettu empiirisesti oikeiksi. Täältä voi asiaa tutkia: https://kauppa.ursa.fi/kauppa/tuote/einsteinin-perinto/

Otsikko: Vs: Epäselvyys suhteellisuusteorian aika ja etäisyys käsityksissä
Kirjoitti: mistral - 05.07.2023, 13:04:51
Lainaus käyttäjältä: velihopea - 04.07.2023, 21:59:11
Kiitos mistral taas vastauksesta. Tosin konseptia "lorentzin suurennuslasi" en ymmärrä, edes googlen avulla. Ja eikös "relativistiset efektit" (relativity?), jotenkin liity yleiseen ST:hen ja gravitaatioon. Kun esimerkissäni pelataan vielä suppean ST:n puolella.

Lorentzin suurennuslasi on vain oma havainnollistus. Lorentzin suurennuslasi = Lorentzin muunnos = Lorentzin kerroin.
Käytin siksi havainnollistuksena suurennuslasia koska se (1) muuntaa samalla kertoimella katsoipa sitä kummalta puolelta tahansa ja (2) lasin omalla puolella kaikki on muuntumatonta, tämä on hyvä muistisääntö. Tämä sama kerroin pätee myös erityisessä suhteellisuudessa. Eli jos K:n ja K':n nopeusero on 0,8c niin molemmat näkee toisensa samalla kertoimella muuntuneina.

  velihopea:
Siis meillä olisi tuollaiset ihmekiikarit (mutta sellaisia ei ole), ja:
- Maan kellon lyötyä 3v Maamies kiikaroi rakettiin ja näkee sen kellon olevan 2v.
- raketin kellon lyötyä 3v rakettimies kiikaroi Maahan ja näkee sen kellon olevan 2v.

Tämä on kai ST-käsitys kellon käynnistä Maassa (suhteessa raketin kelloon) ja raketissa (suhteessa Maan kelloon). ST-ajatus ymmärtääkseni kulkee niin, että kun on liiketilat K (paikallaan) ja K' (liikkuu) on saman arvoista pitää K:ta liikkuvana ja K':a paikallaan olevana, ja kellojen käyntinopeudet liiketiloissansa seuraavat sitä kummin päin asia katsotaan. En osta tuota järkeilyä.


Ei ole mitenkään itsestäänselvää ostaa tuo järkeily, se on ihan oikea asenne. Itselle sen sulattaminen on ollut vaikeaa. Olen kuitenkin tällä hetkellä sulattanut sen juuri suurennuslasi-idean vuoksi. Kun tehdään taulukko eri nopeuksista niin ehkä siitä näkee syyn:

K 0c <-------> K' 0,8c
K 0,2c <--------> K' 0,6c
K 0,4c <--------> K' 0,4c
K 0,6c <--------> K' 0,2c
K 0,8c <--------> K' 0c

Yhteistä noille on se sama suurennuslasi, molemmat näkee toisensa joka kohdassa samalla kertoimella No onko tämä lopullinen johtopäätös? En tiedä mutta varmaa on että suurennuslasi on sama. Ehkä joku nero keksii ulospääsyn tästä todellisuuksien ristiriidasta. Ja onhan pieni yritys jo olemassa: apukoordinaatisto puolessavälissä antaa saman symmetrisen ajankulun molemmille mutta onko se todellinen ratkaisu?

velihopea:
Video sanoo myös: "kelloja voi verrata toisiinsa ainoastaan samassa koordinaatistossa". Esittämässäni auto-kokeessa yhdet kelloajat ovat todellisia maantieltä, toiset todellisia autosta. Samoin kuin kellojen lennätyksessä, saatuja mittaustuloksia verrataan kaikessa rauhassa kokeen jälkeen.


Juuri todellisuuksien ristiriita estää vertaamasta kellonaikoja, vasta kun ovat samassa liiketilassa, voidaan verrata.

Vielä Einsteinin oma ratkaisu kaksosparadoksille: "Dialog about relatitivity theory" sivut 66-75?
https://einsteinpapers.press.princeton.edu/vol7-trans/82
Tämä ratkaisu on aivan erilainen kuin valtavirran käyttämä, tässä Einstein hyödyntää yleistäsuhteellisuusteoriaa.

Otsikko: Vs: Epäselvyys suhteellisuusteorian aika ja etäisyys käsityksissä
Kirjoitti: Eusa - 06.07.2023, 04:47:50
Miksi raketin lähettämiä 12 v signaaleja ei voisi pitää standardina ja muuttaa Maassa vuodenjuoksut sen mukaisiksi?

Edellinen kysymys osoittaa kuinka absurdia on pitää mitään aikaa muita "parempana".

Aikaeron varsinaisesti aiheuttaa nimenomaan gravitaatio - eli matkustajan takaisin kääntymisen kiihdytysvaiheen muodostama gravitaatio.
Otsikko: Vs: Epäselvyys suhteellisuusteorian aika ja etäisyys käsityksissä
Kirjoitti: velihopea - 07.07.2023, 12:03:48
Kiitokset kaikille kommentoijille/Mare Nectaris, mistral ja Eusa.

LainaaKlassisen mekaniikan, fysiikan tai aikakäsityksen avulla näitä ei voi tarkastella oikein.

Minua olisi auttanut, ja kenties jotain muutakin lukijaa, jos kirjoittamistani tapahtumaskenaarioista olisi selvästi osoitettu kohtia, joissa järkeily perustuu sellaisiin klassisen mekaniikan käsityksiin, jotka eivät pidä paikkansa, ja miksi ei.

Linkki auringon taivuttamasta tähden valosta ei ole nyt mielenkiintoni kohteena. Tuon Heikki Oja: Einsteinin perintö -kirjan olen lukenut alleviivaillen useampaan kertaan.

Lainaa
LainaaTämä on kai ST-käsitys kellon käynnistä Maassa (suhteessa raketin kelloon) ja raketissa (suhteessa Maan kelloon). ST-ajatus ymmärtääkseni kulkee niin, että kun on liiketilat K (paikallaan) ja K' (liikkuu) on saman arvoista pitää K:ta liikkuvana ja K':a paikallaan olevana, ja kellojen käyntinopeudet liiketiloissansa seuraavat sitä kummin päin asia katsotaan. En osta tuota järkeilyä.

Ei ole mitenkään itsestäänselvää ostaa tuo järkeily, se on ihan oikea asenne. Itselle sen sulattaminen on ollut vaikeaa. Olen kuitenkin tällä hetkellä sulattanut sen juuri suurennuslasi-idean vuoksi. Kun tehdään taulukko eri nopeuksista niin ehkä siitä näkee syyn:

K 0c <-------> K' 0,8c
K 0,2c <--------> K' 0,6c
K 0,4c <--------> K' 0,4c
K 0,6c <--------> K' 0,2c
K 0,8c <--------> K' 0c

Sorry mistral, pääni taitaa olla umpiluusta ja lisäksi lienen sokea. Mikä syy kustakin rivistä pitäisi voida nähdä.

LainaaJuuri todellisuuksien ristiriita estää vertaamasta kellonaikoja, vasta kun ovat samassa liiketilassa, voidaan verrata.

Eikös ne erilaiset rakettiesimerkit julista tätä symmetriaakin, että (jos olisi valoa nopeammat kiikarit) kun Maan kello tulee 3v, kiikaroitu raketin kello näyttää 2v. Ja sama toisinpäin. Niissäkin esimerkeissä liiketilat ovat vielä erilliset. Maantie/auto -esimerkissä koetin saada järjestelyn, jossa maantien tolpat (maantie liiketilassa) ja auto (auto liiketilassaan) mittaavat ja kirjaavat tolppien välisen ajoajan. Ja väitin, että tolppakellojen mittaama ajoaika on aina suurempi kuin auton mittaama. Eli missään kohtaa ajoa ei voi olla tilannetta, että autosta(raketista) katsottaessa tolppakelloon(Maahan), että auton kellon mukaan matkaa olisi tehty joku aika(3v) mutta tolppakellon mukaan vähemmän(2v). Voisiko joku tehdä tällaisen maantie/auto -kokeen, vai onko ilmiö (tolppakello jätättää) jo todennettu?

LainaaVielä Einsteinin oma ratkaisu kaksosparadoksille: "Dialog about relatitivity theory" sivut 66-75?
    https://einsteinpapers.press.princeton.edu/vol7-trans/82
    Tämä ratkaisu on aivan erilainen kuin valtavirran käyttämä, tässä Einstein hyödyntää yleistäsuhteellisuusteoriaa.

Kiitos mistral tästä linkistä. Teksti on Einsteinin keksimä keskustelu, jossa ST-epäilijä esittää ihmettelyjä (luokkaa: Maa ja raketti näkevät toisen kellon jätättävän samalla tavalla). ST-puolustaja (kai AE itse) selittää asian askel askeleelta. Juuri tällaista olen kaivannut. Mutta tosiaan, selityksessä mukaan tuli gravitaatio (kiihtyvyys) yleisen ST:n puolelta. Pelikenttä tuli isommaksi. Ensilukemalta ei heti selvinnyt, miksi liiketilojen K ja K' (vuoronperään toinen paikallaan ja kappale liikkui edestakaisin) selitettiin eri lailla. Jään sulattelemaan.

LainaaMiksi raketin lähettämiä 12 v signaaleja ei voisi pitää standardina ja muuttaa Maassa vuodenjuoksut sen mukaisiksi?

Ensin oikein pysähdyin kysymyksesi kohdalla. Mutta kai on parempi, että kukin liiketila pitää oman kellonsa kulun. Kun raketissa kellon käynti on hidastunut, samoin on hidastunut rakettimiehen elintoiminnot. Hän voi elää kellonsa mukaisi päiviä, mitkä Maan kellon mukaan ovat a' 40h (jos 0.8c). Ihmiset maassa eivät voi alkaa elää 40h päivillä. Ja hommat menisi sekaisin, jos lunta sataisi milloin mihinkin aikaan vuodesta. Rakettimieskin virkistyy toimimaan taas Maan 24h päivillä kunhan palaa matkaltaan.
Otsikko: Vs: Epäselvyys suhteellisuusteorian aika ja etäisyys käsityksissä
Kirjoitti: mistral - 07.07.2023, 15:30:59
Lainaus käyttäjältä: velihopea - 07.07.2023, 12:03:48

Ei ole mitenkään itsestäänselvää ostaa tuo järkeily, se on ihan oikea asenne. Itselle sen sulattaminen on ollut vaikeaa. Olen kuitenkin tällä hetkellä sulattanut sen juuri suurennuslasi-idean vuoksi. Kun tehdään taulukko eri nopeuksista niin ehkä siitä näkee syyn:

K 0c <-------> K' 0,8c
K 0,2c <--------> K' 0,6c
K 0,4c <--------> K' 0,4c
K 0,6c <--------> K' 0,2c
K 0,8c <--------> K' 0c

Sorry mistral, pääni taitaa olla umpiluusta ja lisäksi lienen sokea. Mikä syy kustakin rivistä pitäisi voida nähdä.

Ai niin K:n nopeuden edessä olisi saanut olla miinus niin näkyisi nopeuden suunta.

Kuitenkin nopeuseron oli tarkoitus olla 0,8c kaikissa. On yhdentekevää kuka on levossa ja kuka liikkuu, vain kahden koordinaatiston suhde toisiinsa ratkaisee.
Suhtis siis on teoria ennenkaikkea kahden välillä mutta se ei estä sitä että suhteita on ääretön määrä, siis kahdenvälisiä. Teoria ratkaisee kahdenvälisiä asioita eikä osaa yhdistää kokonaisuuksia.
Esim minun suhde nopeaan tähteen on voimassa yhtälailla kuin suhteeni nopeaan elektroniin, kvasaarin ainesuihkuun jne. Jokaisen mielestä kelloni käy eri tahtiin. Kuitenkin jokainen suhde on yhtä totta vaikka kelloni näyttäisi käyvän eri tahtiin jokaisen mielestä. Eli kokonaisuus on ikäänkuin mahdotonta sovittaa samaan kuvaan, on mahdotonta laittaa kellotaulu täyteen eri aikoja.
Jospa joku keksisi ratkaisun tähän ongelmaan.
Otsikko: Vs: Epäselvyys suhteellisuusteorian aika ja etäisyys käsityksissä
Kirjoitti: velihopea - 25.07.2023, 17:34:31
Siitä kun viimeksi (07-07-2023, 12:03:48) kirjoitin tänne, olen lomaillut ja sulatellut asioita. Varsinkin mistralin linkki 'Einsteinin oma selitys kaksosparadoksille: "Dialog about relatitivity theory" sivut 66-75, https://einsteinpapers.press.princeton.edu/vol7-trans/82' oli minulle "silmiä avaava". AE selittää perustilanteen, jossa on ympäristö K jossa kello U1 (paikallaan) ja ympäristö K' jossa kello U2 (liikkuu). Eli se usein vatvottu raketilla-Siriukseen-ja-takaisin tapaus.

Sen jälkeen AE selittää "symmetriatilanteen", jossa K':n oletetaan olevan paikallaan ja K liikkuu. Molemmissa tilanteissa U2 kello on jätättänyt! Mutta symmetriatilanteessa tapahtumien kulku ja osallistuvat voimat ovat erilaiset kuin perustilanteessa.

AE kuvaa tapahtumien kulun perustilanteessa seuraavasti:

Massakappaleet U1 ja U2, jotka ovat samakäyntisiä kelloja, ovat xyz-koordinaatiston x-akselilla paikallaan pisteessä A. U2 liikkuu +x-akselin suuntaan vakionopeudella v pisteeseen B, kääntyy ja tulee takaisin samalla v nopeudella pisteeseen A ja pysähtyy. Tapahtuu seuraavaa:

Vaihe 1: Ulkoinen voima kiihdyttää U2:ta +x-akselin suuntaan kunnes U2 on saavuttanut nopeuden v. (Ei tapahdu silmänräpäyksessa, mutta oletetaan, että niin nopeasti kuitenkin, että vaikutus U2:n käyntiin kiihdytysaikana  ei kaada isoa kuvaa)
[käsitykseni: kiihdytyksen energia siirtyy U2:n liike-energiaksi, ja kiihdytyksen päätyttyä U2:n käynti on hidastunut tunnetun kaavan mukaiseksi]

Vaihe 2: U2 liikkuu vakionopeudella v (ja käy saamaansa hidastunutta käyntiä) kunnes se tulee sopivan lähelle B:tä.

Vaihe 3: Ulkoinen kiihdytysvoima kohdistuu U2:een -x-akselin suuntaan (jarrutus, pysähtyminen, kiihdytys) kunnes U2 on saavuttanut nopeuden v -x-akselin suuntaan.
[käsitykseni: jarrutusvaiheessa U2:n saama liike-energia purkautuu (yleensä lämmöksi) ja kun se kaikki on purkautunut U2:n käynti on palautunut samaksi kuin U1:n, kunnes kiihdytysvaiheessa U2:een siirtyy taas liike-energiaa ja sen käynti taas hidastuu]

Vaihe 4: U2 liikkuu vakionopeudella v takaisinpäin (ja se käy saamaansa hidastunutta käyntiä) kunnes se tulee sopivan lähelle A:ta.

Vaihe 5: U2:een kohdistuu ulkoinen voima, joka jarruttaa U2:n ja pysäyttää sen A:ssa U1:n viereen.
[käsitykseni: jarrutuksessa U2:n liike-energiaa purkautuu ja kun se on kaikki purkautunut U2:n käynti on palautunut samaksi kuin U1:n].

Lopputulos on, että palattuaan lenkiltään U2 on jätättänyt U1:een nähden.

---------
Havaintojani/kommenttejani:

1. Alkujaan 1905 AE selosti tapahtuman niin ("A:n perintö", s. 37), että kun vierekkäisistä kelloista toinen liikkuu suljettua käyrää pitkin vakionopeudella ja palaa takaisin, se on jäänyt jälkeen paikallaan pysyneestä kellosta. Vakionopeusvaatimushan kieltää pysähtymisen ja kääntymisen takaisin. Jos taas lenkki olisi jotenkin ympyrä, peliin astuu keskipakovoima, vähän niinkuin gravitaatio, ja se sotkisi tilanteen. Jossain muistan lukeneeni lisävaatimuksen "...kulkee suoraa linjaa pitkin...", mikä kieltää kaartelun. Nyt tämä AE:n 2018 esimerkin realistisuus käy järkeeni, 1905 kuvaus ei.

2. Panen merkille, että kun U2:een on tankattu liike-energiaa, U2-kellon käynti on hidastunut. Jos U2 olisi cesium-133:n varaan rakennettu atomikello, mikä ihme mekanismi aiheuttaa sen, että kellon cesium-klöntin atomit jotenkin laiskistuu ja päästävät ulos fontteja harvempaan tahtiin. Eli emittoitu säteily on pienempi-energistä eli isommalla aallonpituudella. Ja kait tuosta aallonpituudesta se atomikellon muu mekanismi jotenkin saa tick-arvonsa. Osaako joku selittää? Selitys saattaa mennä kvanttimekaniikan puolelle.

Tai jos K'-liiketilassa on myös rakettimies. Kun kaikki K'-liiketilan massojen atomitason prosessit ovat hidastuneet, myös rakettimiehen elintoiminnot ovat hidastuneet samassa suhteessa.

Jos selitys on, niinkuin epäilen, että atomien energiatilan muutos jotenkin aiheuttaa atomin emittoiman säteilyn aallonpituuden muutoksen, tulkitsen, että tarkka atomikello ansiokkaasti vain kirjaa tilanteen. Eli kello näyttää mitä kello näyttää. Se olisi eri asia miten aika kuluu. Joskin löysässä arkipuheessa on tapana sanoa, että aika kuluu siinä tahdissa mitä oma kello näyttää.

3. Panen merkille, että jo perustilanteen selityksessä AE nojaa kiihtyvyyteen. Sehän on vähän samaa asiaa kuin gravitaatio. Vetovoimakentässä massakappaleella on kiloa kohden eri määrä potentiaalienergiaa riippuen siitä onko kappale lähellä massakeskipistettä vai kaukana siitä. Kun liikkeessä kappaleen liike-enrgia näyttää olevan yhteydessä kellon käyntiin, olisikohan gravitaatiossa kappaleen potentiaalienergialla samanlainen yhteys. Eli kellon käynnin määräisi aineen atomien energiatilojen vaihtelu.

Varoitus: voihan olla että jotkut esittämäni asiat eivät varmasti pidä paikkansa tai ovat perusteettomia spekulaatioita. Toivottavasti jotkut paremmin tietävät osoittavat mitä ne kohdat ovat.
Otsikko: Vs: Epäselvyys suhteellisuusteorian aika ja etäisyys käsityksissä
Kirjoitti: velihopea - 25.07.2023, 22:37:46
Kerronpa myös ymmärryksi miten AE kuvaa "symmetriatilanteen", jossa K (jossa on kello U1) liikkuu ja K' (jossa on kello U2) on paikallaan. Tapahtumien kulku on:

Vaihe 1: Nousee gravitaatiokenttä -x-akselin suuntaan, jolla kello U1 kiihtyy kunnes se saavuttaa nopeuden v. Hetkellinen ulkoinen voima +x-akselin suuntaan vaikuttaa U2:een ja estää sitä liikkumasta. Ilmaantunut gravitaatiokenttä katoaa kun U1 on saanut nopeuden v.

Vaiha 2: U1 liikkuu vakionopeudella kunnes se saavuttaa pisteen B' (~vastinpiste B:lle, mutta x-akselin toisella puolella). U2 pysyy paikallaan.

Vaihe 3: Nousee samanlainen gravitaatiokenttä +x-akselin suuntaan, jolla U1 jarruttaa, pysähtyy ja kiihdyttää kunnes U1 on saanut nopeuden v +x-akselin suuntaan, jolloin gravitaatiokenttä katoaa. Hetkellinen ulkoinen voima kohdistuu U2:een ja pitää sen paikallaan tuon gravitaatiokentän aikana.

Vaihe 4: U1 liikkuu nopeudella v +x-akselin suuntaan. U2 on paikoillaan.

Vaihe 5: Nousee gravitaatiokenttä -x-akselin suuntaan, joka tuo U1:n alkuperäiselle paikalleen liikkumattomaksi, jolloin tuo gravitaatiokenttä katoaa. Tässä jarrutusvaiheessa ilmaantuu hetkellinen ulkoinen voima, joka pitää U2:n paikallaan.

Järjestelyn tarkoituksena oli vakuuttaa ST-epäilijä symmetrisyydestä, eli että "ei sen väliä kumman katsotaan olevan paikallaan/liikkuvan, se on silti kello U2, joka jätättää". Ja AE saa tuloksen, että U2 näyttää vähemmän kuin U1. Tulos kaipaa kuitenkin AE:ltä selityksen.

Selityksessä AE toteaa, että tässä symmetriatilanteessa on erilaiset prosessit pelissä kuin oli perustilanteessa. AE ei luettele niitä, mutta olen näkevinäni:
- perustilanteessa kello U2 saatiin vauhtiin, kääntymään ja pysähtymään kiihdyttämällä.
- symmetriatilanteessa kellolle U1 samat temput aikaansaatiin gravitaatiolla (ja pelikentälle astui gravitaatiopotentiaali!)

Ennen kun alan rusikoimaan koejärjestelyä enemmälti, koetan toistaa miten AE selittää mitä eri vaiheissa tapahtuu:

Vaiheissa 2 ja 4 todellakin U1, liikkuessaan nopeudella v, käy hitaammin kuin U2.

Mutta juju on vaiheessa 3. Helpompi siteerata: "But the time lag gets overcompensated by the faster rate of U1 during procedural step 3. Because, according to the general theory of relavitivity, a clock has a more accelerated rate the higher the gravitational potential is at the clock's location; and during procedural step 3, U2 is indeed at a location of higher gravitational potential than U1. Calculation shows that this running-ahead amounts to pricisely twice as much as the lag-behind during procedural steps 2 and 4."

Ja nyt havaintojani/kommenttejani:

1. Tuo AE:n toteamus "Calculation shows" saa minut näkemään punaista, ainakin tiputti minut ymmärryksen polulta. Siksipä kysyn tietäviltä lukijoilta, jotka voisivat kommentoida suunnilleen: "Joo, minä ymmärrän tämän. Ja U1-kellolle tuleva nopeampi käynti vaiheen 3 aikana todella tuplasti kompensoi sen jälkeenjääneisyyden, mikä on tullut matkalla A-B' ja vielä tuleva matkalla B'-A." Ja bonusta olisi: "Ja olen jopa toistanut tuon laskelman."

2. En mitenkään keksi mekanismia miten U1:n time-lag tulisi oikealla määrällä kompensoitua lyhyen vaihe 3:n aikana. Mutta ehkä AE tarkoittikin, että U1:n pohjakäynti oli kyllä hidastunut vaikkapa -1 sek/siivu, mutta U1:n gravitaatioposition vähittäinen muutos jo matkan aikana sai nopeutuksen, että U1 kävikin +1 sek/siivu. Mutta en saanut sitäkään mallia pelaamaan, koska nopeuden vuoksi kellon käynnin hidastuminen pysyy, mutta gravitaatioposition muutoksen käyntivaikutus ei liene lineaarinen.

3. Entä mitä ovat K ja K' ympäristöt. Kai K on U1 ja kaikki mitä on sen ympärillä, eli koko kosmos. Ja mistä voi tilata hetkittäin voimassa olevia gravitaatiokenttiä alkukiihdytykseen, kääntymiseen ja loppujarrutukseen. Ja voimia U2:n paikallaanpitämiseen. Voiko tällaisilla mahdottomilla skenaarioilla perustella asioita.

4. Ja eihän tilanne ole symmetrinen! Jos perustilanteessa U2 jätätti paikallaan ollutta tarkkakäyntistä U1:tä vaikkapa 2 sek, tässä tilanteessa paikallaan ollut tarkkakäyntinen U2 oli kyllä jäljessä U1:tä, mutta ei jätättämisen vuoksi vaan U1 oli edistänyt 2sek!

Sorry, AE:n selitys tuntuu minusta olevan soopaa.

Ehkä olen väärässä. Ja kuten aiemminkin toivon saavani ohjausta, missä kohden. Tai mitä tahansa kommentteja.
Otsikko: Vs: Epäselvyys suhteellisuusteorian aika ja etäisyys käsityksissä
Kirjoitti: mistral - 26.07.2023, 21:57:52
Lainaus käyttäjältä: velihopea - 25.07.2023, 22:37:46

Sorry, AE:n selitys tuntuu minusta olevan soopaa.

AE:n selitys on oikea mutta selitys on jäänyt muusta syystä pois kaksosparadoksista, syy on vissiin se ettei pseudogravitaatiokenttää voi hyväksyä ajan hidastamiseen.

AE:n selitys on se että K-koordinaatisto on U1:n näkökulma ja K'-koordinaatisto on U2:n näkökulma. Siinä kumpainenkin pitää itseään levossa. Ja kun U2 (Uhr2 eli kello2) pitää itseään levossa, niin koko universumin pitää liikkua ja kiihtyä, tämä on juuri pseudogravitaatiota. Miksi vaihe 3 tuottaa suurimman dilaation johtuu siitä että ps-grav kenttä on pisimmillään silloin, siis ps-grav-potentiaali on ylivoimaisesti suurimmillaan kaukaisimmassa pisteessä. (ps-grav kenttä on saman voimainen koko pituudeltaan ja kun pituutta lisätään rajattomasti, jossain kohtaa tulee vastaan tapahtumahorisontti jossa aika pysähtyy)
Tämä on mielestäni ihan oikein mutta ongelma on se ettei ps-grav kenttä ole todellisen väärtti. Toki tässä on kyse ekvivalenssiperiaatteesta mikä on totta. Siis totta on ettei putoavassa hississä oleva tiedä leijuuko paikallaan vai putoaako mutta ympäristö kertoo kyllä kummasta on kyse ja siksi ekvivalenssiperiaatteen soveltaminen ei mielestäni käy tässä kaksosparadoksissa.
Otsikko: Vs: Epäselvyys suhteellisuusteorian aika ja etäisyys käsityksissä
Kirjoitti: velihopea - 28.07.2023, 18:56:29
Kiitos mistral - 26-07-2023, 21:57:52 kun jaksoit kommentoida. Kommentit, millaiset tahansa, kummasti inspiroivat ajatuksia. Mutta kuten on kai käynyt ilmi, taidan olla kovapäinen, ja kommenttiesi jälkeenkin minulle jäi vielä ymmärtämisen nälkää.

Kopioin tähän koko vastauksesi ja upotan sinne kommenttini [vh ... vh] suluin.

    Lainaus käyttäjältä: velihopea - 25-07-2023, 22:37:46

        Sorry, AE:n selitys tuntuu minusta olevan soopaa.

AE:n selitys on oikea
[vh Kiitos selvästä vastauksesta vh]

mutta selitys on jäänyt muusta syystä pois kaksosparadoksista, syy on vissiin se ettei pseudogravitaatiokenttää voi hyväksyä ajan hidastamiseen.
[vh ps-grav-kenttä on minusta hieman kuin sadun taikuri, jonka kertoja tuo esiin sopivassa kohtaa tekemään sopivan tempun, jotta tarina etenisi halutulla tavalla. Mutta se on itse AE, joka tuo selitykseen tuon ps-grav-kentän, jollaista ei kait ole olemassa. Vai onko niin, että tämä symmetriatilanne on teoriassa olemassa (tuon ps-grav-kentän avustuksella), mutta ei käytännössä? vh]

    AE:n selitys on se että K-koordinaatisto on U1:n näkökulma ja K'-koordinaatisto on U2:n näkökulma. Siinä kumpainenkin pitää itseään levossa. Ja kun U2 (Uhr2 eli kello2) pitää itseään levossa, niin koko universumin pitää liikkua ja kiihtyä, tämä on juuri pseudogravitaatiota.
[vh Noista samaa mieltä. Mutta näkökulmasta käsitän, että sitä vaihdettaessa pitää  tapahtumaprosessi myös vaihtaa niinkuin näissä AE:n esimerkeissä on tehty, ei pelkästään tarkkailijan päässä vh]

Miksi vaihe 3 tuottaa suurimman dilaation johtuu siitä että ps-grav kenttä on pisimmillään silloin, siis ps-grav-potentiaali on ylivoimaisesti suurimmillaan kaukaisimmassa pisteessä.
[vh Minun tarvi itsellenikin selvittää käsitteitä: Kun AE sanoo tilanteesta käännöspisteessä B' "U2 is indeed at higher gravitational potential than U1", se merkitsee, että U1 on lähempänä massakeskipistettä (ikäänkuin maan pinnalla) ja U2 on kauempana siitä (ikäänkuin leijuu korkealla).

Kuvitellaan, että kellot U1 ja U2 ovat massaltaan a' 10 kg, koe tehdään mahdollisimman "gravitaatiovapaassa" osassa kosmosta (ei maan pinnalla) ja kosmosta nytkäytetään 1 km -x-akselin suuntaan ja takaisin. En oikein saa päähäni, että ovatko nämä kaksi kellonrääpälettä ne massaosapuolet, joiden välisestä gravitaatiopotentiaalierosta on kyse (sellainenhan niillä on). Vai onko niin, että kun muukin kosmos siirtyi 1 km -x-akselin suuntaan, U1:n puolella massavaikutus kasvoi muun kosmoksen ansiosta. Jos niin, sitten kysyn miksei massavaikutus voinutkin kasvaa U2:n puolella. Ja millä mekanismilla tuossa käännösvaiheessa U1:lle tulee juuri sopiva käynnin kirivaihe päälle? Ja AE-kuvauksesta päättelin, että nyt U1 edisti kun perustilanteessa U2 jätätti!  vh]

(ps-grav kenttä on saman voimainen koko pituudeltaan ja kun pituutta lisätään rajattomasti, jossain kohtaa tulee vastaan tapahtumahorisontti jossa aika pysähtyy)
[vh AE:n selostus selvästi sanoi, että ps-grav-kenttä ei ollut päällä vakionopeustaipaleilla. Ja tässä kokeessa ei menty tapahtumahorisonttien lähelle tms. vh]

    Tämä on mielestäni ihan oikein mutta ongelma on se ettei ps-grav kenttä ole todellisen väärtti. Toki tässä on kyse ekvivalenssiperiaatteesta mikä on totta. Siis totta on ettei putoavassa hississä oleva tiedä leijuuko paikallaan vai putoaako mutta ympäristö kertoo kyllä kummasta on kyse
[vh Miten "putoava hissi" tähän kuuluu? Yleisesti, ei kai tässä kosmoksessa ole yhtään paikkaa, joka olisi paikallaan ja jossa kappale voisi vain leijua. Tasaisessa liikkeessä voi olla liki huomaamattaan, mutta aikaa myöten lähikosmoksen vetovoimat muuttavat ja kiihdyttävät kyllä kulkua vh]

ja siksi ekvivalenssiperiaatteen soveltaminen ei mielestäni käy tässä kaksosparadoksissa.
[vh Onko muita mielipiteitä? vh]
Otsikko: Vs: Epäselvyys suhteellisuusteorian aika ja etäisyys käsityksissä
Kirjoitti: Mare Nectaris - 28.07.2023, 20:40:43
Pseudogravitaatio ei tarkoita turvautumista taikurin hattuun. Se on tapa erotella ajatuskokeessa ("Gedankenexperiment") kiihtyvyyden g-voima (suppean suhteellisuusteorian sovellusala) aika-avaruutta kaareuttavasta massan painovoimakentästä (yleisen suhteellisuusteorian sovellusala).

Auttaisiko tämä:

https://math.ucr.edu/home/baez/physics/Relativity/SR/sr-gr.html
Otsikko: Vs: Epäselvyys suhteellisuusteorian aika ja etäisyys käsityksissä
Kirjoitti: mistral - 28.07.2023, 22:15:33

velihopea:
[vh ps-grav-kenttä on minusta hieman kuin sadun taikuri, jonka kertoja tuo esiin sopivassa kohtaa tekemään sopivan tempun, jotta tarina etenisi halutulla tavalla. Mutta se on itse AE, joka tuo selitykseen tuon ps-grav-kentän, jollaista ei kait ole olemassa. Vai onko niin, että tämä symmetriatilanne on teoriassa olemassa (tuon ps-grav-kentän avustuksella), mutta ei käytännössä? vh]
Grav-kentän ja ps-grav-kentän ero on siinä että grav'issa valo punasiirtyy mutta ps-grav'issa ei löydy gravitaatiolähdettä joka tuottaisi punasiirtymän. Joten siksi se ei vastaa todellisuutta.
----------------
velihopea:
[vh Minun tarvi itsellenikin selvittää käsitteitä: Kun AE sanoo tilanteesta käännöspisteessä B' "U2 is indeed at higher gravitational potential than U1", se merkitsee, että U1 on lähempänä massakeskipistettä (ikäänkuin maan pinnalla) ja U2 on kauempana siitä (ikäänkuin leijuu korkealla).
Kyllä, mitä korkeammalla ollaan sitä suurempi gravitaatiopotentiaali.
1-vaihe: U2 kiihdyttää +suuntaan, siksi U1 on heikossa gravitaatiokuopassa
2-vaihe: ei gravitaatiota
3-vaihe: U2 jarruttaa+kiihdyttää takaisin, siksi U2 on nyt voimakkaassa gravitaatiokuopassa
4-vaihe: ei gravitaatiota
5-vaihe: U2 jarruttaa ja siksi U1 on taas heikossa gravitaatiokuopassa

Näin U2:n voimakas gravitaatiokuoppa tuottaa selvästi suuremman dilaation kuin U1:n heikot kuopat.
-----------------
velihopea:
Kuvitellaan, että kellot U1 ja U2 ovat massaltaan a' 10 kg, koe tehdään mahdollisimman "gravitaatiovapaassa" osassa kosmosta (ei maan pinnalla) ja kosmosta nytkäytetään 1 km -x-akselin suuntaan ja takaisin. En oikein saa päähäni, että ovatko nämä kaksi kellonrääpälettä ne massaosapuolet, joiden välisestä gravitaatiopotentiaalierosta on kyse (sellainenhan niillä on). Vai onko niin, että kun muukin kosmos siirtyi 1 km -x-akselin suuntaan, U1:n puolella massavaikutus kasvoi muun kosmoksen ansiosta. Jos niin, sitten kysyn miksei massavaikutus voinutkin kasvaa U2:n puolella. Ja millä mekanismilla tuossa käännösvaiheessa U1:lle tulee juuri sopiva käynnin kirivaihe päälle? Ja AE-kuvauksesta päättelin, että nyt U1 edisti kun perustilanteessa U2 jätätti!  vh]

Ei kellojen massoilla ole mitään merkitystä vaan ps-grav-kentällä. Dilaatio tulee suoraan verrannollisesti punasiirtymästä. Kun meren pinnasta lähetetään vaikka 600nm aalto ylöspäin ja oletetaan että kenttä säilyy yhtä voimakkaana läpi universumin, niin jossain korkeudella aalto olisi punasiirtynyt 700nm:iin. Tällä korkeudella kello kävisi vissiin 700/600=1,166 kertaa nopeammin kuin merenpinnalla. Tässä kiteytettynä gravitaatiopunasiirtymän mekanismi.
--------------------
velihopea:
[vh Miten "putoava hissi" tähän kuuluu? Yleisesti, ei kai tässä kosmoksessa ole yhtään paikkaa, joka olisi paikallaan ja jossa kappale voisi vain leijua. Tasaisessa liikkeessä voi olla liki huomaamattaan, mutta aikaa myöten lähikosmoksen vetovoimat muuttavat ja kiihdyttävät kyllä kulkua vh]

Ekvivalenssiperiaate on juuri se mihin Einstein perustaa kaksosparadoksinsa, siis sen avulla hän ottaa käyttöön ps-grav-kentän. Ja ajatus on varmaan oikea mutta siis kentältä puuttuu gravitaatiolähde eli suuri massa.
Otsikko: Vs: Epäselvyys suhteellisuusteorian aika ja etäisyys käsityksissä
Kirjoitti: mistral - 28.07.2023, 23:34:29
Tässä taas yksi selitys kaksosparadoksista:
https://www.google.com/search?client=firefox-b-d&q=don+lincoln+twinparadox#fpstate=ive&vld=cid:4ab785d2,vid:GgvajuvSpF4
Otsikko: Vs: Epäselvyys suhteellisuusteorian aika ja etäisyys käsityksissä
Kirjoitti: Eusa - 29.07.2023, 12:52:24
Lainaus käyttäjältä: mistral - 28.07.2023, 23:34:29
Tässä taas yksi selitys kaksosparadoksista:
https://www.google.com/search?client=firefox-b-d&q=don+lincoln+twinparadox#fpstate=ive&vld=cid:4ab785d2,vid:GgvajuvSpF4
Uudempi selostus täsmentää tuohon sisältyneitä epäselvyyksiä:

https://youtu.be/vnGWDYfweTI
Otsikko: Vs: Epäselvyys suhteellisuusteorian aika ja etäisyys käsityksissä
Kirjoitti: velihopea - 30.07.2023, 16:55:51
Lainaus käyttäjältä: Mare Nectaris - 28.07.2023, 20:40:43
Pseudogravitaatio ei tarkoita turvautumista taikurin hattuun. Se on tapa erotella ajatuskokeessa ("Gedankenexperiment") kiihtyvyyden g-voima (suppean suhteellisuusteorian sovellusala) aika-avaruutta kaareuttavasta massan painovoimakentästä (yleisen suhteellisuusteorian sovellusala).

Auttaisiko tämä:

https://math.ucr.edu/home/baez/physics/Relativity/SR/sr-gr.html

Kiitos kommentista. Aina oppii. Aiemmin en ole törmännytkään termiin pseudogravitaatio niissä kirjoissa ja videoissa mitä olen katsellut (ChatGPT tiesi nyt sille myös nimen "artificial gravitation") Tuon linkin takana oli suunnilleen se, mitä edellä tiivistitkin.

Eli AE:n olisi kaksosparadoksin > sen symmetriatapauksen > selityksen vaihe 3:ssa (ks. aiempi postaukseni) pitänyt ennakoida tuleva konseptitarkennus ja käyttää termiä "pseudogravitaatio", ei "[todellinen] gravitaatio".

Mutta silti kaksosparadoksin symmetriatapauksen selityksessä on minusta taikurin sauvaa käytetty rankasti. Ensinnäkin, selityksen mukainen koe ei ole mahdollinen. Ei ole sellaista voimaa joka liikuttaisi kaikkia kosmoksen kappaleita pienellä edestakaisella liikkeellä niiden nykyisten liikeiden päälle. Ja mistä muusta kuin taikurin hatusta tulee esim. "nousee (pseudo)gravitaatiovoima -s-akselin suuntaan".

Kaksosparadoksin perustapaus ("käynpä raketilla Siriuksessa") voidaan kyllä testata (ilman taikasauvaa kun nopeuksia ja matkaa hieman pienennetään).

Mutta ok, ajatuskokeissa voi sattua kaikenlaista, ja voin sulattaa taikasauvan käytön. Mutta puhutun kohdan 3 selitystä en ymmärrä edes taikasauvan kanssa. Siinä kohtaa taikasauva synnytti hetkeksi ps-grav voiman (tavallisen g-voiman?) +x-akselin suuntaan kunnes U1 saavutti taas nopeuden v. Ja tässä U1:n käännöksessä sen käynti nopeutui juuri sopivan verran, että lopputilanteessa kellot näyttävät juuri sopivasti. Millä ihmeen mekanismilla U1:n käyntikiri tapahtui?

Esim. onko ps-grav:ssa grav-potentiaalia? Kyselin asiaa ChatGPT:ltä, joka sanoi että ei ole. Mutta puhutussa kohdassa 3 AE sanoo, juuri grav-potentiaalien ero antaa U1:lle hetkeksi sopivan nopeamman käynnin. Osaako joku tämän selittää?

Kyselin ChatGPT:ltä muitakin asioita, mutta taisin menettää luottamukseni sen asiantuntemukseen. Nimittäin pienen väittelyn ja vastaesimerkkien syöttämisen jälkeen ChatGPT sanoi, että jos auton kello  (verrattuna paikallaan olevann kelloon) jätättää 1 km matkalla (sisältäen lyhyen kiihdytyksen ja jarrutuksen) 2 s, 2 km matkalla se jätättää saman 2 s! Jos näin on, minä olen ymmärtänyt paljon asioita väärin. Mutta, please, sanokaa onko jätättämä 2 km:ssa 2 s vai 4 s.
Otsikko: Vs: Epäselvyys suhteellisuusteorian aika ja etäisyys käsityksissä
Kirjoitti: mistral - 30.07.2023, 21:08:07
velihopea:
Kiitos kommentista. Aina oppii. Aiemmin en ole törmännytkään termiin pseudogravitaatio niissä kirjoissa ja videoissa mitä olen katsellut (ChatGPT tiesi nyt sille myös nimen "artificial gravitation")

Eipä pseudogravitaatiosta paljon kirjoitella koska sillä ei ole vastinetta todellisuudessa, se on vain luonnon tuottama illuusio. Illuusio siksi ettei ps-grav-kentällä ole kykyä dilatoida aikaa. Vain aito gravitaatio pystyy "kuluttamaan" aallon energiaa (kuluttamaan nousussa ja lisäämään laskussa), siis että aalto punasiirtyy. Ja punasiirtymä on juuri se mikä korreloi dilaation kanssa. Mutta koska tämä ei toimi, kaksosparadoksia yritetään selittää nopeuden tuottamalla dilaatiolla ja pituuskontraktiolla. Mutta tämäkin on vaikeaa koska EST on niin vaikea ymmärtää.
Jospa kaksosparadoksista voisi tehdä typistetyn kokeen, mitään sellaisia nopeuksia ei tietenkään saavuteta mutta atomikellojen avulla typistettykin koe voisi kertoa totuuden.
Otsikko: Vs: Epäselvyys suhteellisuusteorian aika ja etäisyys käsityksissä
Kirjoitti: velihopea - 31.07.2023, 00:13:55
Lainaamistral:  Grav-kentän ja ps-grav-kentän ero on siinä että grav'issa valo punasiirtyy mutta ps-grav'issa ei löydy gravitaatiolähdettä joka tuottaisi punasiirtymän. Joten siksi se ei vastaa todellisuutta.

Ei kolahda minulle, oli ilmassa pseudo tai oikeaa gravitaatiota. Tässä tyrkytän käsitystä, että valon puna/sini/eimitään -siirtymä riippuu valolähteen liikesuunnasta ja havaitsijan paikasta. Eli jos valolähde tulee suoraan koti => sinisiirtymä, valolähde etääntyy suoraan vastakkaisiin suuntaan => punasiirtymä, valolähde paikallaan tai liikkuu sivusuuntaan => eimitään siirtymä, valolähde liikkuu johonkin välisuuntaan => joku heikompi puna tai sinisiirtymä.

Tuota puna/sini siirtymän ja kellon käynnin suoraa korrelaatiota en oikein osta. Koska siirtymän määrä käsittääkseni riippuu mihin suuntaan emittoija liikkuu havaitsijaan nähden. Onko asialle muita mielipiteitä.

Kun minulle kävisi tulkinta että itse kello on tietyssä gravitaatiopositioissa ja sen atomikellon  cesium-133 emittoiman säteilyn aallonpituus korreloi gravitaatiopositioon: pitempi aallonpituus => kello käy hitaammin, lyhyempi => nopeammin. Kello ei välittäisi millaisia valonpilkahduksia se ympäriltään näkee.

Lainaa"    velihopea:
    [vh ps-grav-kenttä on minusta hieman kuin sadun taikuri, jonka kertoja tuo esiin sopivassa kohtaa tekemään sopivan tempun, jotta tarina etenisi halutulla tavalla. Mutta se on itse AE, joka tuo selitykseen tuon ps-grav-kentän, jollaista ei kait ole olemassa. Vai onko niin, että tämä symmetriatilanne on teoriassa olemassa (tuon ps-grav-kentän avustuksella), mutta ei käytännössä? vh]
    Grav-kentän ja ps-grav-kentän ero on siinä että grav'issa valo punasiirtyy mutta ps-grav'issa ei löydy gravitaatiolähdettä joka tuottaisi punasiirtymän. Joten siksi se ei vastaa todellisuutta.
    ----------------
    velihopea:
    [vh Minun tarvi itsellenikin selvittää käsitteitä: Kun AE sanoo tilanteesta käännöspisteessä B' "U2 is indeed at higher gravitational potential than U1", se merkitsee, että U1 on lähempänä massakeskipistettä (ikäänkuin maan pinnalla) ja U2 on kauempana siitä (ikäänkuin leijuu korkealla).
    Kyllä, mitä korkeammalla ollaan sitä suurempi gravitaatiopotentiaali.
    1-vaihe: U2 kiihdyttää +suuntaan, siksi U1 on heikossa gravitaatiokuopassa
    2-vaihe: ei gravitaatiota
    3-vaihe: U2 jarruttaa+kiihdyttää takaisin, siksi U2 on nyt voimakkaassa gravitaatiokuopassa
    4-vaihe: ei gravitaatiota
    5-vaihe: U2 jarruttaa ja siksi U1 on taas heikossa gravitaatiokuopassa

    Näin U2:n voimakas gravitaatiokuoppa tuottaa selvästi suuremman dilaation kuin U1:n heikot kuopat.

U2 ja U1 pitäisi edellä kai vaihtaa paikkaa. Sillä kyse on symmetriatapauksesta, jossa U1 (ikäänkuin maa) liikkuu ja U2 (ikäänkuin raketti) on paikallaan. Minähän en urputa perustapauksen selitystä vastaan.

Koska minä olen hidas ymmärtämään (ja kun edellä U2 ja U1 on vaihtaneet paikkaa), en osaa johtaa selityksestäsi missä kohdin matkaa U1 voisi näyttää mitäkin. Eli jos paikallaan oleva U2 saa total trip timeksi vaikka 100 s, millaisia lukemia U1:ssä voisi olla (sekuntilukemat 7 kohdasta), että sen total trip time lopuksi olisi vaikka 102 s. Jotta saadaan haluttu symmetriatulos, että U2 on jäljessä U1:tä (tosin saatu siten, että U1 on kirinyt U2:n ohi?).

Edellisessä postauksessanihan heitin ilmaan ChatGPT:n väitteen, että ps-gravitaatio ei tuota gravitaatiopotentiaalia. Saattaa olla hankala aihe ja keinoäly on päätellyt vastauksensa väärin?

Postauksen loppuosassa oli myös pari epäselvää kohtaa, mutta en nyt osaa formuloida niistä.

Vinkkaamasi linkki https://www.google.com/search?client=firefox-b-d&q=don+lincoln+twinparadox#fpstate=ive&vld=cid:4ab785d2,vid:GgvajuvSpF4 oli Fermilab-hepun video ja kuvasi tosiaan kaksosparadoksin, jossa sekä maamies että rakettimies ymmärsivät ja hyväksyivät kellojensa erilaisen käynnin. Mutta kyse oli perustapauksesta, mikä ei ole minullekaan nyt ongelma. Symmetriatapaus mainittiin, mutta ei selitetty.

Eusan linkki https://youtu.be/vnGWDYfweTI oli saman esittäjän tekninen selvennys tai akateeminen debatti puheenvuoro aiheesta tarvitaanko kaksosparadoksin selittämisessä kiihdytystä (pseudogravitaatiota) vai ei. Kun rakettiajo oli tehty valmiiksi v vauhtia kulkevilla raketeilla, eri puolilta tähteä, ja lentävällä ajanotolla, selvittiin ilman kiihdytyksiä ja yhdellä liiketilalla.
Otsikko: Vs: Epäselvyys suhteellisuusteorian aika ja etäisyys käsityksissä
Kirjoitti: mistral - 31.07.2023, 12:45:50
Lainaus käyttäjältä: velihopea - 31.07.2023, 00:13:55
Ei kolahda minulle, oli ilmassa pseudo tai oikeaa gravitaatiota. Tässä tyrkytän käsitystä, että valon puna/sini/eimitään -siirtymä riippuu valolähteen liikesuunnasta ja havaitsijan paikasta. Eli jos valolähde tulee suoraan koti => sinisiirtymä, valolähde etääntyy suoraan vastakkaisiin suuntaan => punasiirtymä, valolähde paikallaan tai liikkuu sivusuuntaan => eimitään siirtymä, valolähde liikkuu johonkin välisuuntaan => joku heikompi puna tai sinisiirtymä.

Tuota puna/sini siirtymän ja kellon käynnin suoraa korrelaatiota en oikein osta. Koska siirtymän määrä käsittääkseni riippuu mihin suuntaan emittoija liikkuu havaitsijaan nähden. Onko asialle muita mielipiteitä.

Jos ymmärtäisit gravitaatiopunasiirtymän mekanismin, moni muukin kohta selviäisi. Selitän Ylen kanavaesimerkillä sen.

Otetaan joku Ylen kanava vaikka YleSuomi ja tallennetaan sen ohjelmat vuoden ajalta purkkiin, se siis kestää vuoden kuunnella. Purkki lähetetään uusintana mutta nopeudella 0,8x. Silloin sekunti muuttuu kuunneltuna 1,25 sekunniksi ja vuosi 1,25 vuodeksi. Tämä on vain tekninen juttu mutta jos sama purkki lähetään hidastamattomana eli nopeudella 1,0x gravitaatiokaivosta joka punasiirtää(pidentää) aallonpituutta 1,25x niin kyse ei ole teknisestä jutusta vaan väistämättömästä jutusta, jossa 1 vuoden purkki kuullaan 1,25x vuotta kestävänä.
Koe voidaan tehdä myös "väärinpäin", vapaasta avaruudesta lähetetään purkki 1,0x nopeudella gravitaatiokaivoon jossa se kestää 0,8 vuotta mutta ääni tietysti on kimeämpi. (tai jos se lähetetään 1,25x niin kaivossa sen aika on 1,0x)
Tässä on takana yksinkertainen totuus, kun aalto nousee, punasiirtyy, se menettää energiaa, kun laskeutuu, sinisiirtyy, se saa lisää energiaa.

Otsikko: Vs: Epäselvyys suhteellisuusteorian aika ja etäisyys käsityksissä
Kirjoitti: velihopea - 01.08.2023, 23:22:29
Lainaa
Lainaus käyttäjältä: velihopea - eilen kello 00:13:55

    Tuota puna/sini siirtymän ja kellon käynnin suoraa korrelaatiota en oikein osta. Koska siirtymän määrä käsittääkseni riippuu mihin suuntaan emittoija liikkuu havaitsijaan nähden. Onko asialle muita mielipiteitä.

Lainaus mistral - eilen kello 12:45:50
    Jos ymmärtäisit gravitaatiopunasiirtymän mekanismin, moni muukin kohta selviäisi....

Kiitos mistral, taas opin. Vastauksessani olin vahvassa luulossa, että valon puna/sini-siirtymä vaatisi lähettäjän ja havaitsijan välille liikettä. Olin väärässä, sorry. Tosiaan, on myös tuo gravitaatiopunasiirtymä. Piti oikein selata kirjojani, ja löytyihän se Kari Enqvist: Johdatus suhteellisuusteoriaan, Ursa, s. 83. Olin jopa alleviivaillut siitä kohtia...

Jotenkin kai ajattelin, että se korkealla oleva kappale katsoo aikaa omasta kellostaan (joka korkealla olevan gravitaatiopositionsa ansiosta käy liukkaammin) eikä päättele sitä gravitaatiokaivosta tulevan valon punasiirtymästä.

Ja vielä oups, sorry mistral, nyt huomasin, että olin sotkenut: Edellisessä postauksessani epäilin että sinä olisit sotkenut U2:n ja U1:n paikat. Koska aloitit vaiheiden selitykset "1-vaihe: U2 kiihdyttää +suuntaan...", U1 ja U2 ovatkin ihan oikein päin. Mutta selitätkin ns. perustapausta, jossa U2 (raketti) liikkuu ja U1 (maa) on paikoillaan. Sen kulusta minulla ei ole huomauttamista. Mutta haluaisin pohjia myöten ymmärtää symmetriatapauksen jossa U1 liikkuu ja U2 on paikoillaan.

Taidanpa pohdiskella asioita hieman uusin silmin ja kerätä höyryä seuraavaan postaukseen.

Muuten olen hieman pettynyt, että postausteni suoria kysymyksiä, väitteitä ja skenaarioita ei ole juuri kommentoitu (paitsi sinä joihinkin, kiitos siitä), vain ohjauksia "seuraavalle luukullle". Tosin sellaisetkin ohjaukset otan kyllä vastaan.
Otsikko: Vs: Epäselvyys suhteellisuusteorian aika ja etäisyys käsityksissä
Kirjoitti: mistral - 02.08.2023, 00:32:24
Lainaus käyttäjältä: velihopea - 01.08.2023, 23:22:29
Ja vielä oups, sorry mistral, nyt huomasin, että olin sotkenut: Edellisessä postauksessani epäilin että sinä olisit sotkenut U2:n ja U1:n paikat. Koska aloitit vaiheiden selitykset "1-vaihe: U2 kiihdyttää +suuntaan...", U1 ja U2 ovatkin ihan oikein päin. Mutta selitätkin ns. perustapausta, jossa U2 (raketti) liikkuu ja U1 (maa) on paikoillaan. Sen kulusta minulla ei ole huomauttamista. Mutta haluaisin pohjia myöten ymmärtää symmetriatapauksen jossa U1 liikkuu ja U2 on paikoillaan.

U1 vastaa vaikka maapalloa vaikka sekin kyllä liikkuu aurinkoa kiertävällä radalla, mutta näin on luonnikasta ajatella. U2 taas vastaa rakettia. Näillä on suuri ero, U1 ei kiihdytä mutta U2 kiihdyttää. Siksi U1 vain putoaa ps-grav-kentässä koko universumin kanssa, kyllä, koko universumi putoaa silloin kun U2 kiihdyttää! Mutta U2 ei putoa vaan kiihdyttää pysyäkseen paikallaan juuri samassa ps-grav-kentässä. Ja silloin kun U2 liikkuu tasanopeudella, niin U2:n näkökulmasta se on paikallaan mutta koko universumi liikkuu tasanopeudella. Ja ps-grav-kentän suunta määräytyy raketin suihkun suunnasta, suihku siis näyttää alaspäin kentässä ja kärki ylöspäin. Suihkun suunnassa on hitaampi aika.
Mutta kuten sanottu, en usko että ps-grav-kenttää on edes olemassa. Ja varmaan monet muutkaan ei usko siihen koska kaksosparadoksi selitetään ES:n avulla.
Otsikko: Vs: Epäselvyys suhteellisuusteorian aika ja etäisyys käsityksissä
Kirjoitti: mistral - 02.08.2023, 11:12:26
Korjaus viestiin 44
Otan suoran lainauksen Princeton-paperista:
-----------
"On huolellisesti pidettävä mielessä, että molemmat numeroinnit vuoronperään (siis 1.1.  2.2.  3.3.  4.4. ja 5.5.) kuvaavat täsmälleen samaa prosessia, mutta ensin on numeroituna vaiheet kuvauksena viitaten koordinaattijärjestelmään K, ja jälkimmäisenä koordinaattijärjestelmään K'. Molempien kuvausten mukaan kello U2 on tietyn verran jäljessä kellosta U1 tarkasteltavan järjestelmän lopussa."
------------
Esim 1.1. menee näin:
K levossa:
Kello U2 kiihtyy ulkoisella voimalla positiivista akselia pitkin, kunnes se saavuttaa nopeuden v. U1 pysyy levossa.
K' levossa:
Negatiivisen x-akselin suuntaan syntyy painovoimakenttä, jossa kello U1 putoaa kunnes saavuttaa nopeuden v. Ulkoinen voima positiivista x-akselia pitkin vaikuttaa kelloon U2 ja estää sen liikkumisen. Painovoimakenttä häviää jälleen heti kun kello U1 saavuttaa nopeuden v.

Siis kaikissa 5:ssä kohdassa tulee ensin K levossa ja sitten K'.
Otsikko: Vs: Epäselvyys suhteellisuusteorian aika ja etäisyys käsityksissä
Kirjoitti: velihopea - 03.08.2023, 21:50:18
Kommentoin mistralin - 02-08-2023, 00:32:24 postausta. Kopioin sen olennaisen osan tähän ja lisään kommenttini [vh ... vh] suluin.

[vh nyt siis koko ajan toivottavasti puhutaan symmetriatapauksesta, jossa U1 ja muukin avaruus ensin liikkuu -x-akselin suuntaan ja palaa sitten takaisin +x-akselin suuntaaan entiselle paikallaan U2:n viereen vh]

...    U1 vastaa vaikka maapalloa vaikka sekin kyllä liikkuu aurinkoa kiertävällä radalla, mutta näin on luonnikasta ajatella. U2 taas vastaa rakettia.
[vh Yes vh]

... Näillä on suuri ero,
[vh Millä näillä? Jos ero-vertailun osapuolet ajatellaan olevan perustapaus ja symmetriatapaus, keskustelu harhautuu ja sotkeutuu. Haluaisin että puhutaan vain symmetriatapauksesta ja miten sen mekanismi toimii, jotta lopputuloksena saadaan U2:n samamääräinen jätättäminen U1:sta. AE lausuu artikkelissaan "But it can be easily seen that systems K and K' are by no means equivalent what it comes to the process under consideration." vh]

... U1 ei kiihdytä mutta U2 kiihdyttää. Siksi U1 vain putoaa ps-grav-kentässä koko universumin kanssa, kyllä, koko universumi putoaa silloin kun U2 kiihdyttää! Mutta U2 ei putoa vaan kiihdyttää pysyäkseen paikallaan juuri samassa ps-grav-kentässä. Ja silloin kun U2 liikkuu tasanopeudella, niin U2:n näkökulmasta se on paikallaan mutta koko universumi liikkuu tasanopeudella.
[vh Äh, ei. Tai olen ymmärtänyt AE:n sanat väärin. Jos edellä tarkoitetaan sitä symmetriatapauksen aloittavaa vaihe 1:tä, niin siinä taikasauvalla synnytetyllä (ps-)gravitaatiolla "U1 falls accelerated until it attains velocity v". Siis U1 tavallaan putoaa ja sen myötä saa lisää nopeutta. Ok U1 ei itse kiihdytä (esim rakettimoottorilla), mutta se kiihtyy muun voiman eli (ps-)gravitaation ansiosta.

U2:sta AE kaikissa vaiheissa sanoo, että se "remains at rest" ja hetkellisten (ps-)gravitaatiokenttien ajaksi sama taikasauva loihtii ulkoisen voiman, joka estää U2:ta liikkumasta. U1:n liikkuessa v-nopeudella, AE:n selostuksen mukaan (ps-)gravitaatiovoimaa ei ole, joten silloin U2 ei tarvi mitään kiinnipitäjää tai vastaliikettä. vh]

... Ja ps-grav-kentän suunta määräytyy raketin suihkun suunnasta, suihku siis näyttää alaspäin kentässä ja kärki ylöspäin. Suihkun suunnassa on hitaampi aika.
[vh Ajattelen, että alkukiihdytyksen ja loppujarrutuksen kohdalla U1 ja U2 ovat ihan vierekkäin ja silloin niiden välillä ei juurikaan ole (ps-)gravitaatiopotentiaalieroa, mikä tekisi olennaista eroa niiden käyntiin. Mutta vaiheessa 3, missä U1 on edennyt pitkälle -x-akselin suuntaan, ja jossa taikasauva synnyttää hetkeksi ps-gravitaatiokentän +x-akselin suuntaan, AE selittää "U2 indeed is at a location of higher (ps-)gravitational potential than U1". (Korkea gravitaatiopotentiaali on ikäänkuin lähempänä massakeskipistettä ja siellä kello läy hitaammin.)

Valitettavasti AE ei kerro tapahtuiko vaiheessa 3 a) U1:lle hetkellinen sopiva nopea käynti, b) U2:lle hetkellinen sopiva hidas käynti, c) kaiken aikaa kun U1 oli matkallaan U2 koki gravitaatio-hidastumista, mikä yli-kompensoi U1:n nopeus-hidastumisen, d) kuten (c) mutta se olikin U1 joka koki nopeus-hidastumisensa päälle yli-kompensoivaa gravitaatio-nopeutusta. Ja mistä järkeilystä nuo potentiaalit muuttuivat juuri tuolla lailla? vh]

...    Mutta kuten sanottu, en usko että ps-grav-kenttää on edes olemassa. Ja varmaan monet muutkaan ei usko siihen koska kaksosparadoksi selitetään ES:n avulla.
[vh Mikä on "ES"? erityinen suteellisuusteoriako? vh]
[vh Taidanpa luovuttaa tästä symmetriatapauksen kaivamisesta. AE:n selityksestä en ymmärrä sitä. Ja tämän ryhmän viisaiden kommenteistakaan en ole saanut helpotusta. Webin twin paradox jutut selittävät vain perustapauksen.

Jään käsitykseen, että symmetriatapausta ei voi ainakaan tuollaisena toteuttaa (ei ole mm. taikasauvaa), ja en edes ajatuskokeena ymmärrä miten se toimii. Joten en pidä pätevänä esim. kaksosparadoksin selityksessä heittää, kuviteltuun ja ei-selitettyyn ja prosessina eri lailla toimivaan symmetriatapaukseen viitaten, "ei sen väliä kumman katsotaan pysyvän paikallaan, se on molemmissa tapauksissa U2, joka jätättää".

  Mutta jos joku voi osoittaa ajatteluni sokeita pisteitä, olen tietysti kiitollinen. vh]

--------
Mistralin myöhempään mistral - 02-08-2023, 11:12:26/"Korjaus viestiin 44" postaukseen en osaa kommentoida mitä tässä edellä en olisi jo sanonut.
Otsikko: Vs: Epäselvyys suhteellisuusteorian aika ja etäisyys käsityksissä
Kirjoitti: mistral - 03.08.2023, 23:09:37
Mielestäni Einstein oli historian merkittävin teoreettinen fyysikko ja myös tämä "kaksosparadoksi" vaikka hän ei sitä sillä nimellä kutsukaan, on teoreettisesti oikein tai sanotaan mieluummin loogisesti oikein. Ongelma vaan on se taikasauva, että kuva näyttää virheettömältä mutta kun se ei ole todellinen todellisen gravitaatiolähteen kanssa, niin siihen se kaatuu. Eikä fyysikon virhe ole maailmaa kaatava, jokaiselle sattuu virheitä. Esimerkkinä muistelen että Planck'ille kvantin eli "kappaleen" ottaminen fysiikkaan oli vain laskutekninen apuväline mutta jälkeenpäin se osoittautui todelliseksi fyysiseksi olioksi.

Jos hermostut kaksosparadoksiin, se on normaalia koska niin tekee kaikki muutkin, siis kun siitä ei ole selvää voittajaa näkyvissä, on vain erilaisten selitysten kirjo. Jos kaksosparadoksi testattaisiin atomikelloilla, syntyisi vedenpitävä ymmärrys mikä on oikein. Mutta testi maksaisi paljon, sellaisen raketin valmistus ei ole helppoa. Ehkä tulevaisuudessa on mahdollista tehdä ydinvoimalla toimiva raketti jolla testi onnistuisi.
Otsikko: Vs: Epäselvyys suhteellisuusteorian aika ja etäisyys käsityksissä
Kirjoitti: Eusa - 04.08.2023, 21:39:42
Vision aika-avaruuden kohteiden välisistä etäisyyksistä ja niiden kehittymisestä voi kuvata vaikka niin, että kertyvät ainekset ryystävät aika-avaruutta pitääkseen yllä rakenteitaan alkeisimpien hiukkasten rakentamina sidottuina tiloina. Noin ne eivät ole gravitaatiolähteitä vaan aika-avaruuden nieluja ja etäisyyssuhteiden määrittäjiä. Gravitaation lähde voisi olla avaruuden laajenemisessa, josta riittää syöksyä aineksiin ilman, että kaikki romahtaa yhteen.
Otsikko: Vs: Epäselvyys suhteellisuusteorian aika ja etäisyys käsityksissä
Kirjoitti: velihopea - 07.08.2023, 11:22:18
Lainaus käyttäjältä: mistral - 03.08.2023, 23:09:37
Mielestäni Einstein oli historian merkittävin teoreettinen fyysikko ja myös tämä "kaksosparadoksi" vaikka hän ei sitä sillä nimellä kutsukaan, on teoreettisesti oikein tai sanotaan mieluummin loogisesti oikein. Ongelma vaan on se taikasauva, että kuva näyttää virheettömältä mutta kun se ei ole todellinen todellisen gravitaatiolähteen kanssa, niin siihen se kaatuu. Eikä fyysikon virhe ole maailmaa kaatava, jokaiselle sattuu virheitä.

Minäkään en halua vähätellä Einsteinin merkitystä. Se kommentti kuitenkin, ettei vain hänen auktoriteettiasemansa ole noussut niin korkealle, ja tiedeyhteisö on sementoinut sen vallitsevaksi käsitykseksi, että AE:n esittämiin asioihin ei kehdata (eikä voi?) kohdistaa kriitiikkiä.

Edellä vatvomastani symmetriatapauksesta vielä: Siinä AE kirjoittaa "calculation shows that..." (että U2-kello on tapahtumaketjun jälkeen jätättänyt samalla tavalla kuin perustilanteessa). Oletan että tuo laskelma on maailman tiedepiireissä joskus todettu oikein tehdyksi. Tuskin AE:n 1918 kirjoitusten koostetoimittaja olisi vääräksi todettuja laskelmia julkaissut ilman huomautusta asiasta.

Vaikka tuon ajatuskokeen laskelma olisikin oikeín, kokeen tapahtumaketju ei voi tapahtua. Ei nimittäin ole taikasauvaa, jolla kokeen eri vaiheet saadaan tapahtumaan (koko kosmoksen liikuttaminen ees-taas, U2-kellon paikallaanpito). Mistral epäilee koetta myös sen gravitaatiokäyttäytymisen kannalta.

Tästä edelleen teen johtopäätöksen, että tätä AE:n ajatuskoe-selitystä ei voi pitää todistuksena "ei sen väliä kumpi liikkuu, se on silti U2-kello, joka jätättää" -asialle.

Onko sille parempaa todistusta? Kai AE olisi sellaisen esittänyt, jos olisi tullut mieleen.
Otsikko: Vs: Epäselvyys suhteellisuusteorian aika ja etäisyys käsityksissä
Kirjoitti: velihopea - 07.08.2023, 11:42:02
Lainaus käyttäjältä: Eusa - 04.08.2023, 21:39:42
Vision aika-avaruuden kohteiden välisistä etäisyyksistä ja niiden kehittymisestä voi kuvata vaikka niin, että kertyvät ainekset ryystävät aika-avaruutta pitääkseen yllä rakenteitaan alkeisimpien hiukkasten rakentamina sidottuina tiloina. Noin ne eivät ole gravitaatiolähteitä vaan aika-avaruuden nieluja ja etäisyyssuhteiden määrittäjiä. Gravitaation lähde voisi olla avaruuden laajenemisessa, josta riittää syöksyä aineksiin ilman, että kaikki romahtaa yhteen.

Kiitos Eusa kommentoinnistasi. Pidän lennokkaasta ja mielikuvarikkaasta tavastasi kirjoittaa. Tällä kertaa en kuitenkaan päässyt jujulle mikä oli pointtisi. Ison mittakaavan asioita kuitenkin.

Gravitaatiosta: Sen rooli kosmoksessa, tai "mitä gravitaation on", on minusta saman luokan vaikea kysymys kuin "mitä aika on".
Otsikko: Vs: Epäselvyys suhteellisuusteorian aika ja etäisyys käsityksissä
Kirjoitti: mistral - 07.08.2023, 23:29:33
velihopea:
Tästä edelleen teen johtopäätöksen, että tätä AE:n ajatuskoe-selitystä ei voi pitää todistuksena "ei sen väliä kumpi liikkuu, se on silti U2-kello, joka jätättää" -asialle.

Suhteellisuudessa on ilmeisesti sellainen "sopimus" että oma koordinaatisto on levossa. En tiedä onko sille vedenpitävät perusteet. Toki jos ajatellaan suurennuslasi havainnollistusta, niin oma koordinaatisto on aina fyysisesti normaali siis perustilassa. Esim valokello heijastaa valoa kahden peilin välissä normaalisti. Valokellon voi kuvata myös superpallolla kun se pomppii kahden levyn välissä. Ohi ajavasta autosta pallo pomppii siksakkia eikä edestakaisin. Tähän kätkeytyy salaisuus tai idea miksi ajan pitää hidastua toisessa koordinaatistossa, ei omassa. Relativistisessa nopeudessa ohi ajava "näkee" valon menemässä siksakkia. Mitä pitempää siksakkia valo matkustaa, sitä pitemmän matkan se tekee. Näin valo voisi matkustaa sekunnissa vaikka miljardi kilometriä! Koska kattonopeus on kuitenkin se 300 000km/s, joudutaan aikaa muuttamaan, hidastamaan. Kun aikaa hidastetaan, valokellossa matkustava aalto saa matkustaa vaikka sen mrd kilometriä kun vastaavasti aikaa hidastetaan. Näin omassa koordinaatistossa valokello on levossa ja valo matkustaa peilien välissä sen 300 000km/s, siis peilien koordinaatistossa. Tämä on sekavaa selitystä mutta tavoittelen perusteita sille miksi suhteellisuudessa oma koordinaatisto on aina levossa. En tiedä onko tämä tulkinta levosta myös Einsteinin näkemys. Kuitenkin tulkinta on puolivalmis. Yksi tärkeä asia unohtui. Pituuskontraktio. Mutta sen yhdistäminen tähän sekoittaa kyllä ajatukset siis en vaan tajua miten asiat menee.
Joka tapauksessa U1 ja U2 katselee toisiaan omasta mielestään levossa, siis kumpikin on levossa omasta mielestään. Hinta mikä siitä maksetaan on pseudogavitaatio. Tämä taas on vain illuusio, ikäänkuin lentokonesimulaattori. Mutta sinänsä Einstein on oikeassa että U1:n näkökulmasta ei saada juuri aikaeroa kelloihin mutta U2:n näkökulmasta saadaan. Ja U2:n näkökulma jostain syystä ratkaisee.
Otsikko: Vs: Epäselvyys suhteellisuusteorian aika ja etäisyys käsityksissä
Kirjoitti: Mare Nectaris - 08.08.2023, 10:00:49
Auttaisiko tämä selventämään teorioiden sovellusalaa ja helpottamaan paradoksin hyväksymistä:

https://www.britannica.com/science/twin-paradox
Otsikko: Vs: Epäselvyys suhteellisuusteorian aika ja etäisyys käsityksissä
Kirjoitti: velihopea - 08.08.2023, 11:54:16
Kiitos mistral kommenteistasi. Monikin kohta inspiroi minua vastaamaan, mutta ensin esille ottamasi valokello, josta olin ajatellut kirjoittaa sopivana hetkenä muutenkin

Lainaus käyttäjältä: mistral - 07.08.2023, 23:29:33
Esim valokello heijastaa valoa kahden peilin välissä normaalisti. Valokellon voi kuvata myös superpallolla kun se pomppii kahden levyn välissä. Ohi ajavasta autosta pallo pomppii siksakkia eikä edestakaisin [vh selvennän: tien laidassa olevalle katsojalle näyttää kuin pallo autossa menisi siksakkia. Valokello on samanlainen, mutta pomppija onkin fotoni vh]. Tähän kätkeytyy salaisuus tai idea miksi ajan pitää hidastua toisessa koordinaatistossa, ei omassa. Relativistisessa nopeudessa ohi ajava "näkee" valon menemässä siksakkia.

Joku aika sitten havahduin ymmärtämään (ihan itse), että valokello on huuhaata!!

Miksi? Siksi, että fotoni ei voi periä emittoijansa liiketilaa. Liike-energia liittyy massaan, jota fotonilla ei ole. Jos emittoija lähettäisi massallisen kappaleen (vaikka pallon) suoraan ylöpäin, (pallo)kellon idea toimisi.

Ensin kysyin ChatGPT:ltä, joka sanoi "No, photons do not inherit the motion of the object from which they were emitted. Photons are elementary particles and have their own independent motion. When a photon is emitted from an object, such as an atom or a particle, it moves independently from that object at the speed of light in a vacuum. The motion of the object itself does not affect the velocity or trajectory of the photon. This is one of the fundamental principles of light propagation and is described by the laws of electromagnetism."

Halusin löytää webistä vahvistuksen, ja sitten löysinkin artikkelin (2021) "On two thought experiments revealing two massive theoretical anomalies, proving both the contemporary "ray of light" paradigm to be flawed and the impossibility of a photon to inherit any velocity vector component from its source" https://www.sciencedirect.com/science/article/pii/S0030402620316752

Lainaus artikkelista: "Each photon trajectory is perfectly perpendicular to the x-axis since a photon is not inheriting any of the photon source velocity vector components, thus also not in the x-axis direction."

-----
Siis, jos olen ymmärtänyt, valokello ei toimi aikadilaation ja siitä seuraavan pituuskontraktion todisteena. Tosin AE ei kai perustellut asioita valokellolla 1905...

Pointti kuitenkin: Valokello-esimerkkiin olen törmännyt useassa paikassa, jossa ST:aa selitetään. Jos valokellolla perustelu on huuhaata, niinkuin minusta näyttää, niin millaisia kritiikittömiä ST-lampaita ST-selittäjät oikein ovatkaan. Mitä muuta loperöä siellä kenties on.
Otsikko: Vs: Epäselvyys suhteellisuusteorian aika ja etäisyys käsityksissä
Kirjoitti: mistral - 08.08.2023, 12:16:50
Lainaus käyttäjältä: Mare Nectaris - 08.08.2023, 10:00:49
Auttaisiko tämä selventämään teorioiden sovellusalaa ja helpottamaan paradoksin hyväksymistä:

https://www.britannica.com/science/twin-paradox

Itsellä on jo käsitys paradoksista:
viesti 5:

Itse olen vuosia ihmetellyt näitä asioita ja päätynyt lopulta seuraavaan, hyvin lyhyt kuvaus:

1. molemmat, sekä astronautti että maassa oleva liikkuu toistensa suhteen. Siksi molempien näkökulma pitää ottaa huomioon. Tämä onnistuu kun molempien maailmanviivat tunnetaan. Maailmanviiva kuvaa ajankulua, mitä lyhyempi viiva, sitä lyhyempi aika. Juuri pituuskontraktio lyhentää maailmanviivaa. Esim valon maailmanviiva on tasan nolla koska kontraktio on litistänyt avaruuden nollaan (etenemissuunnassa).
2. Kun maailmanviivat tunnetaan kummaltakin, niiden erotus kertoo matkan tuottaneen ikäeron.
3. Jotta maailmanviivat saadaan määritettyä, teen sen vain näiden kahden osapuolen välillä, unohdan koko muun universumin. Astronautin viiva määritetään suhteessa maan koordinaatistoon ja taas maan viiva määritetään suhteessa astronautin koordinaatistoon. Tilanne näyttää äkkiseltään symmetriseltä mutta ei ole sitä. Epäsymmetrisyys tulee siitä että vain astronautti kiihdyttää ja jarruttaa. Siksi astronautti on jatkuvasti (ilman viivettä) "tietoinen" maan koordinaatiston liiketilasta kun taas maa on etäisyydestä johtuen viiveellä "tietoinen" astronautin koordinaatistosta. Tämä viive juuri vaikuttaa maailmanviivan muodostukseen ja siksi astronautin saavuttua maahan, sen viiva on lyhyempi. Se on siis matkustanut voimakkaammassa kontraktiossa keskimäärin.

En väitä että välttämättä olisin oikeassa, ompahan yksi käsitys muiden joukossa.


Mutta se mitä en ymmärrä, koski edellisessä viestissä suhteellisuuden lepokoordinaatistoja. Siis jostain syystä Einstein on valinnut kaikki koordinaatistot lepokoordinaatistoiksi ja tähän hänellä on ollut syynsä mitkä ei ole itselle auenneet. Ehkä syy on ollut matematiikassa, että se helpottuisi tai onko syy sitten ollut siinä että valon nopeus on aina c vaikka havaitsijan liiketila muuttuisi kuinka paljon tahansa.
Kuitenkin U2:n kiihdytykset pakottaa sen olemaan levossa. Kuinka voi olla yhtä aikaa levossa ja kiihdyttää? Siten että kiihdyttää vasten gravitaatiota. Mutta jos gravitaatiota ei ole? Silloin se pitää tehdä pseudokenttänä. Tässä on ongelma joka juontaa siitä että kaikkien koordinaatistojen pitää olla "omasta mielestään" levossa. Joku voi toki sanoa ettei se ole ongelma vaan pelkkä sopimus. Joka tapauksessa pseudokenttä on ongelma koska se ei voi tuottaa dilaatiota niinkuin aito kenttä.


Otsikko: Vs: Epäselvyys suhteellisuusteorian aika ja etäisyys käsityksissä
Kirjoitti: Kaizu - 08.08.2023, 13:45:27
Fotonin liikemäärä riippuu fotonin taajuudesta / aallonpituudesta. Etääntyvän kappaleen säteilemä fotoni näkyy punasiirtyneenä eli matalampitaajuuksisena. Wikipediassa asia on selostettu kohtuu hyvin.

Kaizu
Otsikko: Vs: Epäselvyys suhteellisuusteorian aika ja etäisyys käsityksissä
Kirjoitti: mistral - 08.08.2023, 15:17:24
Lainaus käyttäjältä: velihopea - 08.08.2023, 11:54:16

Joku aika sitten havahduin ymmärtämään (ihan itse), että valokello on huuhaata!!

Miksi? Siksi, että fotoni ei voi periä emittoijansa liiketilaa. Liike-energia liittyy massaan, jota fotonilla ei ole. Jos emittoija lähettäisi massallisen kappaleen (vaikka pallon) suoraan ylöpäin, (pallo)kellon idea toimisi.

Ensin kysyin ChatGPT:ltä, joka sanoi "No, photons do not inherit the motion of the object from which they were emitted. Photons are elementary particles and have their own independent motion. When a photon is emitted from an object, such as an atom or a particle, it moves independently from that object at the speed of light in a vacuum. The motion of the object itself does not affect the velocity or trajectory of the photon. This is one of the fundamental principles of light propagation and is described by the laws of electromagnetism."

Totta fotoni ei peri nopeuttaan emittoijan nopeudesta. Mutta ei se estä valokelloa todistamasta dilaatiosta. Toki tässä kokeessa valo matkustaa ylivalonnopeudella! Miten se on selitettävissä? Shapiro-efekti selittää sen, efekti mahdollistaa yli/alivalonnopeuden. Se on todistettu tietääkseni jollain satelliitilla joka kiersi auringon toisella puolella. Satelliitin lähettämä aalto matkusti auringon gravitaation vuoksi pitemmän ajan kuin vapaassa avaruudessa, näin sen nopeus oli maahan nähden alle c. Ylivalonnopeus taas todetaan päinvastaisessa tilanteessa, kun mennään itse gravitaatiokaivoon, niin vapaassa avaruudessa menevät aallot etenee ylivalonnopeudella.
Näin valon nopeus rikkoutuu kun havaitsija ei ole läsnä. Mutta jos havaitsija itse ottaa aallon vastaan, siinä sen nopeus on aina c.
Valokello on vähän arvoituksellinen juttu, onko kello eri systeemi kuin aalto? Jos on, silloin Shapiro-efekti selittää sen. Mutta jos samaa systeemiä, silloin dilaatio selittää sen. Tähän tulisi tosiaan se ongelma että valo voidaan pakottaa matkustajaksi rakettiin, sille pannaan "käsiraudat". Täytyy miettiä ja sulatella.
Otsikko: Vs: Epäselvyys suhteellisuusteorian aika ja etäisyys käsityksissä
Kirjoitti: velihopea - 08.08.2023, 21:26:03
Alkuun pyydän anteeksi edellisessä postauksessa käyttämiäni huonosti valittuja sanoja "kritiikittömiä, ST-lampaita, ST-selittäjät". Tahdon uskoa, että me kaikki yritetään parhaamme.

Tällä kertaa kiitokset Mare Nectarikselle, joka sinnikkäästi vinkkaa erilaisia luukkuja, josko niiden takaa löytyisi minullekin ymmärrystä ST-asioista. Mistral jo tätä toiselta kannalta kommentoikin, mutta kun tulin kirjoittaneeksi:

Lainaus käyttäjältä: Mare Nectaris - 08.08.2023, 10:00:49
Auttaisiko tämä selventämään teorioiden sovellusalaa ja helpottamaan paradoksin hyväksymistä:

https://www.britannica.com/science/twin-paradox

No, tuo oli lyhyt kuvaus kaksosparadoksin perustapauksesta, tai vain lähes maininta mitä se tarkoittaa. Mutta nyt luin seuraavan rivin, mitä MN kai toivot, että ymmärtäisin ST:stä. Seuraavan:

"But in relativity, what one observer sees as happening to a second one, the second one sees as happening to the first one."

Hienoa. Sain määritelmän mitä relatiivisuus mm. tarkoittaa ja edellyttää. Kaksosparadoksiin tämä liittyy siten, että kun rakettisisko on palannut maahan ja on nuorekkaampi kuin maasisko (tässä Britannican esimerkissä roolihenkilöt ovat siskoksia), rakettisisko sanoo, että hänpä kuvittelee olleensa paikallaan ja maasisko onkin liikkunut, ja rupsahtaminen pitäisi olla toisinpäin. Tämä on se paradoksi.

Tuo "minä olenkin paikallaan, toinen liikkuu" on kai se symmetriatapaus, jota olemme jo puineen väsyksiin asti. (Ja josta olen jäänyt kasitykseen, että sitä ei voi toteuttaa). Tai sitten tehdään tietoinen järjestelymuutos, että se toinen todella liikkuu (ja hänen ei tarvi raahata koko kosmosta mukanaan) ja minä olen todella paikallaan. Mutta tämäkään ei käy, koska relatiivisuusperiaate vaatii "ristiinkuvittelun".

Tuolla sivulla paradoksi selitetään tavalliseen tapaan ei-paradoksiksi sillä, että matka pysähtyi käännösvaikeessa ja rakettisisko kokikin kaksi liiketilaa. Sivu ei muuten helpottanut ymmärrystäni.

Aihepiirin esimerkkeihin kuuluu myös seuraava: Raketti lähtee maasta 0.8c nopeudella. Maassa on iso led-taulu, joka näyttää maan kelloa. Raketissa on vastaava led-taulu raketin kellolle. Maassa ja raketissa kiikarit (joilla on ääretön signaalinopeus), joilla näkee toisen led-taulun lukeman.

Ymmärtääkseni, kaiketi, tuohon relatiivisuusperiaatteeseen nojaten Suomen ehkä ykkösguru Kari Enqvist kirjassaan Johdatus ST:hen, s. 42, ja myös videolla https://www.youtube.com/watch?v=7ZlIoleJkHI, 17 min-->, esittää että matkan alussa:
- a) kun maassa tulee 1 h täyteen raketin kello näyttää maasta katsoen 36 min [vh Käy järkeeni vh]
- ja b) kun raketissa tulee 1 h täyteen maan kello näyttää raketista katsoen 36 min. [vh KE ei sanonut, mutta tämän täytyy olla se "symmetriatapaus", jota olemme veivanneet, ja jossa raketti katsoo olevansa paikallaan ja maa liikkuu vh]

Sujuvasti KE ohitti (b)-tapauksen selittämisen, niinkuin kaikki muutkin kaksosparadoksin selittäjät. KE toteaa vain, että "molemmat ovat oikeassa, sillä absoluuttista aikaa ei ole". Videolla KE selitti, että puhutut asiat on todennettu mm. kelloja lennättämällä. Onko kukaan kuullut että myös (b) tapaus olisi todennettu. Jos on, niin tässä minulla on vielä paradoksia purtavaksi.

Mutta minun järkeilyn mukaan on vielä (minusta todellisin tilanne, jonka "ostan")
- c) kun raketissa tulee 1 h täyteen maan kello näyttää raketista katsoen 100 min. Tällaisilla kellojen käynneillä saadaan maassa 20 v, raketissa 12 v, koko 16 vv matkan kestoksi.
Otsikko: Vs: Epäselvyys suhteellisuusteorian aika ja etäisyys käsityksissä
Kirjoitti: velihopea - 08.08.2023, 22:33:39
Lainaus käyttäjältä: Kaizu - 08.08.2023, 13:45:27
Fotonin liikemäärä [vh sanoisin "energia" vh] riippuu fotonin taajuudesta / aallonpituudesta. Etääntyvän kappaleen säteilemä fotoni näkyy punasiirtyneenä eli matalampitaajuuksisena [vh samaa mieltä vh]. Wikipediassa asia on selostettu kohtuu hyvin [vh Missä kohtaa, linkki, että voisi katsoa vh].

Oletan, että kommentti liittyy aiempaan valokello-postaukseeni (08-08-2023, 11:54:16), jossa kerroin "löydöksen", että valokello ei toimi. Syynä, että valokellon pohjan suoraan ylös emittoima fotoni ei peri valokellon pohjan mitään liikettä lentorataansa. Löydös on pätevän tuntuisessa tiedeartikkelissa https://www.sciencedirect.com/science/article/pii/S0030402620316752. Artikkeli sanoo suorin sanoin, että valokello ei toimi. Artikkeli on vuodelta 2021, joten onkohan Wikipedia huomioinut vielä sitä. Vai onko Wikipediassa edes artikkelia fotonin lentoradasta suhteessa fotonin emittoijan liikkeeseen.
Otsikko: Vs: Epäselvyys suhteellisuusteorian aika ja etäisyys käsityksissä
Kirjoitti: Eusa - 09.08.2023, 18:05:19
Lainaus käyttäjältä: velihopea - 08.08.2023, 22:33:39
Oletan, että kommentti liittyy aiempaan valokello-postaukseeni (08-08-2023, 11:54:16), jossa kerroin "löydöksen", että valokello ei toimi. Syynä, että valokellon pohjan suoraan ylös emittoima fotoni ei peri valokellon pohjan mitään liikettä lentorataansa. Löydös on pätevän tuntuisessa tiedeartikkelissa https://www.sciencedirect.com/science/article/pii/S0030402620316752. Artikkeli sanoo suorin sanoin, että valokello ei toimi. Artikkeli on vuodelta 2021, joten onkohan Wikipedia huomioinut vielä sitä. Vai onko Wikipediassa edes artikkelia fotonin lentoradasta suhteessa fotonin emittoijan liikkeeseen.
Ei valokellon valon tarvitse "periä" mitään liiketilaa - se säilyttää muun liiketilansa paitsi kokonaisheijastuu vuoroin. Eri koordinaatistoista tarkasteltuina, kun on nopeutta, se liike näkyy siksakkina.
Otsikko: Vs: Epäselvyys suhteellisuusteorian aika ja etäisyys käsityksissä
Kirjoitti: mistral - 10.08.2023, 10:59:52
Yritin  saada tolkkua valokellosta ja pitkän pyörittelyn jälkeen päädyin outoon ratkaisuun. Ensin valitsin Shapiro-viiveen A vaihtoehdon B:n sijaan:
A Shapiro-viive toteutuu gravitaatiossa (valon nopeus pienenee vapaan avaruuden koordinaatistoon verrattuna) ---> siksi viive toteutuu myös ES:n tapauksessa peilien välissä. Näin kellon tikitys harvenee. (jos peilien välinen matka on 1,5m niin tikitystaajuus on 200 000 000Hz, jossain nopeudessa taajuus harvenee esim 2Hz:iin).
B Shapiro-viive ei toteudu ES:n tapauksessa koska vain peilit luetaan dilaation alaiseen systeemiin, siis valoa ei.

https://www.einstein-online.info/en/explandict/shapiro-delay/

Siis A valittiin. Tässä on se kummallisuus että valon nopeus hidastuu mutta raketin nopeus joka menee poikittaiseen suuntaan valoon nähden, ei hidastu ja poikittaisesta liikkeestä tulee itse asiassa valolle edelleen valon nopeus. Nyt valon nopeus ei hidastukaan. Onko valokellon idea ikäänkuin serpentiini, paperinauhan pituus pysyy vakiona mutta sen muoto muuttuu? (paikallaan ollessa serpentiini on rullassa mutta huippunopeudessa se on vedetty suoraksi kuin viiva)
Otsikko: Vs: Epäselvyys suhteellisuusteorian aika ja etäisyys käsityksissä
Kirjoitti: velihopea - 10.08.2023, 15:43:32
Lainaus käyttäjältä: mistral - 07.08.2023, 23:29:33
Suhteellisuudessa on ilmeisesti sellainen "sopimus" että oma koordinaatisto on levossa. En tiedä onko sille vedenpitävät perusteet. Toki jos ajatellaan suurennuslasi havainnollistusta, niin oma koordinaatisto on aina fyysisesti normaali siis perustilassa.

En ole perillä ST:n valintaperusteista lepokoordinaatistoksi. Yleisesti näissä ST:n peruskonseptien aivojumppakekkereissä puhutaan jotenkin: On liikeympäristö K, joka on paikallaan eli levossa ja liikeympäristö  K', joka liikkuu nopeudella v suhteessa K:hon. Siis koska K:n katsotaan olevan paikallaan, K' liikkuu nopeammin kuin K.

Aihe sai minut mallintamaan liikeympäristöjä kosmos-mittakaavassa. Varoitus: lukekaa seuraava hyppysellisen suolaa kanssa, sillä esitys on lähinnä vain omaa ymmärrystäni (korjauksia otetaan vastaan).

Kosmoksessa lilluu kaikenlaisia massakappaleita, kuten planeetta, raketti, auto, tähti, asteroidi, myoni, yksittäinen vety -atomi. Ne kaikki liikkuvat jollakin nopeudella, joka on välillä (0-c), ei (0-c], johonkin suuntaan. Minkään paikallaan pysyvän 0-nopeuksisen kappaleen löytäminen lienee vaikeaa, josko sellaista onkaan.

Jos kunkin tuollaisen kappaleen kylkeen liimataan laadukas atomikello, sen käyntinopeus määräytyy kappaleen sen-hetkisestä kosmos-mittakaavaisesta nopeudesta ja gravitaatiotilanteesta. Ja tällaisen kappaleen kellon käyntinopeus on verrattavissa jonkun toisen kappaleen kellon käyntinopeuteen AE:n esittämien kaavojen mukaan (nopeus-ero ja gravitaatiotilanne-ero kosmos mittakaaavasta).

AE esittää jotkut teoreettiset tilanteet, esim suppean ST:n paikallaan olevan kappaleen (K:ssa) ja liikkuvan kappaleen (K':ssa) vaikutuksesta K':ssa olevan kellon käyntiin, niin kuin ympärillä ei mitään muuta olisikaan. Maan piirissä liiketilat ovat kuitenkin "kehystyneet". Esimerkkinä
- auto ajaa päiväntasaajalla itä-länsi suuntaista maantietä 100 km/h
- maan pyöriminen huomioiden ajossa itäänpäin vauhti on 1670+100 km/h, länteen 1670-100 km/h
- mutta koko maapallo kiertää aurinkoa 107.000 km/h, myös tuo auto
- ja auto saa lisävauhtia, kun auton puoli maapallosta kiertää samaan suuntaan kuin on maan kiertorata, puolen vrk päästä taas auton meno on hitaampaa
- mutta aurinkokuntammekin liikkuu linnunradassa, arviolta 828.000 km/h
- eikä tämäkään riitä, vauhdikas törmäys on odotettavissa heti kohta jonkun toisen galaksin kanssa
- ja vielä isompiakin rakenteita on

Tuo autoesimerkki on mielenkiintoinen. Se on hieman samanlainen kuin kellojen lennätys 1971 https://en.wikipedia.org/wiki/Hafele%E2%80%93Keating_experiment. Kellojen lennätys itäkautta vs. länsikautta antoivat nopeussyystä (gravitaatiosyy suodatettu pois) eri tuloksen, mutta odotusten mukaisen kun maan pyöriminen oli huomioitu.

Pointti, jonka haluan tuoda esiin: Oman (K, maantie) ja liikkuvan (K', auto) liiketilan valinnassa (tai saatujen tulosten tulkinnassa) on otettava huomioon sopivaan mittaan asti isompi konteksti. Eli että pieleen menee ajattelu, että minä olen paikallani ja lähetän auton matkaan 100 km/h ja odotan tulosta, että auton kello on nopeuden vuoksi hidastunut v = 100 km/h arvolla AE-kaavan mukaan. Jos auto lähtee "väärään" suuntaan, sen kellon käynti onkin nopeutunut! Eli tilanne ei ole, että vaan valitsee näkökulman.
Otsikko: Vs: Epäselvyys suhteellisuusteorian aika ja etäisyys käsityksissä
Kirjoitti: velihopea - 10.08.2023, 17:57:20
Shokeeraavaan väitteeseeni "valokello ei toimi" on tullut muutama kommentti, että ~"kyllä se toimii". Onkohan lukijakunnassa sellaisia taitohenkilöitä, jotka voivat jostain tietokannasta katsoa miten viittaamaani artikkelia https://www.sciencedirect.com/science/article/pii/S0030402620316752 on referoitu tiedepiireissä, onko sen väitteet hyväksytty vai todettu vääräksi. Onko artikkeli vertaisarvioitu? Minkä painoarvon lehdessä se on julkaistu? Kiitos etukäteen.

Tai osaako joku sanoa, että olen tulkinnut (artikkeliin viitaten) artikkelin johtopäätöksen "valokello ei toimi" väärin.

Nimittäin minusta tuo artikkelin johtopäätös pitäisi voida osoittaa vääräksi, jos haluaa pitää valokellon voimissaan.
Otsikko: Vs: Epäselvyys suhteellisuusteorian aika ja etäisyys käsityksissä
Kirjoitti: mistral - 10.08.2023, 22:01:12
Lainaus käyttäjältä: velihopea - 10.08.2023, 15:43:32

AE esittää jotkut teoreettiset tilanteet, esim suppean ST:n paikallaan olevan kappaleen (K:ssa) ja liikkuvan kappaleen (K':ssa) vaikutuksesta K':ssa olevan kellon käyntiin, niin kuin ympärillä ei mitään muuta olisikaan. Maan piirissä liiketilat ovat kuitenkin "kehystyneet". Esimerkkinä
- auto ajaa päiväntasaajalla itä-länsi suuntaista maantietä 100 km/h
- maan pyöriminen huomioiden ajossa itäänpäin vauhti on 1670+100 km/h, länteen 1670-100 km/h
- mutta koko maapallo kiertää aurinkoa 107.000 km/h, myös tuo auto
- ja auto saa lisävauhtia, kun auton puoli maapallosta kiertää samaan suuntaan kuin on maan kiertorata, puolen vrk päästä taas auton meno on hitaampaa
- mutta aurinkokuntammekin liikkuu linnunradassa, arviolta 828.000 km/h
- eikä tämäkään riitä, vauhdikas törmäys on odotettavissa heti kohta jonkun toisen galaksin kanssa
- ja vielä isompiakin rakenteita on

Käsittääkseni suhteellisuus on vain kahden eri koordinaatiston välinen asia, muu ympäristö ei muuta kaavoja paitsi tietysti gravitaation kautta. Suhteellisuudessa on se hyvä puoli että se toimii kaikkialla, vaikka menisi gravitaatiokaivoon. Siis ankkuri joka pitää on suhteellisuus, se on "pohjassa" kiinni, kaikki koordinaatistot vaan leijuu äärettömyydessä. Joku sanoi että avaruuden lepokoordinaatisto voisi olla siinä kun kosmisen taustasäteilyn punasiirtymä olisi sama joka suunnassa mutta sillä ei päästä tarkkaan tulokseen.
Otsikko: Vs: Epäselvyys suhteellisuusteorian aika ja etäisyys käsityksissä
Kirjoitti: mistral - 10.08.2023, 22:36:10
Lainaus käyttäjältä: velihopea - 10.08.2023, 17:57:20
Shokeeraavaan väitteeseeni "valokello ei toimi" on tullut muutama kommentti, että ~"kyllä se toimii". Onkohan lukijakunnassa sellaisia taitohenkilöitä, jotka voivat jostain tietokannasta katsoa miten viittaamaani artikkelia https://www.sciencedirect.com/science/article/pii/S0030402620316752 on referoitu tiedepiireissä, onko sen väitteet hyväksytty vai todettu vääräksi. Onko artikkeli vertaisarvioitu? Minkä painoarvon lehdessä se on julkaistu? Kiitos etukäteen.

Tai osaako joku sanoa, että olen tulkinnut (artikkeliin viitaten) artikkelin johtopäätöksen "valokello ei toimi" väärin.

Nimittäin minusta tuo artikkelin johtopäätös pitäisi voida osoittaa vääräksi, jos haluaa pitää valokellon voimissaan.

Mikä on mielestäsi valokellon ongelma?
Artikkelin oli tehnyt Etienne Brauns, belgialainen tutkija. Aiemmin ajattelin että valokellon pidentyvä hypotenuusa kertoo suoraan ajan hidastumisesta mutta nyt en ole ollenkaan varma. Täytyy vissiin opiskella valokelloa enemmän...
Otsikko: Vs: Epäselvyys suhteellisuusteorian aika ja etäisyys käsityksissä
Kirjoitti: velihopea - 10.08.2023, 23:46:06
Lainaus käyttäjältä: mistral - 10.08.2023, 22:36:10
Mikä on mielestäsi valokellon ongelma?
Artikkelin oli tehnyt Etienne Brauns, belgialainen tutkija. Aiemmin ajattelin että valokellon pidentyvä hypotenuusa kertoo suoraan ajan hidastumisesta mutta nyt en ole ollenkaan varma. Täytyy vissiin opiskella valokelloa enemmän...

Ongelmana näen, että esim. kun valokello liikkuu 0.5c nopeudella oikealle, fotoni ei "lennä" (ulkopuolisen tarkkailijan näkemänä) 45 asteen kulmassa etu-oikealle vaan suoraan ylöspäin. Näin ymmärtääkseni silloinkin vaikka valokellon sisällä olisi fotonin lentoa tarkkaileva henkilö. (Hän näkisi fotonin heti kohta törmäävän 45 asteen kulmassa kellon vasempaan seinään.) Eihän fotoni peri "buustia" myöskään, jos se lähetetään liikkeeseen nähden taaksepäin tai eteenpäin.

Jos jotain pitää tutkia, niin mielestäni se on tuo artikkeli eli onko Etienne Braunsin "löydös" oikea.
Otsikko: Vs: Epäselvyys suhteellisuusteorian aika ja etäisyys käsityksissä
Kirjoitti: mistral - 11.08.2023, 08:08:19
Lainaus käyttäjältä: velihopea - 10.08.2023, 23:46:06
Ongelmana näen, että esim. kun valokello liikkuu 0.5c nopeudella oikealle, fotoni ei "lennä" (ulkopuolisen tarkkailijan näkemänä) 45 asteen kulmassa etu-oikealle vaan suoraan ylöspäin. Näin ymmärtääkseni silloinkin vaikka valokellon sisällä olisi fotonin lentoa tarkkaileva henkilö. (Hän näkisi fotonin heti kohta törmäävän 45 asteen kulmassa kellon vasempaan seinään.) Eihän fotoni peri "buustia" myöskään, jos se lähetetään liikkeeseen nähden taaksepäin tai eteenpäin.

Jos jotain pitää tutkia, niin mielestäni se on tuo artikkeli eli onko Etienne Braunsin "löydös" oikea.

Kovassa kiihdytyksessä fotoni osuisi vasempaan seinään mutta ei tasaisessa vauhdissa. Jos alusta kiihdytetään, pitää peilien asentoa säätää mutta tasanopeudessa niiden pitää olla linjassa.
Kun Shapiro-viive vaikuttaa, saa valoaalto edetä hitaammin kuin c. Esim avaruusasemalta katsottuna maan pinnalla valo etenee alle c:n nopeudella eli Shapiro-viive = gravitaation tuottama dilaatio.
Mutta kun käsitellään nopeuden tuottamaa dilaatiota, ainakin itselle on vaikea hahmottaa dilaatiota valokellosta:

A: peilit on dilaation alaisuudessa, myös valo on dilaation alaisuudessa, mutta valon poikittainen liike (pitkittäinen liike = peilien välillä, poikittainen liike = aluksen suuntainen liike) ei ole dilaation alaisuudessa.

B: peilit on dilaation alaisuudessa, valo ei ole dilaation alaisuudessa koska aina liikkuu c nopeudella, se on erikoistapaus

Siis gravitaation tuottama dilaatio/c-nopeuden hidastuma on helppo ymmärtää mutta nopeuden tuottama dilaatio/c-nopeuden hidastuma on vaikeampi.
Otsikko: Vs: Epäselvyys suhteellisuusteorian aika ja etäisyys käsityksissä
Kirjoitti: Mare Nectaris - 11.08.2023, 11:26:49
Olisiko tästä artikkelista hyötyä (ja lopussa olevasta linkkilistasta aiheesta käytävään keskusteluun):

https://www.scirp.org/journal/paperinformation.aspx?paperid=75772
Otsikko: Vs: Epäselvyys suhteellisuusteorian aika ja etäisyys käsityksissä
Kirjoitti: mistral - 11.08.2023, 15:45:26
Lainaus käyttäjältä: Mare Nectaris - 11.08.2023, 11:26:49
Olisiko tästä artikkelista hyötyä (ja lopussa olevasta linkkilistasta aiheesta käytävään keskusteluun):

https://www.scirp.org/journal/paperinformation.aspx?paperid=75772

Kaavat oli hepreaa mutta asiat tuttuja.

Mutta onko sinulla kommentoitavaa edelliseen viestiin:

A: peilit on dilaation alaisuudessa, myös valo on dilaation alaisuudessa, mutta valon poikittainen liike (pitkittäinen liike = peilien välillä, poikittainen liike = aluksen suuntainen liike) ei ole dilaation alaisuudessa.

B: peilit on dilaation alaisuudessa, valo ei ole dilaation alaisuudessa koska aina liikkuu c nopeudella, se on erikoistapaus


Itse pidän A:ta ainakin osittain oikeana, siinä valon nopeus peilien välillä hidastuu ja siksi tikitys harvenee. Hämäräksi kuitenkin jää aluksen suuntaisen komponentin dilaatio, sehän on koko ajan kiihtynyt kun lähestytään valon nopeutta. Eli peilien välinen on hidastunut mutta aluksen suuntainen kiihtynyt!
Otsikko: Vs: Epäselvyys suhteellisuusteorian aika ja etäisyys käsityksissä
Kirjoitti: velihopea - 11.08.2023, 16:00:23
Lainaus käyttäjältä: mistral - 11.08.2023, 08:08:19
Kovassa kiihdytyksessä fotoni osuisi vasempaan seinään mutta ei tasaisessa vauhdissa.
...
Kun Shapiro-viive vaikuttaa, saa valoaalto edetä hitaammin kuin c.

Valokelloasiassa meidän ei minusta kannata debatoida keskenämme. Viittaamani artikkelin väite pitäisi kumota.

Tuosta valon nopeudesta kommenttisi, että se voisi joskus olla c+ ja joskus c-, ovat minusta epäilyttäviä. Mutta sain aasinsillan purkaa mieltäni valosta laajemminkin.

Minusta valon nopeutta on tiedemielessä kyllä asiallista kyseenalaistaa. Nimittäin valon vakionopeus c tyhjiössä on ST:n postulaatti. Postulaatti on todistamaton lähtöoletus. Sittemmin vallitsevä käsitys on tullut niin vakuuttuneeksi ST:n oikeassa olemisesta, ja kun erilaiset ST-mallin mukaiset tapahtumat on kokein todettu (ei tosin kaikkia, esim. pituuskontraktiota ei tietääkseni ole), c on nostettu luonnonvakion asemaan.

Nopeus on m/s. Cesium-133 emittoi jotain värähtelyä, ikäänkuin valoa. 1 s on määritelty ajaksi, jossa tapahtuu 9,192,631,770 värähtelyjaksoa tuota säteilyä (merenpinnan tasalla). 1 m on määritelty matkaksi, jonka valo kulkee tyhjiössä 1/299,792,458 s:ssa.

Kun osoittaja ja nimittäjä perustuu valon nopeuteen, määrittely on minusta jotenkin sisäsiittoinen, mutta kai meidän itsekeskeisen maapallomme tarpeisiin nyt käytännöllinen. Mutta kun Grönlannin jäät sulavat ja meren pinta nousee 2 m, pitääkö normin numeroita muuttaa? Tai kun muutetaan Marsiin, tulee ongelmaksi miten siellä saadaan referenssiaika ja -etäisyys, kun ei ole enää meren pintaakaan.

Kirjoissa joita olen lukenut on aina painotettu, että valon nopeus on vakio tyhjiössä, eli kun valon reitille ei osu massakappaleita. Tuo tyhjiöalue voi kyllä olla muiden massattomien asioiden vaikutuksessa, kuten tausta- ja muu säteily, gravitaatio, gravitaatioallot. Ja uusimpina asioina, voihan siellä olla pimeää ainetta ja pimeää energiaa. En ole kuullut että nuo viimemainitut vaikuttaisivat mitenkään valon nopeuteen.

Kun luin kirjaa Heikki Oja: AE:n perintö, meinasin tippua penkiltä, kun näin kappaleotsikon "Valon nopeus pienenee painovoimakentässä" (s. 96). Kappaleen teksti selvästi sanoo, että AE ajatteli noin 1911, antoipa siihen kaavankin. Mutta nyt ymmärrän (=tahdon ymmärtää), että AE:n olisi pitänyt ilmaista asia seuraavasti: valo ei hidastu gravitaatikentässä, mutta gravitaatiokenttä voi tehdä valon kulkureitille kuopan, ja kun valo joutuun käymään siinä kuopassa, sille tulee pidempi matka, josta johtuen valolta kestää kauemmin päästä A:sta B:hen.

Seuraavaksi kappaleessa puhuttiin Shapiron viivästysmittauksista. Siinä tutkaheijasteen paluuaika Venuksesta vaihteli riippuen oliko Auringon gravitaatiokuoppa välissä vai ei. Ja tuon kuopan syvyys (pitempi matka valolle) oli niiden 1911 AE-kaavojen mukainen, kai. Ja siksi nämä mittaukset listattiin taas uudeksi ST:n riemuvoitoksi.

Mutta, kas, nyt huomasin Mare Nektariksen äskeisen linkin artikkeliin "Why the Speed of Light (c) Keeps Constant?". Sen matematiikasta en juuri ymmärtänyt ja tekstiin en kerinnyt syventyä, josko ymmärtäisinkään, mutta voisitko MN paremmin asioita ymmärtävänä sanoa onko yllä oleva käsitykseni valon kulusta asiallinen.
Otsikko: Vs: Epäselvyys suhteellisuusteorian aika ja etäisyys käsityksissä
Kirjoitti: Eusa - 11.08.2023, 18:39:17
Kuinka voisi olla eri mieltä - minkowskilaisessa hengessä - ajan ja etäisyyden syvästä tasa-arvosta? On yleinen väärinkäsitys, että todellisessa avaruudessa olisi mahdollista liikkua edestakaisin. Aine määrittelee aika-avaruuden (tai aika-avaruus ylläpitää ainetta) ja kun liikut eteenpäin, niin takanasi oleva tila ei ole enää sama tila - siellä määrittävä aine on muuttunut - et voi koskaan palata menneisiin paikkapositioihin; kun kuvittelet kääntyväsi takaisin, käännytkin vain siihen suuntaan, josta seuraava eteenpäin-suunta on löydettävissä. Erillisyyksiin tarvittavat etäisyys ja aika ovat todella samanarvoisia.

Vain siksi on mahdollista kuvitella palaavansa johonkin paikkaan, koska tunnistettavat rakenteet pysyvät tarpeeksi kauan koossa.
Otsikko: Vs: Epäselvyys suhteellisuusteorian aika ja etäisyys käsityksissä
Kirjoitti: mistral - 11.08.2023, 20:28:02
Lainaus käyttäjältä: velihopea - 11.08.2023, 16:00:23


Kun luin kirjaa Heikki Oja: AE:n perintö, meinasin tippua penkiltä, kun näin kappaleotsikon "Valon nopeus pienenee painovoimakentässä" (s. 96). Kappaleen teksti selvästi sanoo, että AE ajatteli noin 1911, antoipa siihen kaavankin. Mutta nyt ymmärrän (=tahdon ymmärtää), että AE:n olisi pitänyt ilmaista asia seuraavasti: valo ei hidastu gravitaatikentässä, mutta gravitaatiokenttä voi tehdä valon kulkureitille kuopan, ja kun valo joutuun käymään siinä kuopassa, sille tulee pidempi matka, josta johtuen valolta kestää kauemmin päästä A:sta B:hen.

Seuraavaksi kappaleessa puhuttiin Shapiron viivästysmittauksista. Siinä tutkaheijasteen paluuaika Venuksesta vaihteli riippuen oliko Auringon gravitaatiokuoppa välissä vai ei. Ja tuon kuopan syvyys (pitempi matka valolle) oli niiden 1911 AE-kaavojen mukainen, kai. Ja siksi nämä mittaukset listattiin taas uudeksi ST:n riemuvoitoksi.

Olen kanssa lukenut Einsteinin perintö kirjan, tärkeä teos ymmärrettävyytensä vuoksi. Oja tarkoitti juuri Shapiro-viivettä, se ilmenee vapaan avaruuden näkökulmasta, ei samassa gravitaatiopotentiaalissa. Eli kun Michelson-Morley mittaus tehtiin, siinä ei ilmene maan gravitaatiokaivon dilaatiota koska ollaan samalla korkeudella. Siis valon nopeus on vakio mutta dilaatio vaikuttaa sen että nopeus näyttää laskevan.
Tämä on oma ajatukseni, Shapiro-viive on saanut merkityksensä juuri siksi koska aina sanotaan että valon nopeus on riippumaton havaitsijan liiketilasta ja kun sitten Shapiro todisti että dilaatio hidastaa valon etenemistä, niin silloin vasta havahduttiin mitä "havaitsijan liiketila" tarkoittaa - sehän tarkoittaa sitä hetkeä kun silmä tai detektori rekisteröi valon.

Valo voi kuten sanottu myös "ylittää" valon nopeuden kun osat vaihdetaan, havaitsija menee gravitaatiokaivoon ja valo kulkee ylempänä. Jos astronautti menee syvälle kaivoon, hän näkee universumin historian etenevän pikavauhtia ja näin valokin menee hurjaa vauhtia. "Yksi sekunti singulariteetissa on ikuisuus vapaassa avaruudessa".
Otsikko: Vs: Epäselvyys suhteellisuusteorian aika ja etäisyys käsityksissä
Kirjoitti: mistral - 13.08.2023, 00:19:48
Lainaus käyttäjältä: velihopea - 11.08.2023, 16:00:23
Nopeus on m/s. Cesium-133 emittoi jotain värähtelyä, ikäänkuin valoa. 1 s on määritelty ajaksi, jossa tapahtuu 9,192,631,770 värähtelyjaksoa tuota säteilyä (merenpinnan tasalla). 1 m on määritelty matkaksi, jonka valo kulkee tyhjiössä 1/299,792,458 s:ssa.

Tämä oli uutta, että meren pinnalla pitäisi olla. Eikö suhteellisuus tarkoita luonnonlakien olevan samat maan ytimessä tai ulkoavaruudessa?
Otsikko: Vs: Epäselvyys suhteellisuusteorian aika ja etäisyys käsityksissä
Kirjoitti: velihopea - 13.08.2023, 11:08:32
Lainaus käyttäjältä: mistral - 13.08.2023, 00:19:48
Lainaus käyttäjältä: velihopea - 11-08-2023, 16:00:23
    Nopeus on m/s. Cesium-133 emittoi jotain värähtelyä, ikäänkuin valoa. 1 s on määritelty ajaksi, jossa tapahtuu 9,192,631,770 värähtelyjaksoa tuota säteilyä (merenpinnan tasalla). 1 m on määritelty matkaksi, jonka valo kulkee tyhjiössä 1/299,792,458 s:ssa.

Tämä oli uutta, että meren pinnalla pitäisi olla. Eikö suhteellisuus tarkoita luonnonlakien olevan samat maan ytimessä tai ulkoavaruudessa?

Muistan kuulleeni tuo merenpinta-asian jostain tiede1 radio-ohjelmasta tai vastaavasta. Google ei osannut minulle vahvistaa muistikuvaani. ChatGPT antoi minun ymmärtää, että pienet korkeuserot mittauspaikassa voi voi jättää huomiotta, koska niillä ei ole käytännön vaikutusta. Mutta totesi kyllä, että säteily käy tiheämmäksi korkeammalla AE:n kaavan mukaisesti.

Meren pinnan taso on minusta järkevä. Sitä ei voi jätää auki, esim korkea vuorenhuippu voisi ehkä tuoda jo merkittävän eron. Viittaamani kellojen lennätys selostus https://en.wikipedia.org/wiki/Hafele%E2%80%93Keating_experiment kertoo 2010 tilanteen, että tarkat atomikellot tunnistavat 36 km/h nopeuseron ja 38 cm korkeuseron. Ja kai kellot ovat 2010 jälkeen tulleet tarkemmiksi.

Jos c on luonnonvakio, kai sen pitää olla tarkka niin pitkälle desimaaleja kuin joku jaksaa vain luetella. Minusta ei käy, että se vaeltaa sen mukaan missä mitataan.
Otsikko: Vs: Epäselvyys suhteellisuusteorian aika ja etäisyys käsityksissä
Kirjoitti: mistral - 13.08.2023, 14:52:22
Lainaus käyttäjältä: velihopea - 13.08.2023, 11:08:32
Muistan kuulleeni tuo merenpinta-asian jostain tiede1 radio-ohjelmasta tai vastaavasta. Google ei osannut minulle vahvistaa muistikuvaani. ChatGPT antoi minun ymmärtää, että pienet korkeuserot mittauspaikassa voi voi jättää huomiotta, koska niillä ei ole käytännön vaikutusta. Mutta totesi kyllä, että säteily käy tiheämmäksi korkeammalla AE:n kaavan mukaisesti.

Meren pinnan taso on minusta järkevä. Sitä ei voi jätää auki, esim korkea vuorenhuippu voisi ehkä tuoda jo merkittävän eron. Viittaamani kellojen lennätys selostus https://en.wikipedia.org/wiki/Hafele%E2%80%93Keating_experiment kertoo 2010 tilanteen, että tarkat atomikellot tunnistavat 36 km/h nopeuseron ja 38 cm korkeuseron. Ja kai kellot ovat 2010 jälkeen tulleet tarkemmiksi.

Jos c on luonnonvakio, kai sen pitää olla tarkka niin pitkälle desimaaleja kuin joku jaksaa vain luetella. Minusta ei käy, että se vaeltaa sen mukaan missä mitataan.

Onkohan Chat GPT keinoäly? Joka tapauksessa se on väärässä koska suhteellisuuden mukaan kaikki koordinaatistot on luonnonlakien suhteen samoja, erot tulee kun katsotaan toiseen koordinaatistoon.

Eri asia on jos cesium-133 reagoi gravitaation kenttävoimakkuuteen tai ilmanpaineeseen, se ei liity suhteellisuuteen.
Cesium on ihme aine, ominaispaino vain 1,93kg/L kun alumiinilla on 2,7kg/L. Ja sulamispiste on 28,44 astetta.
Otsikko: Vs: Epäselvyys suhteellisuusteorian aika ja etäisyys käsityksissä
Kirjoitti: velihopea - 13.08.2023, 19:24:39
Lainaus käyttäjältä: mistral - 13.08.2023, 14:52:22
Onkohan Chat GPT keinoäly? Joka tapauksessa se on väärässä koska suhteellisuuden mukaan kaikki koordinaatistot on luonnonlakien suhteen samoja, erot tulee kun katsotaan toiseen koordinaatistoon.

Eri asia on jos cesium-133 reagoi gravitaation kenttävoimakkuuteen tai ilmanpaineeseen, se ei liity suhteellisuuteen.
Cesium on ihme aine, ominaispaino vain 1,93kg/L kun alumiinilla on 2,7kg/L. Ja sulamispiste on 28,44 astetta.

Joo, ChatGPT on keinöäly. Minusta se on tässä oikeassa.

Mutta kiitos mistral. Kommenttisi auttoi minua selventämään ymmärrystäni siitä mitä en ymmärrä! Nimittäin suhteellisuuden tuollaista tulkintaa.

Ajatellaan GPS systeemiä. N. 20.000 km korkeudessa on GPS-satelliitti jossa on kello (ehkä cesium-133:n säteilyn aallonpituudesta tick-arvonsa saava). Maassa on samanlainen kello. Tosiaan, jos ne ovat samanlaisia, satelliitin kello edistää n. 38 mikrosekunttia (korkeus-nopeutusta n. 45, liike-hidastumista n. 7) vuorokaudessa.

Jos nyt maasta katsotaan kiikarilla satelliitin kellon lukemaa joka paivä klo 24:00 (kiikarit voivat olla normaalit valon nopeuksiset, sillä matka on niin lyhyt, että välitysviive ei kai ole merkitsevä). Maasta näkyy tuo edistäminen. Tämä kai on se mistralin tarkoittama "katsotaan toisesta koordinaatistosta".

Mikä olisi "oma koordinaatisto"? Eikös se ole GPS satelliitti itse, jonka kyydissä voisi olla astronautti, joka katsoo raketin kelloa. Oletetaan vielä, että kun maasta on "katsottu" satelliitin kello, maasta lähetetään radioviesti "onhan kellosi nyt n. sejase". Ymmärtääkseni astronautti ei voi muuta kuin todeta, että "kyllä".

Eli, en voi ymmärtää että olisi mistralin vihjaamaa fantasia-suhteellisuusmaailmaa, jossa oma kello käy omia aikojaan. Satelliitissa sekunnin määrittäminen cesium-133 emittoiman säteilyn 9,192,631,770 värähtelyn ajaksi antaisi erään sekunnin ajan, mutta eri pituisen kuin maassa. Koska todennettu fysikaalinen totuus on, että liikkeessä ja 20.000 km korkeudessa emittoidun säteilyn frekvenssi muuttuu (korkeus ja nopeus syyt).

Pointtini on että maassa tiedetään, että satelliitin kello edistää. Ja kyllä satelliittikin sen tietää. Minusta on jotenkin epärealismia ajatella, että satelliitissa sulkeuduttaisiin omaan maailmaansa, kun kerran tiedetään oman kellon hidastuminen suhteessa ympäristöön, jonka kanssa kuitenkin pelataan yhteen.

Relatiivisuuden ymmärtämiseksi minusta riittää, että ymmärtää liikkeen ja gravitaation vaikutuksen kellon käyntiin. Ja gravitaation vaikutuksen valon kulkureittiin. Mitä muuta tarvitaan?
Otsikko: Vs: Epäselvyys suhteellisuusteorian aika ja etäisyys käsityksissä
Kirjoitti: mistral - 13.08.2023, 22:15:01
Lainaus käyttäjältä: velihopea - 13.08.2023, 19:24:39
Jos nyt maasta katsotaan kiikarilla satelliitin kellon lukemaa joka paivä klo 24:00 (kiikarit voivat olla normaalit valon nopeuksiset, sillä matka on niin lyhyt, että välitysviive ei kai ole merkitsevä). Maasta näkyy tuo edistäminen. Tämä kai on se mistralin tarkoittama "katsotaan toisesta koordinaatistosta".

Joo tuota tarkoitin.
Mutta nyt tajuan merenpinnan idean. Ensin luulin että gravitaation kenttävoimakkuuden ja ilmanpaineen piti olla juuri pinnan tasossa, mutta idea onkin siinä että atomikellon tarkkuuden vuoksi on pakko määrittää sen korkeus tarkkaan eli merenpintaan jotta kellot saadaan kalibroitua oikein. Eli ei riitä sekunnin määritelmäksi vain tikitysluku vaan tarvitaan lisäksi korkeustieto. Ja jos tarkkuutta lisätään, tarvitaan geologinen kartoitus koska raskaampien kivilajien kohdalla meren pinta menee kuopalle. Joo, kyllä meren pinta liittyy sittenkin suhteellisuuteen eikä atomin sisäisiin muutoksiin.
Otsikko: Vs: Epäselvyys suhteellisuusteorian aika ja etäisyys käsityksissä
Kirjoitti: mistral - 14.08.2023, 11:33:39
Löytyi sattumalta arvokas tieto gravitaatiopotentiaalista:
https://fi.wikipedia.org/wiki/Geopotentiaali
Avaruudessa geopotentiaali merkitsee taas inertiaalijärjestelmässä gravitaatiokentän potentiaalia, jonka gradientti on vetovoima. Tämä potentiaali on nolla äärettömyydessä. Fysikaalisen määritelmän mukaan se on negatiivinen maapallon läheisyydessä. Geodesiassa se kuitenkin määritetään päinvastaisella etumerkillä, jolloin se on Maan läheisyydessä positiivinen.


Tähtitieteessä gravitaatiopotentiaali on aina negatiivinen, paitsi äärettömän kaukana tyhjyydessä nolla.
Geodesiassa taas aina positiivinen. Tämä aiheuttaa sekaannusta, kuitenkin kyse on samasta asiasta.
Otsikko: Vs: Epäselvyys suhteellisuusteorian aika ja etäisyys käsityksissä
Kirjoitti: velihopea - 14.08.2023, 23:08:28
Lainaus käyttäjältä: mistral - 13.08.2023, 22:15:01
Joo tuota tarkoitin.

Loistavaa, olemme samaa mieltä. Siis siitä, että liiketila-maasta nähdään liiketila-gps-satelliitin kellon edistäneen n. 38 mikrosekuntia/vrk. Ja kai olemme samaa mieltä myös siitä, että satelliitista nähdään tilanne samoin (että satelliitin kello on käynyt nopeammin).

Entä, ajatuskokeena, jos tilanteesta poistettaisiin gravitaatio, jäisi -7 mikrosekuntia nopeusvaikutusta satelliitin kelloon. Kun satelliitista katsottaisiin maan kelloon, olisiko se edellä vai jäljessä satelliitin kelloa? Anyone?
Otsikko: Vs: Epäselvyys suhteellisuusteorian aika ja etäisyys käsityksissä
Kirjoitti: velihopea - 14.08.2023, 23:17:51
Lainaus käyttäjältä: mistral - 13.08.2023, 22:15:01
Mutta nyt tajuan merenpinnan idean.  [vh Hyvä vh]...

ja seuraava täydennypostaus:
Löytyi sattumalta arvokas tieto gravitaatiopotentiaalista:
https://fi.wikipedia.org/wiki/Geopotentiaali
Avaruudessa geopotentiaali merkitsee taas inertiaalijärjestelmässä gravitaatiokentän potentiaalia, jonka gradientti on vetovoima. Tämä potentiaali on nolla äärettömyydessä. Fysikaalisen määritelmän mukaan se on negatiivinen maapallon läheisyydessä. Geodesiassa se kuitenkin määritetään päinvastaisella etumerkillä, jolloin se on Maan läheisyydessä positiivinen.

Tähtitieteessä gravitaatiopotentiaali on aina negatiivinen, paitsi äärettömän kaukana tyhjyydessä nolla.
Geodesiassa taas aina positiivinen. Tämä aiheuttaa sekaannusta, kuitenkin kyse on samasta asiasta.

Tuo linkin kuvaus oli niin hienosti sanottu, että en ollenkaan osaa hahmottaa mikä kuvattu asia on ja mitä sillä on mihinkään (esim kellon käyntiin) vaikutusta. Linkin teksti jotenkin mallinsi painovoimaa maan pinnalla ja maan pyörähdysliikkeen yhteisvaikutusta.

Otsikko: Vs: Epäselvyys suhteellisuusteorian aika ja etäisyys käsityksissä
Kirjoitti: mistral - 15.08.2023, 00:10:49
Lainaus käyttäjältä: velihopea - 14.08.2023, 23:08:28
Loistavaa, olemme samaa mieltä. Siis siitä, että liiketila-maasta nähdään liiketila-gps-satelliitin kellon edistäneen n. 38 mikrosekuntia/vrk. Ja kai olemme samaa mieltä myös siitä, että satelliitista nähdään tilanne samoin (että satelliitin kello on käynyt nopeammin).

Entä, ajatuskokeena, jos tilanteesta poistettaisiin gravitaatio, jäisi -7 mikrosekuntia nopeusvaikutusta satelliitin kelloon. Kun satelliitista katsottaisiin maan kelloon, olisiko se edellä vai jäljessä satelliitin kelloa? Anyone?

Einstein sanoi että inertiaalikoordinaatistoissa ei saada dilaatiota aikaan joten sen mukaan myös maan kellon pitäisi jätättää 7 mikrosekuntia. Mutta riippuu siitä keneltä spesialistilta kysytään, on erilaista ajattelua. Kuitenkaan nämä koordinaatistot ei ole inertiaalisia koska kaartavat, sekin sekoittaa laskuja. Hafele-Keating Wikissä kerrotaan uusintamittauksista mitä ollaan jopa 2000-luvulla tehty ja niistä sanotaan että todistavat suhteellisuuden puolesta. Kuitenkin pientä poikkeamaa niissäkin on.
Otsikko: Vs: Epäselvyys suhteellisuusteorian aika ja etäisyys käsityksissä
Kirjoitti: mistral - 15.08.2023, 00:43:09
Lainaus käyttäjältä: velihopea - 14.08.2023, 23:17:51
Tuo linkin kuvaus oli niin hienosti sanottu, että en ollenkaan osaa hahmottaa mikä kuvattu asia on ja mitä sillä on mihinkään (esim kellon käyntiin) vaikutusta. Linkin teksti jotenkin mallinsi painovoimaa maan pinnalla ja maan pyörähdysliikkeen yhteisvaikutusta.

Ymmärsin että tähtitieteilijät pitää gravitaatiopotentiaalin nollaa suurimpana potentiaalina ja näin käytännössä kaikki arvot on negatiivisia. Esim Voyagerkin on melkoisen kaukana nollasta ja tietysti James Webb vielä alempana. Mutta gravitaatiokaivoissa vasta alas mennäänkin.

Mutta geodesiassa ovat antaneet potentiaalille plus merkin, ilmeisesti sen asteikko voidaan valita vapaasti aina tilanteen mukaan. Vaikkapa pyramidin potentiaalienergia maan suhteen?

Mutta mitä halusin sanoa oli että tähtitieteessä lukema -0,0001 ei ole pieni vaan valtavan suuri :) Pieni potentiaali voisi olla vaikka -1111,0.

Alunperin kiinnitin huomiota gravitaatiopotentiaaliin niissä Einsteinin papereissa.
Otsikko: Vs: Epäselvyys suhteellisuusteorian aika ja etäisyys käsityksissä
Kirjoitti: velihopea - 15.08.2023, 17:09:17
Lainaus käyttäjältä: mistral - 15.08.2023, 00:10:49
Lainaus velihopea:
    Entä, ajatuskokeena, jos tilanteesta poistettaisiin gravitaatio, jäisi -7 mikrosekuntia nopeusvaikutusta satelliitin kelloon. Kun satelliitista katsottaisiin maan kelloon, olisiko se edellä vai jäljessä satelliitin kelloa? Anyone?

Einstein sanoi että inertiaalikoordinaatistoissa ei saada dilaatiota aikaan joten sen mukaan myös maan kellon pitäisi jätättää 7 mikrosekuntia. Mutta riippuu siitä keneltä spesialistilta kysytään, on erilaista ajattelua. Kuitenkaan nämä koordinaatistot ei ole inertiaalisia koska kaartavat, sekin sekoittaa laskuja. Hafele-Keating Wikissä kerrotaan uusintamittauksista mitä ollaan jopa 2000-luvulla tehty ja niistä sanotaan että todistavat suhteellisuuden puolesta. Kuitenkin pientä poikkeamaa niissäkin on.

Niin arvelinkin, että "virallinen ST-vastaus" on, että kun maasta näkyy satelliitin kello jätättaneen 7 mikrosekuntia, myös satelliitista näkyy maan kello jätättäneen 7 mikrosekuntia. Minun järkeeni käy vain, että maa ja satelliitti näkevät toistensä kellot "oikein" niinkuin tuossa GPS-tilanteessa.

Tälle relatiivisuustulkinnalle en tiedä muuta selitystä kuin se aiemmin keskustelemamme "symmetriatapaus", jonka AE selitti ajatuskokeella 1918 kirjoituksessaan. Ja jota kukaan tässä ryhmässä ei ole osannut selittää, miten se tarkkaan ottaen toimii tai on edes mahdollinen. Jos se vaikka olisikin mahdollinen, tässä tapahtuu vain yhdenlainen prosessi, ei sellainen toisenlaisen, jolla AE ilmiön selittää.

Modifioidaan ajatuskokeena GPS tilannetta. GPS-satelliitit kiertävät maapallon n. 12 h kiertoajalla milloin mitenkin päin maan kiertoliikkeen suhteen. Jospa se satelliitti lentääkin vain 10.000 m korkeudella, niin varmaan gravitaatio-nopeutus ja liike-hidastus satelliitin kelloon on erilainen, ja yhtä lailla laskettavissa. Ja yhä maasta ja satelliitista nähdään toistensa kellot "oikein".

Sitten tiputetaan satelliitin korkeutta asteettain, lopuksi että satelliitti lentää ihan maanpinnan tasossa. Tällöin satelliittiin ei tule yhtään sen enempää gravitaatio-nopeutusta kuin maan kelloonkaan. Jää vain satelliitin liike kellojen käynnin eron syyksi. Jos vielä 1 mm korkuisella lentoradalla maa ja satelliitti näkivät toistensa kellot "oikein", miten ihmeessa 0 korkuisella lentoradalla tapahtuu epäjatkuvuuskohta, että ristiinkatsomisnäkymä muuttuu mahdottomaksi.

Ok, maan pinnan ja satelliitin lentorata ei ole suora. Mutta jos otetaan vaikka 1 km pätkä sitä, ei se matka niin kauhean lenko ole, että menisikö suppean-ST:n ennustamat vaikutukset ihan pieleen. Ja kun kelloja lennätettiin 1971, samalla lailla maan ja lentokokeen kello kiersivät maapallon ympäri. Eikä sen raportoinneissa tuota matkan kaarevuutta mainittu ongelmana.

Jään käsitykseen, että "molemmat näkevät toisen kellon jättäneen omaa kelloaan 7 mikrosekuntia" -käsitys on ST-laumakäyttäytymistä, jonka syynä on väärinkäsitys. Mutta toivon, että käsitykseni osoitettaisiin vääräksi, ei pelkästään AE:hen vetoamalla.

Onko kellään yhteyksiä johonkin suureen guruun, joka voisi sanoa tähän jotain viisasta.
Otsikko: Vs: Epäselvyys suhteellisuusteorian aika ja etäisyys käsityksissä
Kirjoitti: Kaizu - 15.08.2023, 21:57:29
Lainaus käyttäjältä: velihopea - 15.08.2023, 17:09:17
Onko kellään yhteyksiä johonkin suureen guruun, joka voisi sanoa tähän jotain viisasta.

Albert Einstein, Stepen Hawking ja Richard Feynman eivät valitettavasti ole enää kekuudessamme.

Kaizu
Otsikko: Vs: Epäselvyys suhteellisuusteorian aika ja etäisyys käsityksissä
Kirjoitti: mistral - 16.08.2023, 01:08:19
Lainaus käyttäjältä: velihopea - 15.08.2023, 17:09:17

Sitten tiputetaan satelliitin korkeutta asteettain, lopuksi että satelliitti lentää ihan maanpinnan tasossa. Tällöin satelliittiin ei tule yhtään sen enempää gravitaatio-nopeutusta kuin maan kelloonkaan. Jää vain satelliitin liike kellojen käynnin eron syyksi. Jos vielä 1 mm korkuisella lentoradalla maa ja satelliitti näkivät toistensa kellot "oikein", miten ihmeessa 0 korkuisella lentoradalla tapahtuu epäjatkuvuuskohta, että ristiinkatsomisnäkymä muuttuu mahdottomaksi.

Ok, maan pinnan ja satelliitin lentorata ei ole suora. Mutta jos otetaan vaikka 1 km pätkä sitä, ei se matka niin kauhean lenko ole, että menisikö suppean-ST:n ennustamat vaikutukset ihan pieleen. Ja kun kelloja lennätettiin 1971, samalla lailla maan ja lentokokeen kello kiersivät maapallon ympäri. Eikä sen raportoinneissa tuota matkan kaarevuutta mainittu ongelmana.

ES:n mukaan kummankin mielestä toisen kello alkaa jätättää kun alus on saavuttanut relativistisen nopeuden. Sillä ei liene väliä onko välimatka mrd kilometriä tai mrd nanometriä. Ristiinkatsomisnäkymä on haaste suhteellisuudelle. Nykyparadigmassa se vaan ohitetaan mutta jos joku haasteen eli ongelman onnistuu ratkaisemaan, saa kyllä nimensä maailmanhistoriaan. Uskon että ainakin moni yrittää ratkaista ongelman mutta läpimurtoa ei ole tullut. Toisaalta sama tilanne on doppler-ilmiössä, kun Harley-Daavidsonit ohittaa toisensa , siis vastakkaisesta suunnasta, niin molempien mielestä toisen ääni madaltuu, tapahtuu "ristiinkuuleminen". Tämä ei kuitenkaan ole iso ongelma, se on ratkaistu ajat sitten.

Mielestäni Hafele-Keating kokeessa on sellainen vaikeus että lentokoneiden nopeus toistensa suhteen ja myös lentokentän suhteen elää jatkuvasti, ainakin vastakkaisiin suuntiin mennessä maan vastakkaisilla puolilla menevät hetken samassa inertiaalikoordinaatistossa eli eivät liiku toisiinsa nähden. Ei ole helppoa laskea sitäkään. Myös outoa on se että itään päin lento antoi eri tuloksen kuin länteen päin.
Otsikko: Vs: Epäselvyys suhteellisuusteorian aika ja etäisyys käsityksissä
Kirjoitti: velihopea - 16.08.2023, 12:10:01
Lainaus käyttäjältä: Kaizu - 15.08.2023, 21:57:29
LainaaLainaus velihopea
    Onko kellään yhteyksiä johonkin suureen guruun, joka voisi sanoa tähän jotain viisasta.
Albert Einstein, Stepen Hawking ja Richard Feynman eivät valitettavasti ole enää kekuudessamme.

Kaizu

Tarkoitin guruilla näitä kotimaisia elossa olevia emerituksia ja viroissa olevia, varmaan tiedätte keitä.

Jos nyt tässä kohden guru-kysymys olisi tuo viimeksi keskusteltu "ristiinnäkeminen" (K:sta ka K':sta molemmistä nähdään toisen kellon jätättäneen). Miten sen järkevä selitys kuuluu. Tai missä kohtaa edellä esittämässäni tapahtumaskenaariossa on nojattu ei-pätevään Newtonilaiseen ajatteluun. Tai kommentoikoon guru mitä tahansa, mitä pitää tässä olennaisena.
Otsikko: Vs: Epäselvyys suhteellisuusteorian aika ja etäisyys käsityksissä
Kirjoitti: velihopea - 29.08.2023, 23:56:29
Päivää taas. Viimeksi kirjoitin:

Lainaus käyttäjältä: velihopea - 15.08.2023, 17:09:17
Jään käsitykseen, että "molemmat näkevät toisen kellon jättäneen omaa kelloaan 7 mikrosekuntia" -käsitys on ST-laumakäyttäytymistä, jonka syynä on väärinkäsitys. Mutta toivon, että käsitykseni osoitettaisiin vääräksi, ei pelkästään AE:hen vetoamalla.

Onko kellään yhteyksiä johonkin suureen guruun, joka voisi sanoa tähän jotain viisasta.

Ei ole ilmaantunut asiantuntijaa, joka olisi laittanut itsensä peliin ja koettanut selittää minulle poloiselle miten olenkaan harhautunut. Nuo "laumakäyttäytyminen" ja "väärinkäsitys" on kai aika lailla provokatoorisia, joten yritän selventää, mitä niillä olen tarkoittanut. Ehkä silloin joku asiantuntija voisi paremmin nähdä missä kohden ajattelen väärin.

(Apropos, nyt muistaessani, lukiessani aiempia postauksiani huomaan, että olen itse kirjoittanut vääriä asioiden ymmärryksiä useaankin kohtaan, joissa nyt luulen ymmärtäväni asian oikeammin. Joten lukekaa varovaisuudella tekstejäni...)

Tässä väliaikana olen hankkinut minulle suositellun kirjan, lukenutkin sitä, AE: Erityisestä ja yleisestä suhteellisuusteoriasta ("yleistajuisesti" no-joo), 1917. Suomennus Raimo Lehti, 2003, on loistava. Hän on selventänyt  alaviitteissä useita AE:n epäselvästi sanomia kohtia sekä kirjoittanut tinkimättömällä otteella taustoittavat osiot "Suomentajan selityksiä Einsteinin kirjoituksiin" ja "Suomentajan komentaari" yli 300 sivua.

Mutta asiaan. Aiemmassa postauksessani siteerasin linkistä https://www.britannica.com/science/twin-paradox löytyvää tekstiä:

"    But in relativity, what one observer sees as happening to a second one, the second one sees as happening to the first one."

ja huudahdin, "Hienoa. Sain määritelmän mitä relatiivisuus mm. tarkoittaa ja edellyttää." Tarkoitin siis suhteellisuusperiaatetta suppeassa ST:ssa (viralliselta nimeltään Erityinen ST). Olin väärässä. Se ei tarkoita tuota, vaan se (principle of relativity) on "fysikaalinen periaate, jonka mukaan fysiikan lait ovat samat kaikissa inertiaalijärjestelmissä". [Oho/hmm..., wikipedia sanoi, että tuo on ensimmäinen ST:n postulaatti! Postulaattihan on todistamaton lähtöoletus! Mutta oletan nyt, että se pitää paikkansa.]

Väärinkäsityksellä tarkoitan tuon Britannica-sitaatin kaltaisen käsityksen nousemisen yleisen uskomuksen asemaan, ikäänkuin ST:n ominaisuudeksi. Ja ST-laumakäyttäytymisellä tarkoitan kritiikitöntä toistelua ST-piirien yleisiä uskomuksia. [Oho, tulipahan tykitettyä!]

Mutta kaipaatte varmaan perusteluja.

Suhteellisuusperiaate minusta sanoo esimerkiksi, että jos on liiketilat K (paikallaan, jossa kello U1) ja K' (liikkuu, jossa kello U2), kumpaaankiin liiketilaan, erikseen ja itsenäisesti, sovelletaan fysiikan lakeja mitä ne ovatkaan.

Minusta yksi ylikäymätön fysiikan laki on, että jos jotain on jossakin, se sitten on.

Käsittääkseni liiketilan K asiat (K:n ON-asiaintila, esim. U1:n lukema siellä) ei muutu vaikka sitä katsottaisiin kiikareilla (joiden signaalinopeus on ääretön) K':sta. Tai vaikka K':n kyytilänen tuntisi olevansa kuinka paikallaan, kun ei tunne olevansa liikkeessä ollenkaan, ja kuvittelisi että se onkin K joka liikkuu.

Toisin sanoen en osta käsitystä, että molemmat K että K' näkevät toisen kellon hidastuneen omaansa nähden.

Jottei tämä postaus menisi turhan pitkäksi, en vyörytä lisää argumentaatiota. Mutta jos joku pyytää, voin niitä esittää.

Mutta todella, please, koska käsitykseni ymmärtääkseni nakertaa yhtä ST:n merkittävää osa-aluetta, toivoisin, että joku asiantuntija osoittaisi missä kohden järkeilyni menee metsään.
Otsikko: Vs: Epäselvyys suhteellisuusteorian aika ja etäisyys käsityksissä
Kirjoitti: Eusa - 30.08.2023, 00:04:49
Toriaukion vastalaidoilla ovat raatihuone ja kirkko, joissa on ovisyvennys.

Instant-kameran optiikka vastaa vakioitua valonnopeutta ja kertoo minkä kokoisena paperille piirtyvät kohteet etäisyyden 45 m suhteen, saadaan gamma = 0,002. Tämä sama gamma pätee rakennusten syvennyksestä toistaan valokuvattaessa.

Kun raatihuoneelta kuvataan kirkon 30 m leveä julkisivu, piirtyy se paperille 6 cm levyisenä - 45 m levyinen raatihuone puolestaan piirtyy 9 cm levyisenä kirkolta kuvattuna.

Ei ole mieltä ajatella, että ensin kuvataan kirkko ja se pienenee 6 cm:iin, jonne vietynä pienenee myös kamera ja sitten kuvattuna raatihuone pienenisi 0,002^2 -kokoon eli mikrometriluokkaan.

Toisin sanoen, Lorentz-muunnoksessa kyse on vain kertaallisesta kahdenkeskisestä perspektiivistä - skaala on symmetrinen; valokuvauksessa etäisyys on sama, liikkeen mukaisessa ajan suhteellisuudessa nopeus on sama molemmille.
Otsikko: Vs: Epäselvyys suhteellisuusteorian aika ja etäisyys käsityksissä
Kirjoitti: velihopea - 30.08.2023, 11:02:54
Kiitos Eusa kommenteistasi. En oikein ymmärtänyt kommentteja, mutta kai lopputulemanasi oli, että minä olin väärässä.  Koska olen huono ymmärtämään asioita, muut kommentoijat, please, sanokaa selvästi "kyllä", "ei", "kyllä muttta...", "ei koska...", "ei kantaa mutta muu kommentti..."

Lainaus käyttäjältä: Eusa - 30.08.2023, 00:04:49
Toisin sanoen, Lorentz-muunnoksessa kyse on vain kertaallisesta kahdenkeskisestä perspektiivistä - skaala on symmetrinen; valokuvauksessa etäisyys on sama, liikkeen mukaisessa ajan suhteellisuudessa nopeus on sama molemmille.

"Kertaallinen kahdenkeskinen perspektiivi" on hiton hieno termi, jos vielä ymmärtäisin sen. Valokuvaus-rinnastuksesta en saanut irti pointtia. Mutta tuohon viimeiseen lauseeseen ehkä pääsin kiinni.

Siis ymmärtääkseni esität, minusta yleisen ST-virhekäsityksen mukaisesti, että ST voi tosta vaan rikkoa lanseeraamaani ON fysiikan lakia. Eli että vain ajattelun voimalla liikkuva voikin kuvitella olevansa paikallaan ja ajatella sen paikallaan olleen nyt liikkuvan. Ja kuvittelun jälkeen fysiikan lakeja voidaan soveltaa todella näin liikkuviin kappaleisiin.
Otsikko: Vs: Epäselvyys suhteellisuusteorian aika ja etäisyys käsityksissä
Kirjoitti: mistral - 30.08.2023, 17:14:44
Lainaus käyttäjältä: velihopea - 30.08.2023, 11:02:54
, muut kommentoijat, please, sanokaa selvästi "kyllä", "ei", "kyllä muttta...", "ei koska...", "ei kantaa mutta muu kommentti..."

Kuten sanoin jossain vaiheessa keskustelua, ES on epätäydellinen juuri siinä että hyväksytään ajankulun ristiin meno, siis että molemmat kellot menee ristiriitaisesti. Mikä ratkaisuksi? Auttaisiko se ettei juututa lepokoordinaatistoihin? Tämä tarkoittaa ettei laskua voisi tehdä jos molemmat luetaan lepokoordinaatistoiksi eli laskusääntöä pitää jotenkin muokata jos molemmat luetaan lepokoordinaatistoiksi. Todellisuuttahan ei voi muokata mutta laskusääntöä voi. Siispä säilytetään lepokoordinaatistot ja muokataan laskua.
Itse ottaisin sen suurennuslasi idean käyttöön, ja hioisin lasia niin että tulos ei olisi ristiriitainen. Suurennuslasi on yleensä symmetrinen, jos sen halkaisee keskeltä, tulee kaksi samanlaista osaa. Tässä olisi lähtökohta. Seuraavaksi tarvitaan puolisko miinus puolisko laskuoperaatio. Samanlaiset puoliskot tuottaa nollan, siis dilaatio olisi nolla puolin ja toisin, tällainen on todellisuus eli kellot ei ole ristiriidassa. Jos taas suurennuslasi on epäsymmetrinen, niin kun se halkaistaan, tulos ei olekaan nolla, näin kellojen aikaero saadaan laskettua. Lasku menisi niin että pienempi puolisko miinus isompi = negatiivinen linssi tai isompi puolisko miinus pienempi = positiivinen linssi. Näin saadaan suoraan kellojen erot laskettua.
Tiedä vaikka laskut olisi jo 100 vuotta sitten laskettu mutta niitä ei vaan löydy mistään??

Niin vielä suurennuslasista, eihän se ole kuin havainnollistus, mutta mistä löytyy fyysikko joka saa sen taipumaan kaavaksi?
Otsikko: Vs: Epäselvyys suhteellisuusteorian aika ja etäisyys käsityksissä
Kirjoitti: velihopea - 30.08.2023, 21:41:22
Kiitos mistral kommenteistasi.

Lainaus käyttäjältä: mistral - 30.08.2023, 17:14:44
Kuten sanoin jossain vaiheessa keskustelua, ES on epätäydellinen juuri siinä että hyväksytään ajankulun ristiin meno, siis että molemmat kellot menee ristiriitaisesti. Mikä ratkaisuksi? <erilaisia spekulaatioita...>

Mitä? ES:kö (ES=Erityinen suhteellisuusteoria?) epätäydellinen? Olisko käynyt niin satumainen onni, että vain me kaksi epäilemme tässä kohden jotain epäselvyyttä. Sinä koetat kiemurrella epäselvyydestä ulos kehittämällä erilaisia spekulaatiota, efektiivisesti uutta selittävää mallia, miten asiat voisivat olla. Minä taas ajattelen, että ST:llä täytyy olla tässäkin kohdat selvät ja perustellut sävelet miten asiat ovat. Kunhan vaan joku, joka tuntee ne meitä paremmin, kertoisi ne.

Apropos. Viimeisessä Tähdet ja avaruus lehdessä 5/2023 s. 57 oli aikaa koskeva yleisökysymys. Kari Enqvist vastasi siihen ja johdatti vastauksen miten Maasta ja raketista nähdään toisen kellonaika. Hän kirjoitti, että molemmmista päistä nähdään toisen kello jätättäneenä. Juuri tuota pidän ST:n (ES:n) vääränä tulkintana. Jos tuo kuitenkin on ST:n virallinen tulkinta, miten se perustellaan? Voiko lanseeraamani ON fysiikan lain ylijyrätä vain mielestämällä.
Otsikko: Vs: Epäselvyys suhteellisuusteorian aika ja etäisyys käsityksissä
Kirjoitti: mistral - 31.08.2023, 11:38:22
Lainaus käyttäjältä: velihopea - 30.08.2023, 21:41:22
Apropos. Viimeisessä Tähdet ja avaruus lehdessä 5/2023 s. 57 oli aikaa koskeva yleisökysymys. Kari Enqvist vastasi siihen ja johdatti vastauksen miten Maasta ja raketista nähdään toisen kellonaika. Hän kirjoitti, että molemmmista päistä nähdään toisen kello jätättäneenä. Juuri tuota pidän ST:n (ES:n) vääränä tulkintana. Jos tuo kuitenkin on ST:n virallinen tulkinta, miten se perustellaan? Voiko lanseeraamani ON fysiikan lain ylijyrätä vain mielestämällä.

Kyllä se vaikuttaa viralliselta tulkinnalta, siis juuri se missä on ristiriita. Uskon kyllä että nykyparadigma on oikeassa siinä merkityksessä että aika hidastuu kun nopeus lähestyy c:tä, ja näin ES on tässä mielessä oikeassa mutta sitten kun ES on viety loppuun, ollaan alistuttu ristiriidalle. Olisiko noudatettu Occamin partaveitsen ideaa: yksinkertainen selitys asetetaan etusijalle?

Kuitenkin ES:ssä vaikuttaa aina 2 koordinaatiston välillä kumoutuminen, siis että K ja K' kumoavat toistensa dilaation, tätä kumoutumista ei voi pitää vain laskuteknisenä asiana vaan sen juuret menevät syvälle luonnon olemukseen. Mieleen tulee kuva kahdesta suohon uponneesta marjastajasta, kun toinen ponnistaa ylöspäin toista painamalla vuorotellen, mitään nousua ei tapahdu. Ehkä jotain tällaista on K:n K':n välillä?
Otsikko: Vs: Epäselvyys suhteellisuusteorian aika ja etäisyys käsityksissä
Kirjoitti: Eusa - 31.08.2023, 19:17:58
Lainaus käyttäjältä: velihopea - 30.08.2023, 11:02:54
Kiitos Eusa kommenteistasi. En oikein ymmärtänyt kommentteja, mutta kai lopputulemanasi oli, että minä olin väärässä.  Koska olen huono ymmärtämään asioita, muut kommentoijat, please, sanokaa selvästi "kyllä", "ei", "kyllä muttta...", "ei koska...", "ei kantaa mutta muu kommentti..."

"Kertaallinen kahdenkeskinen perspektiivi" on hiton hieno termi, jos vielä ymmärtäisin sen. Valokuvaus-rinnastuksesta en saanut irti pointtia. Mutta tuohon viimeiseen lauseeseen ehkä pääsin kiinni.

Siis ymmärtääkseni esität, minusta yleisen ST-virhekäsityksen mukaisesti, että ST voi tosta vaan rikkoa lanseeraamaani ON fysiikan lakia. Eli että vain ajattelun voimalla liikkuva voikin kuvitella olevansa paikallaan ja ajatella sen paikallaan olleen nyt liikkuvan. Ja kuvittelun jälkeen fysiikan lakeja voidaan soveltaa todella näin liikkuviin kappaleisiin.
Kyllä. Kuvitellaanpa. Nimittäin fysikaalisesti oleellista on kunkin ikääntyminen ja vertailussa tapahtuuko ikääntymiseroja.

Olkoon kaksi avaruusalusta ajamassa relativistisella tasaisella nopeudella toisiaan kohti. Molemmat toteavat Suppean Suhteellisuuden opeilla toisen kellon käyvän hitaammin kuin omansa. Ne vaihtavat keskenään signaaleja. Keskiarvoistuksen perusteella ne kuitenkin onnistuvat synkronoimaan kellonsa sovittaen niin, että etäisyys ennakoituun kohtauspaikkaan kellojen synkronointiperusteessa on molemmilla sama etäisyys. Kohdatessaan alukset vertailevat kellojaan ja huomaavat niiden olevan edelleen synkronissa ja molemmat ovat ikääntyneet saman etäisyyden matkalla yhtä paljon.

Ei ole paradoksia, koska perspektiivi merkitsee vain sitä, että etäisyydessä tapahtuva projisoituu merkitykselliseksi informaatioksi toiselle ajan hidastumisena ja etäisyyksien lyhenemisenä - sillä ei ole mitään fysikaalista muutosvoimaisuutta.
Otsikko: Vs: Epäselvyys suhteellisuusteorian aika ja etäisyys käsityksissä
Kirjoitti: mistral - 31.08.2023, 22:37:12
Lainaus käyttäjältä: Eusa - 31.08.2023, 19:17:58
Ei ole paradoksia, koska perspektiivi merkitsee vain sitä, että etäisyydessä tapahtuva projisoituu merkitykselliseksi informaatioksi toiselle ajan hidastumisena ja etäisyyksien lyhenemisenä - sillä ei ole mitään fysikaalista muutosvoimaisuutta.

Erikoista on se että pituuskontraktio ilmenee vain etenemissuunnassa mutta dilaatio taas ilmenee kaikissa suunnissa. Ainakin youtubella näytettiin kuinka kaupungin katukiveys oli pienessä nopeudessa suora poikittaissuunnassa mutta rel. nopeudessa kiveys kaareutuu.
Otsikko: Vs: Epäselvyys suhteellisuusteorian aika ja etäisyys käsityksissä
Kirjoitti: Eusa - 01.09.2023, 23:19:16
Lainaus käyttäjältä: mistral - 31.08.2023, 22:37:12
Erikoista on se että pituuskontraktio ilmenee vain etenemissuunnassa mutta dilaatio taas ilmenee kaikissa suunnissa. Ainakin youtubella näytettiin kuinka kaupungin katukiveys oli pienessä nopeudessa suora poikittaissuunnassa mutta rel. nopeudessa kiveys kaareutuu.
Eihän tuo ole totta. Aikadilaatio ja pituuskontraktio esiintyy kaikissa suunnissa liikkuvissa koordinaatistoissa. ;)

No, perspektiivikuvassakin mittojen lyheneminen tapahtuu vain syvyyssuunnassa, ei kuva-alassa sivusuunnassa, kun seurataan vakioetäisyydellä olevaa piirrettä. Sen sijaan kaikissa suunnissa etäisyys sinänsä voi vaihdella. Samoin kuin Lorentz-muunnoksessa pituus ja aika yhdessä kuvaavat invariantin muuttumattoman tapahtumien intervallin, perspektiivikuvassa syvyyden mukana tapahtuva pieneneminen ja etäisyys kohteeseen yhdessä säilyttävät tiedon mittakaavainvarianssista; etäisyys kohteeseen x kohteen leveys säilyy vietiinpä esine mihin tahansa.

Yritän pakotetusti kuvata aihetta perspektiivikuvavertauksella, koska oikeasti kyse on vain aivan samasta näkökulmaperiaatteesta.
Otsikko: Vs: Epäselvyys suhteellisuusteorian aika ja etäisyys käsityksissä
Kirjoitti: mistral - 02.09.2023, 11:07:38
Muistaakseni tallensin videon mutta saattaa olla toisella koneella. Siinä oli aito kaupunkikuva.
Otsikko: Vs: Epäselvyys suhteellisuusteorian aika ja etäisyys käsityksissä
Kirjoitti: mistral - 02.09.2023, 22:11:50
Tämmöinen se olikin:
https://www.spacetimetravel.org/aur/1
Otsikko: Vs: Epäselvyys suhteellisuusteorian aika ja etäisyys käsityksissä
Kirjoitti: mistral - 02.09.2023, 23:23:20
Lainaus käyttäjältä: velihopea - 30.08.2023, 21:41:22
Mitä? ES:kö (ES=Erityinen suhteellisuusteoria?) epätäydellinen? Olisko käynyt niin satumainen onni, että vain me kaksi epäilemme tässä kohden jotain epäselvyyttä. Sinä koetat kiemurrella epäselvyydestä ulos kehittämällä erilaisia spekulaatiota, efektiivisesti uutta selittävää mallia, miten asiat voisivat olla. Minä taas ajattelen, että ST:llä täytyy olla tässäkin kohdat selvät ja perustellut sävelet miten asiat ovat. Kunhan vaan joku, joka tuntee ne meitä paremmin, kertoisi ne.

Olettaisin että Einstein perustautui lepokoordinaatistoihin siksi että koordinaatisto on vain yksi luonnon tila, siinä ei ole mitään tietoista valintaa olla levossa tai olla tietyssä nopeudessa. Siksi kaikki koordinaatistot on vain luonnon tilassa jossa ne vuorovaikuttavat ympäristönsä kanssa. Vaikka massaan olisi pumpattu liike-energiaa kuinka paljon tahansa, se vaan on vuorovaikutuksessa ympäristönsä kanssa. Vuorovaikutus on se ydin, siitä suhteellisuusteoria on saanut nimensä.
Se että teoria tuottaa ristiriitaisia kellonaikoja tai yhteisen nyt-hetken menetyksen, pitäisi innostaa ratkaisemaan nämä ongelmat mutta kukaan ei ole keksinyt ratkaisua.
Otsikko: Vs: Epäselvyys suhteellisuusteorian aika ja etäisyys käsityksissä
Kirjoitti: Eusa - 03.09.2023, 11:04:02
Lainaus käyttäjältä: mistral - 02.09.2023, 23:23:20
Olettaisin että Einstein perustautui lepokoordinaatistoihin siksi että koordinaatisto on vain yksi luonnon tila, siinä ei ole mitään tietoista valintaa olla levossa tai olla tietyssä nopeudessa. Siksi kaikki koordinaatistot on vain luonnon tilassa jossa ne vuorovaikuttavat ympäristönsä kanssa. Vaikka massaan olisi pumpattu liike-energiaa kuinka paljon tahansa, se vaan on vuorovaikutuksessa ympäristönsä kanssa. Vuorovaikutus on se ydin, siitä suhteellisuusteoria on saanut nimensä.
Se että teoria tuottaa ristiriitaisia kellonaikoja tai yhteisen nyt-hetken menetyksen, pitäisi innostaa ratkaisemaan nämä ongelmat mutta kukaan ei ole keksinyt ratkaisua.
Ei ole mitään ongelmia - tai ne ovat opiskelijassa, ei aiheessa.
Otsikko: Vs: Epäselvyys suhteellisuusteorian aika ja etäisyys käsityksissä
Kirjoitti: mistral - 03.09.2023, 13:27:42
Lainaus käyttäjältä: mistral - 02.09.2023, 22:11:50
Tämmöinen se olikin:
https://www.spacetimetravel.org/aur/1

Myös minun mielestä tässä on virhe, Eusa löydätkö saman virheen?
Otsikko: Vs: Epäselvyys suhteellisuusteorian aika ja etäisyys käsityksissä
Kirjoitti: Eusa - 03.09.2023, 18:32:42
Lainaus käyttäjältä: mistral - 03.09.2023, 13:27:42
Myös minun mielestä tässä on virhe, Eusa löydätkö saman virheen?
Enpä hoksaa virhettä.
Otsikko: Vs: Epäselvyys suhteellisuusteorian aika ja etäisyys käsityksissä
Kirjoitti: mistral - 03.09.2023, 21:15:03
Lainaus käyttäjältä: Eusa - 03.09.2023, 18:32:42
Enpä hoksaa virhettä.

Ainakin se on virhe että mitä sivummalle katsotaan liikesuunnasta, sitä pienempi on kontraktio. Tämä tarkoittaa että sivusuunnassa se on nolla. Tämä taas tarkoittaa että rel. nopeudessa sivulla oleva kappale joudutaan ohittamaan yli c nopeudella.
Mainitsin jossain aiemmassa viestissä juuri tästä mutta se oli vain viittaus siihen että kontraktio pienenisi sivusuunnassa, siis että tällaista tulkintaa on. Mutta oma järki sanoo että koko leveydeltä pitää avaruuden litistyä, silloin ohitettavan kappaleen kohdalla nopeus on alle c.
Otsikko: Vs: Epäselvyys suhteellisuusteorian aika ja etäisyys käsityksissä
Kirjoitti: Eusa - 03.09.2023, 22:39:38
Lainaus käyttäjältä: mistral - 03.09.2023, 21:15:03
Ainakin se on virhe että mitä sivummalle katsotaan liikesuunnasta, sitä pienempi on kontraktio. Tämä tarkoittaa että sivusuunnassa se on nolla. Tämä taas tarkoittaa että rel. nopeudessa sivulla oleva kappale joudutaan ohittamaan yli c nopeudella.
Mainitsin jossain aiemmassa viestissä juuri tästä mutta se oli vain viittaus siihen että kontraktio pienenisi sivusuunnassa, siis että tällaista tulkintaa on. Mutta oma järki sanoo että koko leveydeltä pitää avaruuden litistyä, silloin ohitettavan kappaleen kohdalla nopeus on alle c.
Tuollaiset kuvauksethan perustuvat havaittaviin signaaleihin ja nimenomaan signaaliviiveistä johtuvat saadut vääristyneet näkymät. Lorentz-muunnettu mallinnus ottaa lähtökohdaksi koordinaatiston samanaikaisuustason ja silloin on kyseessä ihan muu kuin tuollaiset havaintokuvaukset.
Otsikko: Vs: Epäselvyys suhteellisuusteorian aika ja etäisyys käsityksissä
Kirjoitti: mistral - 04.09.2023, 00:25:49
Lainaus käyttäjältä: Eusa - 03.09.2023, 22:39:38
Tuollaiset kuvauksethan perustuvat havaittaviin signaaleihin ja nimenomaan signaaliviiveistä johtuvat saadut vääristyneet näkymät. Lorentz-muunnettu mallinnus ottaa lähtökohdaksi koordinaatiston samanaikaisuustason ja silloin on kyseessä ihan muu kuin tuollaiset havaintokuvaukset.

Jos olisi tutkakuva, niin sitten ymmärtäisin signaaliviiveet mutta kun on kamera joka ottaa vastaan jo kennolle tulleita aaltoja niin mitä viiveitä siinä on?
Otsikko: Vs: Epäselvyys suhteellisuusteorian aika ja etäisyys käsityksissä
Kirjoitti: Eusa - 04.09.2023, 04:23:02
Lainaus käyttäjältä: mistral - 04.09.2023, 00:25:49
Jos olisi tutkakuva, niin sitten ymmärtäisin signaaliviiveet mutta kun on kamera joka ottaa vastaan jo kennolle tulleita aaltoja niin mitä viiveitä siinä on?
Esim. Lorentz-samanaikaisuuden perusteella taakse jäävät kohteet ovat menneisyyttä, mutta valokuvattaessa ne näkyvät, koska signaaliviiveen takia suurella nopeudella voi saavuttaa hieman takaviistostakin suuntautuneen valon.
Otsikko: Vs: Epäselvyys suhteellisuusteorian aika ja etäisyys käsityksissä
Kirjoitti: mistral - 04.09.2023, 11:30:34
Lainaus käyttäjältä: Eusa - 04.09.2023, 04:23:02
Esim. Lorentz-samanaikaisuuden perusteella taakse jäävät kohteet ovat menneisyyttä, mutta valokuvattaessa ne näkyvät, koska signaaliviiveen takia suurella nopeudella voi saavuttaa hieman takaviistostakin suuntautuneen valon.

Ai niin saattaa olla mahdollista. Nyt jos jaetaan valon nopeus 2 komponenttiin niin takaviistosta tuleva valo menee A eteenpäin B sivulle. Kun valon nopeus on riippumaton kennon liiketilasta niin A komp. kääntyykin miinus A komp'iksi, siis suunta muuttuu vastakkaiseksi.
Muokkaus:
Tarkemmin ajatellen ei A välttämättä muutu ihan miinusmerkkiseksi mutta  kuitenkin paljon. A:n ja B:n resultantti pitää olla kennoon nähden c-nopeuksinen.
Otsikko: Vs: Epäselvyys suhteellisuusteorian aika ja etäisyys käsityksissä
Kirjoitti: velihopea - 04.09.2023, 23:06:50
Päivää taas. Tosiaan ei ole ilmaantunut asiantuntijaa, joka olisi osoittanut ST-käsitykseni vääräksi. Siis minkä? Että 1) paikallaan olevansta tilasta K "nähdään" liikkuvan tilan K' kellon jätättäneen, ja muka K':sta "nähdään" K:n kellon yhtä lailla jätättäneen. Tuota 2)-kohtaa pidän yleisenä ST:n väärinkäsityksenä.

Siispä vyörytän lisää argumentaatiota. Olen lukenut AE:n (Einsteinin) kirjaa "Erityisestä ja yleisestä suhteellisuusteoriasta. Se on kirjoitettu 1916/1917, kun AE oli saanut teoriansa valmiiksi. Johdannossa AE sanoo, että esittää "ST:n keskeiset ajatukset mahdollisimman selkeästi ja yksinkertaisesti". Etsin kirjasta tietoa, joka tukisi tuota 2)-tulkintaa. En löytänyt.

Arvelen, että 2)-tulkinnan mukainen väärinkäsitys on syntynyt esim kun AE sanoo selittäessän asioita ratapenger+junanvaunu -esimerkillä:
"Senpä takia [vaunussa] sisällä olija voikin vastaan panematta tulkita siten, että vaunu pysyy paikallaan, mutta ratapenger liikkuu. Tämä tulkinta on sitä paitsi erityisen suhteellisuusperiaatteen mukaan fysikaalisesti täysin oikeutettu."

Suhteellisuusperiaate (jo Galileon esittämä) on vaatimus, että fysiikan lakeja kuvaavat yhtälöt ovat samat kaikissa liikekehyksissä. Ei muuta. Minusta, vaikka matkustajasta kuinka tuntuu, että ratapenger ja maisema liikkuu, se on silti juna joka liikkuu. Juna ja ratapenger eivät ole symmetriset liiketilat sen suhteen kumpi de-facto liikkuu.

Jos AE olisi ajatellut, että edellä kirjoittamastaan seuraisi, että niin K:sta kuin K':stakin "nähdään" toisen kellon hidastuneen, minusta se on niin tajunnan räjäyttävää, että kai AE olisi sellaisen shokkituloksen tuonut esiin. Mutta ei ole.

Tässä kirjassa, ja kai muissakin teksteissä, AE:n tapa on ollut kirjoittaa usein epätäsmällisesti ja tukinnanvaraisesti. Liki kaikille sivuille suomentaja (Raimo Lehti, 2003) on pitänyt tarpeellisena laittaa useita selittäviä alaviitteitä. RL kirjoittaa myös kommentaariosassa (s. 480):
"Suhteellisuusperiaate on tietenkin Einsteinin suhteellisuusteorioiden avainkäsite. Einsteinin omat sitä koskevat tekstit ovat ambivalentteja tavalla, mikä on antanut aiheen periaatteen erilaisiin tulkintoihin."

Tuon kirjan jälkeen kaivoin taas esiin jo käsitellyn  https://einsteinpapers.press.princeton.edu/vol7-trans/82, 1918, "Dialog about relatitivity theory" sivut 66-75. Siinä ST-epäilijä ja ST-puolustaja keskustelevat. AE on kirjoittanut molempien vuorosanat.

Ensin tarinassa käsitellään tapaus, jossa kello U2 liikkuu ees-taas ja lopussa se on jätättänyt paikallaan ollutta kelloa U1 (~kaksosparadoksi). Tästä molemmat ovat samaa mieltä. Sitten ST-epäilijä sanoo:
~"Mutta relatiivisuusperiatteen mukaan koko prosessi pitää tapahtua tarkalleen samalla tavalla kun tulkitaankin, että U2 on paikallaan ja se onkin U1 joka liikkuu ees-taas. Ja nyt pitäisi U1:n puolestaan olla jätättänyt U2:ta. Mikä on mahdotonta, että molemmat ovat jätättäneen toistaan".
Tähän ensin totean: Aha, jo tuolloin 1918 AE on tunnistanut, että relatiivisuusperiaatteen kylkeen on pesiytynyt tuollainen käsitys.

Epäilijälle AE vastaa:
~"Joo joo, mutta liiketilat K ja K' eivät ole suppean ST:n tarkoittamia samanlaisia systeemejä. Eli ne ei ole tasavauhtisia suoraviivaisesti kulkevia yms."
Ok, näin AE pääsi osin selitys-pälkähästä, koska tilanteessa oli (ees-taa liikettä ja) kiihtyvyyksiä. Mutta AE jatkaa selitystä nojautuen yleiseen ST:aan:
~"On todellakin oikein että yleisessä ST:ssä me voimme käyttää koordinaattisysteemi K:ta kuin K':akin. Mutta voidaan helposti nähdä, että systeemit K ja K' eivät ole ollenkaan ekvivalentteja mitä tulee millaiset prosessit tapahtuvat kummassakin tilanteessa".
Ja näitä prosesseja, jotka nimesin "perustilanne" ja "symmetriatilanne", olen kuvannut postauksissa #31 ja #32.

Siis teen johtopäätöksen, että AE:nkin mielestä symmetriatilanne ei aktivoidu vain ajattelemalla, että "minä olenkin tässä nyt paikallaan". Pitää oikeasti saada se toinen liikkumaan, eli saada se toinen prosessi tapahtumaan, eikä siihen riitä ST-epäilijän "kun tulkitaankin" katsantokanta.

Tapaus, jossa raketti lähtee maasta 0.8c vauhtia ja hetken päästä katsotaan ristiin mitä kello näyttää maassa ja raketissa, on minusta samanlainen kuin tässä edellä puhuttu.

Yhteenvetona: pidän vääränä nykyistä ilmeisesti yleistä ST-käsitystä, että molemmmista liiketiloista nähdään toisen kello jättäneenä.

Seuraavassa postauksessa aion esittää mieltä ravisuttavan ajatusharjoituksen, joka varmemmaksi vakuudeksi osoittaa 2)-käsityksen mahdottomuuden.
Otsikko: Vs: Epäselvyys suhteellisuusteorian aika ja etäisyys käsityksissä
Kirjoitti: Eusa - 04.09.2023, 23:16:31
Lorentz-muunnoksen ollessa täysin vastaava näkökulmaprojektio kuin perspektiivivalokuvaus nuo Velihopean kuvittelemat ristiriidat kuivuvat kasaan. Vasta signaalisimuloinnin sini- tai punasiirtymät aiheuttavat tilanteen, jossa ikäännytään eri tahtia.
Otsikko: Vs: Epäselvyys suhteellisuusteorian aika ja etäisyys käsityksissä
Kirjoitti: velihopea - 05.09.2023, 11:54:55
Tässä pyrin ajatuskokeella osoittamaan vääräksi väitteen, että kahdesta liiketilasta katsomalla ristiin näkisi aina sen toisen kellon jätättäneen, niinkuin esim. TA-lehden 5/2023 yleisökysymyksen vastauksessa esitetään.

Otetaan kaksi rakettia. Raketti 1 (R1) kulkee tasaista 0.8c vauhtia. Yhdessä tunnissa se kulkee ~864'' km. Raketti 2 (R2) kulkee tasaista 0.4c vauhtia. Tunnissa R2 kulkee puolet R1:n matkasta ~432'' km.

Valolta (tai radioviestiltä) kuluu 864'' km matkan taittamiseen 48 min ja puoleen väliin 24 min.

Maassa ja molemmissa raketeissa on tarkat kellot.

AE:n kaavan mukaan 0.8c vauhtinen R1:n kello hidastuu maan kelloon nähden ~0.6-osaan (maa 60 min, R1 ~36 min). Vastaavasti 0.4c vauhtinen R2:n kello hidastuu ~0.92 osaan (maa 60 min, R2 ~55 min).

Ensin lähtee R2. Se ajelee kaikessa rauhassa puoleen väliin 432'' km päähän ja pysäköi. Kun R2 on pysähtynyt, se on samassa liiketilassa kuin maa ja sen kello käy samaan tahtiin maan kellon kanssa. R2 lähettää maahan suuntaan viestin "start". R2 odottaa vielä paikallaan 24 min, ja kun maa-asema ja odottava R1 ovat juuri saaneet start-viestin, tapahtuu:
- R2 nollaa kellonsa ja säntää 0.4c vauhdilla eteenpäin
- R1 nollaa kellonsa ja säntää 0.8c vauhdilla R2:n perään
- maa-asema nollaa kellonsa ja alkaa lähettämään minuutin välein kellonaikaansa ja kuuntelee mitä raketit lähettävät
- raketit kuuntelevevat mitä maa-asema lähettää ja lähettävät maa-asemalle minuutin välein kellonsa ajan

On saatu järjestely, jossa voidaan "katsoa" liiketiloista toiseen mitä kello siellä on, informaation välitysaika huomioiden.

Kun (maan kellon mukaan) 60 min on kulunut R1 saavuttaa R2:n. Ohitushetkellä kuskit moikkaavat toisiaan ja kysyvät minkä kellolukeman sait maasta. Kysymys on siitä saivatko molemmat saman lukeman.

Nimittäin TA-lehden vastauksessa, ja muissakin yhteyksissä, esitetään myös valiidina tulkintamallina, että raketti on paikallaan ja maa liikkuu. Joten maan kello jätättää raketin kelloon nähden. Kuinka paljon? Tietysti AE:n kaavan mukaan. R1:n tapauksessa se tarkoittaa, että maan kello on hidastunut 0.6-osaan mitä R1:n kello käy. R2:n tapauksessa maan kello olisi hidastunut 0.92-osaan mitä R2:n kello käy.

Näin ei ollut. Maan kello ei ollut valikoidusti hidastunut erikseen R1:een nähden ja R2:een nähden. Yksi viesti ei voi valikoida arvoaan sen mukaan kuka sen vastaanottaa. Itse asiassa olikohan maan kello hidastunut ollenkaan?

Tästä päättelen, että esitetty tulkintamalli "aina toisen kello jätättää" ei voi olla oikea, koska se johtaa ristiriitaan.
Otsikko: Vs: Epäselvyys suhteellisuusteorian aika ja etäisyys käsityksissä
Kirjoitti: Eusa - 05.09.2023, 19:11:33
Lainaus käyttäjältä: velihopea - 05.09.2023, 11:54:55
Tässä pyrin ajatuskokeella osoittamaan vääräksi väitteen, että kahdesta liiketilasta katsomalla ristiin näkisi aina sen toisen kellon jätättäneen, niinkuin esim. TA-lehden 5/2023 yleisökysymyksen vastauksessa esitetään.

Otetaan kaksi rakettia. Raketti 1 (R1) kulkee tasaista 0.8c vauhtia. Yhdessä tunnissa se kulkee ~864'' km. Raketti 2 (R2) kulkee tasaista 0.4c vauhtia. Tunnissa R2 kulkee puolet R1:n matkasta ~432'' km.

Valolta (tai radioviestiltä) kuluu 864'' km matkan taittamiseen 48 min ja puoleen väliin 24 min.

Maassa ja molemmissa raketeissa on tarkat kellot.

AE:n kaavan mukaan 0.8c vauhtinen R1:n kello hidastuu maan kelloon nähden ~0.6-osaan (maa 60 min, R1 ~36 min). Vastaavasti 0.4c vauhtinen R2:n kello hidastuu ~0.92 osaan (maa 60 min, R2 ~55 min).

Ensin lähtee R2. Se ajelee kaikessa rauhassa puoleen väliin 432'' km päähän ja pysäköi. Kun R2 on pysähtynyt, se on samassa liiketilassa kuin maa ja sen kello käy samaan tahtiin maan kellon kanssa. R2 lähettää maahan suuntaan viestin "start". R2 odottaa vielä paikallaan 24 min, ja kun maa-asema ja odottava R1 ovat juuri saaneet start-viestin, tapahtuu:
- R2 nollaa kellonsa ja säntää 0.4c vauhdilla eteenpäin
- R1 nollaa kellonsa ja säntää 0.8c vauhdilla R2:n perään
- maa-asema nollaa kellonsa ja alkaa lähettämään minuutin välein kellonaikaansa ja kuuntelee mitä raketit lähettävät
- raketit kuuntelevevat mitä maa-asema lähettää ja lähettävät maa-asemalle minuutin välein kellonsa ajan

On saatu järjestely, jossa voidaan "katsoa" liiketiloista toiseen mitä kello siellä on, informaation välitysaika huomioiden.

Kun (maan kellon mukaan) 60 min on kulunut R1 saavuttaa R2:n. Ohitushetkellä kuskit moikkaavat toisiaan ja kysyvät minkä kellolukeman sait maasta. Kysymys on siitä saivatko molemmat saman lukeman.

Nimittäin TA-lehden vastauksessa, ja muissakin yhteyksissä, esitetään myös valiidina tulkintamallina, että raketti on paikallaan ja maa liikkuu. Joten maan kello jätättää raketin kelloon nähden. Kuinka paljon? Tietysti AE:n kaavan mukaan. R1:n tapauksessa se tarkoittaa, että maan kello on hidastunut 0.6-osaan mitä R1:n kello käy. R2:n tapauksessa maan kello olisi hidastunut 0.92-osaan mitä R2:n kello käy.

Näin ei ollut. Maan kello ei ollut valikoidusti hidastunut erikseen R1:een nähden ja R2:een nähden. Yksi viesti ei voi valikoida arvoaan sen mukaan kuka sen vastaanottaa. Itse asiassa olikohan maan kello hidastunut ollenkaan?

Tästä päättelen, että esitetty tulkintamalli "aina toisen kello jätättää" ei voi olla oikea, koska se johtaa ristiriitaan.
Kun R1 saavuttaa R2:n, molemmat havaitsevat toistensa kellot hidastuneiksi omasta näkökulmastaan johtuen aikadilataatiosta. Tämä johtuu siitä, että eri nopeudella liikkuvissa kehyksissä kulkeva aika projisoituu tarkastelukehykseen aina hidastuneena aivan kuten perspektiivikuvassa kaverin kämmen näyttää aina pienemmältä kuin oma kämmen. Aikadilataatio ja Lorentzin transformaatiot huomioivat tämän "kellon jätättämisen" - kuten valokuvaaminen huomioi perspektiivin.

Maan kello ei ole "valikoidusti hidastunut" R1:een tai R2:een nähden. Maapallon kellon aika on tarkastelureferenssi tässä skenaariossa, koska se ei liiku. R1:n ja R2:n kellot sen sijaan hidastuvat suhteessa Maan kellon aikaan.
Otsikko: Vs: Epäselvyys suhteellisuusteorian aika ja etäisyys käsityksissä
Kirjoitti: velihopea - 06.09.2023, 10:17:51
Kiitos Eusa kommenteistasi. Tällä kertaa luulen, että jopa ymmärsin ne. Ja mikä vielä ihmeellisempää, tuntuu kuin olisin about samaa mieltä.

Koska samalla lailla ymmärtäminen on vaikea laji ja päädyitkö samaan johtopäätökseen (= ST:aa opetetaan tässä kohtaa väärin), varmemmaksi vakuudeksi käyn kommenttisi läpi [vh ... vh] suluin.

Kirjoitit:
Kun R1 saavuttaa R2:n, molemmat havaitsevat toistensa kellot hidastuneiksi omasta näkökulmastaan johtuen aikadilataatiosta.
[vh Tuo "havaitsevat toistensa" -relaatio on kai vain yhteen suuntaan. Eli R1:n kello on hidastunut suhteessa R2:een, ei toisinpäin. vh]

...Tämä johtuu siitä, että eri nopeudella liikkuvissa kehyksissä kulkeva aika projisoituu tarkastelukehykseen aina hidastuneena
[vh Tarkennoksena, minusta tuo pitää paikkansa kun "eri nopeudella" muutetaan "suuremmalla nopeudella":ksi. Eli nopeammassa kehyksessä kello käy hitaammin kuin hitaammassa (jopa paikallaan olevassa) kehyksessä vh]

...aivan kuten perspektiivikuvassa kaverin kämmen näyttää aina pienemmältä kuin oma kämmen. Aikadilataatio ja Lorentzin transformaatiot huomioivat tämän "kellon jätättämisen" - kuten valokuvaaminen huomioi perspektiivin.
[vh Noista valokuva rinnastuksista en osaa sanoa mitään. Olennainen on kai jo edellä sanottu vh]

...Maan kello ei ole "valikoidusti hidastunut" R1:een tai R2:een nähden.
[vh Samaa mieltä. Ja jos maasta poispäin on menossa useita raketteja eri nopeuksilla, ja jos niissä kuskit saavat päähänsä, että ovatkin paikallaan ja maa liikkuu, ei maan kello voi vastata kaikkien rakettikuskien kuvitelmaan. Voikohan edes yhteenkään vaan pitää sinnikkäästi oman käyntinsä. vh]
[vh ST-opetus nyt sanoo, että raketista katsoen maan kello on hidastunut. Sen olisi pitänyt olla hidastunut R1:lle ja R2:lle eri lailla. Mutta koejärjestely osoitti, että näin ei ollut. vh]

...Maapallon kellon aika on tarkastelureferenssi tässä skenaariossa, koska se ei liiku. R1:n ja R2:n kellot sen sijaan hidastuvat suhteessa Maan kellon aikaan.
[vh Kyllä vh]
[vh Eli teen johtopäätöksen, että sinunkin mielestä tämä kohta ST:sta opetetaan väärin. vh]
Otsikko: Vs: Epäselvyys suhteellisuusteorian aika ja etäisyys käsityksissä
Kirjoitti: Eusa - 06.09.2023, 15:12:02
Velihopea,

Et ymmärtänyt. Kyllä sinun kannattaa mieltää kunnolla se kuinka silmiesi edessä huiskuttamasi oma kämmen on "itsesi kokoinen" mutta 20 m päässä kaverisi huiskutuskämmen on kontraktoitunut pieneksi - ja näkymäala kämmenen ympärillä vastaavasti dilatoitunut laajaksi. Kaverisi toteaa samat asiat, mutta mittaerot ovat hänen kannaltaan suhteessa hänen kehykseensä.

Sama tilanne on R1: ja R2:n välillä - siinä kämmenen tilalle vain laitetaan liikkeen suuntainen etäisyys ja ympäröivän näköalan tilalle aika.

Eli samoin kuin jokainen näkee muiden kämmenet pienentyneinä, on Lorentz-muunnoksessa kaikkien muiden liikkeen suuntaiset etäisyydet lyhentyneet - ja samoin kuin jokainen näkee kaikkien muiden kämmentä ympäröivän näkymäsektorin laajentuneen (dilatoituneen), on Lorentz-muunnoksessa kaikkien muiden ajat (kellot) hidastuneet, ajan kulut laajentuneet (dilatoituneet).

Jos ei tästä vertauksesta asia avaudu, niin tuskinpa mistään muustakaan selittämisestä olisi hyötyä.
Otsikko: Vs: Epäselvyys suhteellisuusteorian aika ja etäisyys käsityksissä
Kirjoitti: mistral - 06.09.2023, 16:06:18
Lainaus käyttäjältä: velihopea - 06.09.2023, 10:17:51
Kirjoitit:
Kun R1 saavuttaa R2:n, molemmat havaitsevat toistensa kellot hidastuneiksi omasta näkökulmastaan johtuen aikadilataatiosta.
[vh Tuo "havaitsevat toistensa" -relaatio on kai vain yhteen suuntaan. Eli R1:n kello on hidastunut suhteessa R2:een, ei toisinpäin. vh]

Kaikki koordinaatistot on kelluvia, ei ole lukittuja koordinaatistoja. Eri asia jos sovitaan että joku on lukossa, syy voi olla laskutekninen. Mutta käsittääkseni Einstein piti tärkeänä vain suhdetta 2 koordinaatiston välillä. Kun newtonilaisuudessa voidaan niputtaa vaikka 10 koordinaatistoa, niin suhteellisuudessa nippuun kuuluu vain 2. Kun suhteellisuudessa yhdistetään 10 niin se menisi näin:
1<--->2
1<--->3
1<--->4 jne.

2<--->1
2<--->3
2<--->4 jne.

3<--->1
3<--->2
3<--->4 jne.

Näin olen käsittänyt, eli suhteellisuus on vaikeampi teoria.

Kun R1 ja R2 kelluvat, vain niiden välinen nopeusero on ratkaisevaa, ei se kumpi on kiihdyttänyt suuremman nopeuden (kiihdytyksillä on kyllä muuten merkitystä mutta se on eri asia). Kun R1 ja R2 menevät avaruuden ulapalla, luonnonlait määrää niiden suhteen toisiinsa, ei esim maan näkökulma. Eli suhteet rakentuu vain luonnonlakien pohjalta.
Otsikko: Vs: Epäselvyys suhteellisuusteorian aika ja etäisyys käsityksissä
Kirjoitti: velihopea - 07.09.2023, 23:28:46
Kiitos taas Eusa ja mistral kommenteistanne. Harmin paikka, että minä tulin edes Eusalle kommentoineeksi edellisessä varmemmaksi-vakuudeksi -postauksessani rakettien R1 ja R2 kellonaikoja. Ok, minun tarvitsee pohtia esittämiänne näkökohtia erivauhtisten rakettien kellojen näkymistä toisilleen.

Mutta originaalissa ajatuskoe -postauksessani 05-09-2023 en edes maininnut R1 ja R2 kellojen aikoja enkä ottanut kantaa miten ne näkyvät. Ajatuskokeen pointti oli, että kaikkien kellot oli käynnistetty samaan aikaan, että R1 ja R2 kulkivat eri nopeutta, että R1-R2 ohitushetkellä oli maasta saatu viimeinen radioviesti mitä maan kello olikaan näyttänyt sen viestin lähetyshetkellä. Koska molemmat raketit vastaanottivat saman viestin samassa paikassa, ikäänkuin "näkivät" mitä maan kello oli joskus ollut, maan kellon olisi pitänyt olla jätättäneenä jo kyseisen viestin lähetyshetkellä. Tuo yksi kellonaika, oli se mikä tahansa, ei voi olla sopivasti jätättäneenä R1:lle ja toisella lailla sopivasti jätättäneenä R2:lle.

Tuo on todisteluni ja se ristiriita, josta tein johtopäätökseni. Onneksi kumpikaan teistä ei kiistänyt sitä.
Otsikko: Vs: Epäselvyys suhteellisuusteorian aika ja etäisyys käsityksissä
Kirjoitti: mistral - 08.09.2023, 11:20:56
Lainaus käyttäjältä: velihopea - 07.09.2023, 23:28:46


Mutta originaalissa ajatuskoe -postauksessani 05-09-2023 en edes maininnut R1 ja R2 kellojen aikoja enkä ottanut kantaa miten ne näkyvät. Ajatuskokeen pointti oli, että kaikkien kellot oli käynnistetty samaan aikaan, että R1 ja R2 kulkivat eri nopeutta, että R1-R2 ohitushetkellä oli maasta saatu viimeinen radioviesti mitä maan kello olikaan näyttänyt sen viestin lähetyshetkellä. Koska molemmat raketit vastaanottivat saman viestin samassa paikassa, ikäänkuin "näkivät" mitä maan kello oli joskus ollut, maan kellon olisi pitänyt olla jätättäneenä jo kyseisen viestin lähetyshetkellä. Tuo yksi kellonaika, oli se mikä tahansa, ei voi olla sopivasti jätättäneenä R1:lle ja toisella lailla sopivasti jätättäneenä R2:lle.

Tuo on todisteluni ja se ristiriita, josta tein johtopäätökseni. Onneksi kumpikaan teistä ei kiistänyt sitä.

Hyvä koejärjestely. Kun signaaliviiveet jätetään laskuista pois, R1 ja R2 näkevät samalla hetkellä samassa paikassa maan kellossa eri lukemat. Ja vaikka koejärjestely tehtäisiin eri lailla, kohti maata, niin R1 ja R2 näkisivät rinnakkain ollessaan maan kellossa eri lukemat. Tämä on juuri se ristiriita.
Otsikko: Vs: Epäselvyys suhteellisuusteorian aika ja etäisyys käsityksissä
Kirjoitti: mistral - 08.09.2023, 18:32:54
Jos kerran koordinaatisto K ja K' kumoaa toistensa dilaation niin....tulee villi ajatus. Ajatus on ettei dilaatiota tässä merkityksessä ole ensinkään. Ja tästä seuraa että ei pituuskontraktiotakaan ole.
Tähän voin tarjota yhden argumentin joka tukee ettei kontraktio olisikaan todellinen:

Auringosta lähtee aalto maahan, meidän näkökulmasta kestää n. 8min mutta aallon näkökulmasta 0 sekuntia. Meidän kantilta aalto tekee valtavan monta aaltoliikettä jo näin lyhyellä matkalla, mutta omalta kantilta aalto ei tee yhtään aaltoliikettä. Tässä on selvä ristiriita joka ratkeaa jos kontraktiota ei olekaan.

No eihän tällaista vain yhdellä argumentilla todisteta mutta voidaan todella kysyä, onko suhteellisuuden pakko olla ristiriitaista.
Otsikko: Vs: Epäselvyys suhteellisuusteorian aika ja etäisyys käsityksissä
Kirjoitti: velihopea - 09.09.2023, 12:27:16
Lainaus käyttäjältä: mistral - 08.09.2023, 11:20:56
Hyvä koejärjestely. Kun signaaliviiveet jätetään laskuista pois, R1 ja R2 näkevät samalla hetkellä samassa paikassa maan kellossa eri lukemat. ...

Kiitos mistral koejärjestely-kehuista. Mutta en ymmärrä, miksi et ymmärrä, että koe todisti, että raketit R1 ja R2 eivät voi "nähdä" samalla hetkellä maan kellosta eri lukemat.

Ensinnäkin tuollainen "näkeminen" ilman signaaliviivettä on mahdototonta minkään matkan päähän. Asia pitää päätellä muulla tavalla. ST-liturgia (anteeksi termi), johon yhä vetoat ja kai enemmistö lukijoista uskoo, nyt sanoo että R1 ja R2 "näkevät" maasta eri kellon lukeman. Koska kokeessa kaikki kellot käynnistettiin samaan aikaan, olisi maan kellon pitänyt hidastua jo alusta pitäen ja vielä eri lailla R1 ja R2 suhteen. Tämä koe on tehtävissä fysiikan lakien puitteissa eikä edellytä mahdotonta äärettömän nopeaa "näkemistä". Koe osoitti, että erilailla hidastumista ei tapahtunut. Tulos romuttaa ST-liturgian symmetrisen kellojen hidastuminen -ajatusmallin.

Tämä keskustelu on saanut mukavasti lukijoita, viime aikoina 50-150+ lukukertaa/pv. Olisi mielenkiintoista kuulla muidenkin mielipide siitä, että onko esittämäni koejärjestely ja sen tulos vakuuttava.
Otsikko: Vs: Epäselvyys suhteellisuusteorian aika ja etäisyys käsityksissä
Kirjoitti: mistral - 09.09.2023, 15:14:58
Lainaus käyttäjältä: velihopea - 09.09.2023, 12:27:16
Kiitos mistral koejärjestely-kehuista. Mutta en ymmärrä, miksi et ymmärrä, että koe todisti, että raketit R1 ja R2 eivät voi "nähdä" samalla hetkellä maan kellosta eri lukemat.

Ymmärrän kyllä, 3 viestiä sitten sanoin "Tämä on juuri se ristiriita."

ES:ssä YS:ssä on iso ero, YS:n dilaatio on kumoutumatonta ES:n itsensä kumoavaa. Tarkoitan sitä dilaatiota josta nyt on ollut puhe, en viivedilaatiosta.

Otsikko: Vs: Epäselvyys suhteellisuusteorian aika ja etäisyys käsityksissä
Kirjoitti: Eusa - 09.09.2023, 20:01:05
Kun signaaliviiveet huomioidaan ja mallinnetaan liikkuvan kehyksen projektiiviset merkitykset tarkastelukoordinaatistoon, saadaan aikadilataatio ja pituuskontraktio. Liikkuvien kohteiden omat mitat ja oma ajanjuoksu ei tietenkään muutu, vain vuorovaikutusmerkitys muille. Perspektiivikuvausvertauksessa voidaan analogisoida kämmenen heilutusta: "...aivan kuten perspektiivikuvassa kaverin kämmen näyttää aina pienemmältä kuin oma kämmen. Aikadilataatio ja Lorentzin transformaatiot huomioivat tämän "kellon jätättämisen" - kuten valokuvaaminen huomioi perspektiivin."
Otsikko: Vs: Epäselvyys suhteellisuusteorian aika ja etäisyys käsityksissä
Kirjoitti: mistral - 09.09.2023, 21:37:25
Lainaus käyttäjältä: Eusa - 09.09.2023, 20:01:05
Kun signaaliviiveet huomioidaan ja mallinnetaan liikkuvan kehyksen projektiiviset merkitykset tarkastelukoordinaatistoon, saadaan aikadilataatio ja pituuskontraktio.

Näin ajattelen kanssa, eli kun viiveitä ei ole, dilaatio nollautuu mutta kun viive vaikuttaa, se joko vanhentaa tai nuorentaa lepokoordinaatistossa (tarkoittaa suhteessa liikekoordinaatistoon eli kiihdyttävään koordinaatistoon).

Mutta kontraktio taas on seurausta juuri nollautuvasta ajan hidastumisesta. Sen käsittää siitä että aina kun nopeus on relativistinen, kontraktio alkaa vaikuttaa, se ei vaikuta kontraktioon vanheneeko vai "nuoreneeko" signaaliviiveiden vuoksi. Kontraktio on visainen juttu koska käsittääkseni hiukkaskiihdyttimissä se huomioidaan mikä on vahva todiste kontraktion puolesta mutta kuten aiemmin sanottu, jos dilaatio nollautuu niin mistä kontraktio silloin saa perusteensa?
Otsikko: Vs: Epäselvyys suhteellisuusteorian aika ja etäisyys käsityksissä
Kirjoitti: Eusa - 10.09.2023, 06:21:51
Lainaus käyttäjältä: mistral - 09.09.2023, 21:37:25
Näin ajattelen kanssa, eli kun viiveitä ei ole, dilaatio nollautuu mutta kun viive vaikuttaa, se joko vanhentaa tai nuorentaa lepokoordinaatistossa (tarkoittaa suhteessa liikekoordinaatistoon eli kiihdyttävään koordinaatistoon).

Mutta kontraktio taas on seurausta juuri nollautuvasta ajan hidastumisesta. Sen käsittää siitä että aina kun nopeus on relativistinen, kontraktio alkaa vaikuttaa, se ei vaikuta kontraktioon vanheneeko vai "nuoreneeko" signaaliviiveiden vuoksi. Kontraktio on visainen juttu koska käsittääkseni hiukkaskiihdyttimissä se huomioidaan mikä on vahva todiste kontraktion puolesta mutta kuten aiemmin sanottu, jos dilaatio nollautuu niin mistä kontraktio silloin saa perusteensa?
Mikä esittämässäni kämmenenheilutusanalogiassani on vielä puutteellista, kun vaikuttaa, ettei se saa aikaan ahaa-elämyksiä?

Kun Matti vilkuttaa Tepolle ja Teppo Matille 50 m etäisyydellä toisistaan, ei silmän verkkokalvoon vaikuttava kämmenen pienentynyt koko ole katsottavan kohteen kannalta todellista, eikä kämmenen ympärillä näkökentän laajentuminenkaan, mutta havaitsijan kannalta ne ovat sitä todellisuutta millaisena kohteet projisoituvat hänelle.

Perspektivikuva-analogiaan on aika vaativaa lähteä rakentamaan analogiaa sille, miten kappaleet muuttaisivatkin kokoaan todellisesti jouduttuaan fysikaalisen muutoksen alaisiksi. Sellaista tylyä ideaa voisi soveltaa, että olkoon kartiomainen putki, jossa toinen pää on halkaisijaltaan 1 m ja toinen pää 5 cm. Olkoon tuo kartioputki rakennettu pystyyn 5 m korkeana. Ylhäältä pudotetaan halkaisijaltaan metrin muoto ja tilavuudeltaan 1 m³ vettä; toinen määrä kartion läpi ja toinen vierestä. Vierestä pudonnut määrä alta päin tarkasteltuna näyttää aluksi pienemmältä, mutta pudottuaan tulee samaan paikalliseen mittakaavaansa "itsensä" kokoisena. Sen sijaan kartioon pudotettu määrä pienenee leveydeltään ja ottaa lisää aikaa juostessaan alarei'ästä läpi. Vesi vuorovaikuttaa kolmannen osapuolen kanssa.

Esim. kaksosparadoksissa avaruusalus+kaksonen vuorovaikuttaa kolmannen osapuolen, polttoaineen&pakokaasujen kanssa, jotka eivät palaa, vaan jäävät matkalle mahdollistamaan kaksosen pysymisen nuorempana kuin Maassa odotteleva sisaruksensa.
Otsikko: Vs: Epäselvyys suhteellisuusteorian aika ja etäisyys käsityksissä
Kirjoitti: mistral - 10.09.2023, 19:02:18
Lainaus käyttäjältä: Eusa - 10.09.2023, 06:21:51
Mikä esittämässäni kämmenenheilutusanalogiassani on vielä puutteellista, kun vaikuttaa, ettei se saa aikaan ahaa-elämyksiä?

Kun Matti vilkuttaa Tepolle ja Teppo Matille 50 m etäisyydellä toisistaan, ei silmän verkkokalvoon vaikuttava kämmenen pienentynyt koko ole katsottavan kohteen kannalta todellista, eikä kämmenen ympärillä näkökentän laajentuminenkaan, mutta havaitsijan kannalta ne ovat sitä todellisuutta millaisena kohteet projisoituvat hänelle.

Yrität siis rinnastaa kämmenen koon kellon hidastumiseen, mitä kaukaisempi kämmen, sitä hitaampi kello. Ja että kämmen on todellisuudessa aina tietyn kokoinen = että kello todellisuudessa käy aina vakionopeudella. Kyllä toki mutta ongelma on alistuminen ristiriidalle: kello käy suurennuslasin takana hitaammin vaikkei käykään hitaammin.
Eli on kaksi totuutta
1. kello käy hitaammin ---> seurauksena kontraktio
2. kello ei käy hitaammin ---> kontraktio edelleen voimassa mutta ilman perusteita
Otsikko: Vs: Epäselvyys suhteellisuusteorian aika ja etäisyys käsityksissä
Kirjoitti: Eusa - 10.09.2023, 19:58:25
Lainaus käyttäjältä: mistral - 10.09.2023, 19:02:18
Yrität siis rinnastaa kämmenen koon kellon hidastumiseen, mitä kaukaisempi kämmen, sitä hitaampi kello. Ja että kämmen on todellisuudessa aina tietyn kokoinen = että kello todellisuudessa käy aina vakionopeudella. Kyllä toki mutta ongelma alistuminen ristiriidalle: kello käy suurennuslasin takana hitaammin vaikkei käykään hitaammin.
Eli on kaksi totuutta
1. kello käy hitaammin ---> seurauksena kontraktio
2. kello ei käy hitaammin ---> kontraktio edelleen voimassa mutta ilman perusteita
Kämmenen koko vastaa paremminkin pituuskontraktiota ja sen ympärillä vapaa näköala aikadilataatiota (aika laajenee eli sen juoksutiheys hidastuu). Nämähän ovat kytköksissä siten, että aika-avaruusintervalli eli tapahtumien välisen erillisyyden määrä sekä etäisyytenä että aikana on invariantti muuttumaton kaikille havaitsijoille.
Otsikko: Vs: Epäselvyys suhteellisuusteorian aika ja etäisyys käsityksissä
Kirjoitti: velihopea - 18.09.2023, 20:36:54
Päivää taas. Olen pitänyt opiskelutaukoa ja mietiskellyt miten tätä jähnäystä kannattaisi jatkaa, josko ollenkaan. Ensiksi kuitenkin kiitokset Eusalle ja mistralille, jotka urheasti ovat jaksaneet kommentoida viimeisiä tekstejänikin. Muut, ehkä 100+ aiheen seuraajaa, varmaan lukevat näitä juttuja samalla huvituksella kuin minäkin kuuntelen kansanradiota.

Eusa kyselit eikö selityksesi saanut (minussakin) Ahaa-elämystä (ja palaisin oikeaan käsitykseen). Valitettavasti en saanut ahaata. En ymmärtänyt käyttämiäsi konsepteja, esim. perspektiivi-analogiaa, ja kuinka se selittäisi ristiinnäkemisprobleemin signaaliviive huomioiden tai ei.

Mistral puolestaan ymmärsit esimerkkini, ja ymmärtääkseni pidit sitä pätevänä, mutta jätit asian "tässä on se ristiriita" -tasolle versus että olisit todennut, että ST-opetus on tässä kohden väärin. Alku-Einsteinilaisista teksteistäkään en löytänyt tukea tuollaiselle väärinopetukselle.

Mutta otan vielä yhden esimerkin. Siinä myös yritän tuoda esiin, minkä vuoksi keskusteluaiheellamme on juuri tämä otsikko.

Otetaan yksi raketti. Se kulkee 0.8c vauhtia. AE:n kaavan mukaan tuossa vauhdissa raketin kellon käynti hidastuu 0.6-osaan maan kelloon nähden (maassa 60 min, raketissa 36 min). Raketti laukaistaan joku päivä klo 12:00 Helsingin rautatieasemalta kohti Turkua. Raketti alkaa lentämään Hki-Tku väliä 1 km korkeudessa. Sieltä on hyvä näköyhteys maahan ja radioaallotkaan ei juuri tuo signaaliviivettä. Raketissa on maan oloissa kasvanut ja koulutettu rakettimies.

Heti lähdön jälkeen, kun raketin kelloon tuli lukema 12:36, kun raketti on taas kääntymässä Hki:ssä, rakettimies katsoo mitä näyttää aseman kello. Virheelliseksi väittämäni ST-opetuksen mukaan, sen pitäisi näyttää 0.6 x 36 min = 21,6 min, eli ~12:22. Kun minä olen 100-varma, että aseman kello näytti ~13:00.

Ja rakettimies kuuntelee aikansa kuluksi YLE Radio Suomea. Rakettimies ihmettelee miksi kanava lähettää joka 36 min välein aikamerkin! Mutta sitten hän muistaakin, että hän on 0.8c nopeassa liikkeessä joten raketin kello on hidastunut. Sinänsä rakettimiehellä ei ole mitään ongelmaa elää, syödä ja nukkua raketin kellon mukaisia päiviä, koska hänenkin elintoiminnot ovat samalla lailla hidastuneet. Se rakettimiehen unirytmiä kyllä häiritsee, että valoisat/pimeät ajat eivät satu ollenkaan yksiin raketin kellon tuntien kanssa.

Joten vielä tuosta ristiin näkemisestä: Jos raketista nähtäisiin maan kello hidastuneena, kai kaikki muukin maassa pitäisi olla slow motion: maan pyöriminen, täysikuut, vuodenajat, kaikki. Mahdotonta!

Yleensä ottaen kai todellisuuden mallin, kuten ST, pitäisi kuvata ainakin periaatteessa todennettavia asioita todellisuudesta. Tämä ristiin näkemisen malli on minusta täysin absurdi ja ei-todennettavissa, väärä tulkinta ST:n tuosta osasta.

Pituuskontraktio.

Hki-Tku välimatka on linnuntietä 150 km. ST:n mukaan 0.8c nopeudessa myös kuljettu matka typistyy 0.6-osaan, eli 90 km:iin. Mutta, kuitenkin, todistettavasti, kun rakettia on maasta tarkkailtu sen on nähty säntillisesti aina kääntyvän Hki rautatieaseman ja Tku rautatieaseman kohdalla. Missä on pituuskontraktio? En näe sitä missään!

Rakettimiehestä kyllä saattaa tuntua, että eihän tämä Hki-Tku ollut pitkäkään matka, kun se meni näin äkkiä. Ja vielä, jos rakettimies menee siihen "halpaan", että pitää raketin kellon lukemaa samana kuin aika, ja sitten hän soveltaa kaavaa nopeus (pitää olla 0.8c) = matka/aika. Tästä matematiikka jyrää, että kuljetun matkan on lyhennyttävä samassa suhteessa kun aikakin on pienentynyt, jotta ei mentäisi ylivalonnopeuksiin tms.

Mutta tosiasia on, että Hki-Tku väli ei ole typistynyt yhtään. Ja rakettimieskin ymmärtää asian kun hän muistaa, että raketin nopeus (0.8c) ja Hki-Tku etäisyys (150 km) ovat määritetyt maan koodinaatistossa. Ja kun rakettimies konvertoi kellonsa lukeman maan kellon lukemaksi kaava 0.8c = 150 km/konvertoitu_kellolukema on ihan ok. Ja mitään pituuskontraktiota ei ole. Jos joku sanoo, että Hki-Tku välimatka on typistynyt, selittäköön myös miten se typistyminen näkyy a) maasta ja b) raketista.

Rakettimies on homo sapiens. Se on eläilaji, joka on kehittynyt vuosimiljoonien kuluessa, ja luonnonvalinnan myötä sen elintoiminnot ovat sopeutuneet siihen miten maapallo mekanismina toimii. Aika-suhteessa tarkoitan lähinnä maan pyörimistä (päivä/yö vaihtelu), kuun liike (kuukausirytmi), maan kiertäminen aurinkoa (vuodenajat).

Entäpä jos tuo raketti kuskeineen olisikin jostain muualta kosmoksesta, joku cruisailija, joka jostain kumman syystä alkaa sahaamaan Hki-Tku väliä 1 km korkeudessa. Tämä U.F.O.-rakettimies on joku U.F.O.-sapiens, jotain U.F.O.-kansaa, joka on kehittynyt jollain U.F.O.-planeetalla ja sopeutunut elämään sen toimintamekanismin puitteissa. U.F.O.-sapienssit varmaan tuntevat aika, kello, matka käsitteet. Mutta tuskin he ovat normeeranneet sekuntia ja metriä samoin kuin me. U.F.O.-raketille maa ei ole vertailukohta, ei esim. tiedä nopeuseroaan maahan.

Miltä tilanne vaikuttaa U.F.O.-rakettimiehen kannalta? Ei paljon miltään. Seuraaaville havaitsemilleen asioille U.F.O.-rakettimies sanoo vain "aha": Hki-aseman kellon viisarit näyttävät liikkuvan, YLE Radio Suomesta kuuluu tasaisin väliajoin piipitystä, ja joka 24. piipityksen yhteydessä kirkonkellojen soittoa, ja maa näyttää pyörivän tehden valo/pimeä vaihtelua, jne. Ne eivät merkitse U.F.O.-rakettimiehen elämiselle mitään ongelmaa tai ihmetystä. Hän pitää U.F.O.-planeetalla oppimansa tavat. Hki-Tku välimatkaa hän ei ihmettele yhtään, mittaa sen vain niikuin on tottunut välimatkoja mittaamaan U.F.O.-planeetalla. Nopeuden ja gravitaation vaikutuksen kellon käyntinopeuteen hän tietää. Pituuskontraktiosta hän ei ole kuullutkaan.

Tämä esittämäni ajan käsitys (oikeastaan kukaan ei tiedä mitä aika on, mutta mitä kello näyttää liikkeen ja gravitaation vaikutuksen alaisuudessa) ja kuljetun matkan käsitys pelaa matemaattisesti ihan hyvin. Kellon käynnin muutos on todettu kokein. Pituuskontraktiota ei ole. Mikä estää etteikö ST-mallia muuteta näiltä osin yksinkertaisemmaksi, yhtä hyvin asioita selittäväksi ja ymmärrettävämmäksi.
Otsikko: Vs: Epäselvyys suhteellisuusteorian aika ja etäisyys käsityksissä
Kirjoitti: mistral - 19.09.2023, 16:29:30
Lainaus käyttäjältä: velihopea - 18.09.2023, 20:36:54

Pituuskontraktio.

Hki-Tku välimatka on linnuntietä 150 km. ST:n mukaan 0.8c nopeudessa myös kuljettu matka typistyy 0.6-osaan, eli 90 km:iin. Mutta, kuitenkin, todistettavasti, kun rakettia on maasta tarkkailtu sen on nähty säntillisesti aina kääntyvän Hki rautatieaseman ja Tku rautatieaseman kohdalla. Missä on pituuskontraktio? En näe sitä missään!

Pituuskontraktio on käsittääkseni "ennustus", teoreettiset fyysikot kahlaavat eri teoriota ja yrittävät löytää sieltä ennusteita, asioita joita voi testata ja hyväksyä tai hylätä. Pituuskontraktion ennuste on käsittääkseni juuri se ettei missään koordinaatistossa ylitetä c-nopeutta. Ylitys siis tapahtuisi jos aika hidastuu, näin 2 sekunnin pituisessa sekunnissa pääsisi 600 000km. Kumpi on primääri, aikadilaatio vai pituuskontraktio? Vastaus: dilaatio. Jotenka jos kontraktio halutaan kumota, silloin dilaatio pitää kumota.
Olen vähän sulatellut valokellon juttuja enkä ole varma miten se pitäisi ymmärtää, ongelmia on. Esim. kun peilit on raketin ruumassa, niin on selvää että ne on dilatoituneet (jos dilaatio on totta), mutta onko valoaalto niiden välissä dilatoitunut, on minulle arvoitus. Tämä on oleellinen asia, jos aalto ei ole dilatoitunut, tikitys ei hidastu ja se muuttaa kellon piirtämää siksakkia. Joka tapauksessa jos valokello "ennustaa" ajan hidastumisen, se pitää ottaa vakavasti. Ei voi lakaista maton alle.
Ja kun ei voi lakaista maton alle, siitä seuraa myös pituuskontraktion ennuste. Mutta ainahan voi yrittää ratkaista ristiindilaation mysteeriä koska yhtä totta on se että ristiindilaatio kumoaa itsensä. Onko kysymys tästä:
1. ristiindilaatio ja sen kumoutuminen on fyysinen asia?
2. ristiindilaatio ja sen kumoutuminen on pinnallinen taso ja sillä on myös syvempi, todellinen taso?
Otsikko: Vs: Epäselvyys suhteellisuusteorian aika ja etäisyys käsityksissä
Kirjoitti: Eusa - 19.09.2023, 20:13:01
Kaverien morjestaessa toisilleen kämmenellä silmiensä edessä tilanne kuvaa kämmenen ympärillä näkyvän alan ristiindilaatiota ja kämmenen koon ristiinkontraktiota täsmällisesti kuten Lorentz-muunnos aikaa ja pituutta.

Kun kaverit tulevat yhteen ja läimäisevät high five, dilaatiot ja kontraktiot nollautuvat. Samoin käy Lorentz-muunnoksessa, kun erilleen erkaantuvat jarruttavat ja asettuvat samaan yhteiseen nopeuteen.
Otsikko: Vs: Epäselvyys suhteellisuusteorian aika ja etäisyys käsityksissä
Kirjoitti: mistral - 19.09.2023, 23:03:51
Lainaus käyttäjältä: Eusa - 19.09.2023, 20:13:01
Kaverien morjestaessa toisilleen kämmenellä silmiensä edessä tilanne kuvaa kämmenen ympärillä näkyvän alan ristiindilaatiota ja kämmenen koon ristiinkontraktiota täsmällisesti kuten Lorentz-muunnos aikaa ja pituutta.

Kun kaverit tulevat yhteen ja läimäisevät high five, dilaatiot ja kontraktiot nollautuvat. Samoin käy Lorentz-muunnoksessa, kun erilleen erkaantuvat jarruttavat ja asettuvat samaan yhteiseen nopeuteen.

Ai käykö Lorenz-muunnoksessa niin? Ei kai muunnos sitä katso mihin suuntaan mennään vaan nopeutta ylipäänsä. Kaikki suunnat on saman arvoisia kun dilaatio syntyy. Tähän liittyy muuten yksi erikoisuus, kun toisen nopeus on poikittaissuuntainen, niin dilaatio syntyy mutta kun se tapahtuu kaukaisuudessa niin sielläkin dilaatio syntyy. Mutta erikoista on että jos omaa koordinaatistoa kääntää vaikka vain yhden kaarisekunnin ja näin saa kaukaisuudessa olevan "menemään tuplavauhtia", niin dilaatio ei kasvakaan! Joten jostain aika-avaruus vaistoaa pyörimisen nolla-arvon. Tämä on mysteeri.
Otsikko: Vs: Epäselvyys suhteellisuusteorian aika ja etäisyys käsityksissä
Kirjoitti: Eusa - 20.09.2023, 03:18:26
Lainaus käyttäjältä: mistral - 19.09.2023, 23:03:51
Ai käykö Lorenz-muunnoksessa niin? Ei kai muunnos sitä katso mihin suuntaan mennään vaan nopeutta ylipäänsä. Kaikki suunnat on saman arvoisia kun dilaatio syntyy. Tähän liittyy muuten yksi erikoisuus, kun toisen nopeus on poikittaissuuntainen, niin dilaatio syntyy mutta kun se tapahtuu kaukaisuudessa niin sielläkin dilaatio syntyy. Mutta erikoista on että jos omaa koordinaatistoa kääntää vaikka vain yhden kaarisekunnin ja näin saa kaukaisuudessa olevan "menemään tuplavauhtia", niin dilaatio ei kasvakaan! Joten jostain aika-avaruus vaistoaa pyörimisen nolla-arvon. Tämä on mysteeri.
Samoin kun morjestusanalogiassa ei ole väliä kengänkärkien suunnalla vaan vain katseen suunnalla, ei Lorentz-muunnoksessakaan ole merkitystä pyörimisellä vaan vain kappaleiden välisellä nopeudella.
Otsikko: Vs: Epäselvyys suhteellisuusteorian aika ja etäisyys käsityksissä
Kirjoitti: mistral - 21.09.2023, 11:30:33
Lainaus käyttäjältä: Eusa - 20.09.2023, 03:18:26
Samoin kun morjestusanalogiassa ei ole väliä kengänkärkien suunnalla vaan vain katseen suunnalla, ei Lorentz-muunnoksessakaan ole merkitystä pyörimisellä vaan vain kappaleiden välisellä nopeudella.

Ai että vain loitontuminen tai lähestyminen dilatoi?
Otsikko: Vs: Epäselvyys suhteellisuusteorian aika ja etäisyys käsityksissä
Kirjoitti: Eusa - 21.09.2023, 15:35:32
Lainaus käyttäjältä: mistral - 21.09.2023, 11:30:33
Ai että vain loitontuminen tai lähestyminen dilatoi?
Morjestamisvertauksen ja Lorentz-muunnoksen käsitevastaavuudet:
Loitontuminen - nopeus kasvaa
Lähestyminen - nopeus vähenee

Morjestuksessa on merkitystä keskinäisellä etäisyydellä, Lorentz-muunnoksessa keskinäisellä nopeudella. Morjestuksessa EI ole merkitystä nopeudella. Lorentz-muunnoksessa EI ole merkitystä etäisyydellä.

Luulin, että vertausanalogiat helpottaisivat vastaavan logiikan omaksumista...
Otsikko: Vs: Epäselvyys suhteellisuusteorian aika ja etäisyys käsityksissä
Kirjoitti: mistral - 21.09.2023, 19:15:44
Lainaus käyttäjältä: Eusa - 21.09.2023, 15:35:32
Morjestamisvertauksen ja Lorentz-muunnoksen käsitevastaavuudet:
Loitontuminen - nopeus kasvaa
Lähestyminen - nopeus vähenee

Morjestuksessa on merkitystä keskinäisellä etäisyydellä, Lorentz-muunnoksessa keskinäisellä nopeudella. Morjestuksessa EI ole merkitystä nopeudella. Lorentz-muunnoksessa EI ole merkitystä etäisyydellä.

Luulin, että vertausanalogiat helpottaisivat vastaavan logiikan omaksumista...

Eusa yrittää olla nokkela, eikö suoraan kysymykseen voi vastata suoraan? Eli kysymys:
Eikö sivusuuntainen liike tuota dilaatiota? Tarkennan, jos kuu kiertää maata sivusuuntaisesti, dilatoituuko sen aika meidän mielestä vai ei?
Otsikko: Vs: Epäselvyys suhteellisuusteorian aika ja etäisyys käsityksissä
Kirjoitti: Eusa - 21.09.2023, 23:31:44
Lainaus käyttäjältä: mistral - 21.09.2023, 19:15:44
Eusa yrittää olla nokkela, eikö suoraan kysymykseen voi vastata suoraan? Eli kysymys:
Eikö sivusuuntainen liike tuota dilaatiota? Tarkennan, jos kuu kiertää maata sivusuuntaisesti, dilatoituuko sen aika meidän mielestä vai ei?
Siis suppeassa suhteellisuudessa tarkastellaan vain yhdessä kiinnitetyssä koordinaatistossa toisen kappaleen suoraviivaista nopeutta ja aika & etäisyys projisoituvat sen perusteella.

Kiertoradat ja gravitaatio eivät selity kunnolla suppealla suhteellisuudella. Lorentz-muunnos koskee vain suppeaa suhteellisuutta. Yleisessä suhteellisuudessa vastaava projisoituminen on leivottu geometriaan sisäisenä ominaisuutena.
Otsikko: Vs: Epäselvyys suhteellisuusteorian aika ja etäisyys käsityksissä
Kirjoitti: mistral - 22.09.2023, 11:03:24
Lainaus käyttäjältä: mistral - 19.09.2023, 23:03:51
Mutta erikoista on että jos omaa koordinaatistoa kääntää vaikka vain yhden kaarisekunnin ja näin saa kaukaisuudessa olevan "menemään tuplavauhtia", niin dilaatio ei kasvakaan! Joten jostain aika-avaruus vaistoaa pyörimisen nolla-arvon. Tämä on mysteeri.

Eihän mysteeri nyt ratkennut? Yhtäkkiä mieleen juolahti Brian Greenen kuvaus "Kätketyt ulottuvuudet" kirjasta. Siinä hän esitti vesiämpärin pyöritystä tyhjässä universumissa jossa kuitenkin vaikuttaa maan painovoima. Jos ämpäri on puolillaan vettä ja sitä alkaa pyörittää, vesi alkaa nousta lähelle reunaa (parabelin muoto). Tämä on mystistä, kuinka vesi tietää nolla-pyörimisen arvon tyhjässä universumissa? Vesi kuitenkin "tietää" sen koska se asettuu vaateriin nolla-pyörimisen kohdalla!
Tässä on sovelluksen paikka, samoin koordinaatisto "tietää" nolla-pyörimisen arvon ja jos sen suhteen kaukaisuudessa jokin liikkuu poikittaissuunnassa, se dilatoi nolla-koordinaatiston suhteen ES:n merkityksessä? Toki signaaliviiveitä ei huomioida tässä tarkastelussa, vaan ainoastaan nopeuden tuottamaa dilaatiota. Siis on kyse periaatteesta.
Otsikko: Vs: Epäselvyys suhteellisuusteorian aika ja etäisyys käsityksissä
Kirjoitti: velihopea - 24.09.2023, 22:05:32
Lainaus käyttäjältä: mistral - 19.09.2023, 16:29:30
Pituuskontraktio on käsittääkseni "ennustus"

Mitä? Eikö pituuskontraktio ole nykyään virallisena käsityksenä olevan ST-mallin yksi osa, ilman mitään epäröintejä. Samalla lailla kuin sekin, että aika rinnastetaan oman kellon lukemaan. Muutakin kysymystä olisi mistralin kommentteihin, mutta tuollaisen "ennustus"-lähtökohdan päälle en ala niistä vänkäämään.

Toisin on Eusan kommenttien suhteen. Hän tuntuu olevan uskossaan vahva, joten mielessäni on pieni epäilys, joska minulta on jäänyt jotakin ymmärtämättä. Esimerkkiini (postaus #108), jossa raketti R1 saa hitaamman mutta etumatkaa saaneen raketin R2 kiinni, Eusa kommentoi 05-09-2023, 19:11:33

Lainaus käyttäjältä: EusaKun R1 saavuttaa R2:n, molemmat havaitsevat toistensa kellot hidastuneiksi omasta näkökulmastaan johtuen aikadilataatiosta. Tämä johtuu siitä, että eri nopeudella liikkuvissa kehyksissä kulkeva aika projisoituu tarkastelukehykseen aina hidastuneena aivan kuten perspektiivikuvassa kaverin kämmen näyttää aina pienemmältä kuin oma kämmen. Aikadilataatio ja Lorentzin transformaatiot huomioivat tämän "kellon jätättämisen" - kuten valokuvaaminen huomioi perspektiivin.

    Maan kello ei ole "valikoidusti hidastunut" R1:een tai R2:een nähden. Maapallon kellon aika on tarkastelureferenssi tässä skenaariossa, koska se ei liiku. R1:n ja R2:n kellot sen sijaan hidastuvat suhteessa Maan kellon aikaan. 

Kohta "Kun R1 saavuttaa R2:n, molemmat havaitsevat toistensa kellot hidastuneiksi omasta näkökulmastaan" ei mene jakeluuni. Kuinka tuollainen ristiin-näkeminen voi olla mahdollista? Raketit ovat ohituksessa hetken poski poskea vasten, vaikka 10 m etäisyydellä, ja rakettien seinässä voisi olla led-taulussa raketin kellon lukema. Se nyt ei vaan käy järkeeni, että molempien rakettien kuskit näkevät toisen raketin kellossa alemman luvun kuin omassa kellossa, oli Eusan selitys-yritys mikä tahansa.

Jos ST-malli ehdottaa tuollaista, minä heitän kyllä tuolla ST:n kohdalla vesilintua.

Tällä aiheella on ollut runsaasti lukijoita. Pyytäisin, uskaltaisiko jotkut muutkin teistä ottaa kantaa kumpi on oikeassa: Eusako (raketeista R1 ja R2 nähdään ohitushetkellä molempiin suuntiin ristiin toisen kellon jätättäneen), vai minun (että tuollainen näkeminen on mahdotonta, eli ST-mallia opetetaan väärin tuossa kohtaa).

Otsikko: Vs: Epäselvyys suhteellisuusteorian aika ja etäisyys käsityksissä
Kirjoitti: mistral - 25.09.2023, 00:04:10
Lainaus käyttäjältä: velihopea - 24.09.2023, 22:05:32
Mitä? Eikö pituuskontraktio ole nykyään virallisena käsityksenä olevan ST-mallin yksi osa, ilman mitään epäröintejä. Samalla lailla kuin sekin, että aika rinnastetaan oman kellon lukemaan. Muutakin kysymystä olisi mistralin kommentteihin, mutta tuollaisen "ennustus"-lähtökohdan päälle en ala niistä vänkäämään.

Tietysti pituuskontraktio on ES:n virallinen kanta, ei siitä ole kyse. Ennustuksella tarkoitin että kun rel. nopeudessa aika hidastuu, niin se on perustus jonka mukaan sitten "ennustetaan" mitä siitä seuraa. Ja seuraus/ennustus on että avaruus lyhenee liikesuunnassa (K:n mielestä K' lyhenee ja K':n mielestä K lyhenee). Myonin matka yläilmakehästä maan pinnalle on yksi todiste pituuskontraktiosta, se on mahdotonta jos kontraktiota ei ole.

Mutta itse kokonaisuudessa on ainakin 2 ristiriitaa jotka hämmentävät, 1. kuvaamasi rakettiongelma 2. valo ei tee omassa koordinaatistossaan yhtään aaltoliikettä mutta esim maasta katsottuna monta aaltoliikettä (olisko 500nm aallolla 2000 aaltoliikettä 1mm matkalla)
Otsikko: Vs: Epäselvyys suhteellisuusteorian aika ja etäisyys käsityksissä
Kirjoitti: Eusa - 25.09.2023, 05:28:18
Lorentz-muunnos ei ennusta mitään - se on vain perspektiivinen näkökulma eri nopeudella liikkuvaan.

Vasta kappaleelle tapahtuvista vuorovaikutuksista voidaan ennustaa jokin fysikaalinen muutos, liikkeen viitekehyksen vaihtuminen ja sen mukainen ikääntymisero.
Otsikko: Vs: Epäselvyys suhteellisuusteorian aika ja etäisyys käsityksissä
Kirjoitti: mistral - 25.09.2023, 22:22:07
Lainaus käyttäjältä: Eusa - 25.09.2023, 05:28:18
Lorentz-muunnos ei ennusta mitään - se on vain perspektiivinen näkökulma eri nopeudella liikkuvaan.

Vasta kappaleelle tapahtuvista vuorovaikutuksista voidaan ennustaa jokin fysikaalinen muutos, liikkeen viitekehyksen vaihtuminen ja sen mukainen ikääntymisero.

Ajattelen että K ja K' kumoavat toistensa dilaation mutta onko Lorenz-muunnos fysikaalinen vai ei-fysikaalinen vai kolmantena vaihtoehtona molempia. Voihan olla että se muuttuu fysikaaliseksi kun kumoaminen on alle 100%, siis jos esim 10% ei kumoudu niin sen verran muuttuu fysikaaliseksi. Toki kyse on aina fysiikasta mutta ainakaan minun järki tavoita 100% kumoutumisen mekanismia. Ärsyttäväähän se on kun asetelma ei ole monimutkainen.
Kun olen miettinyt valokelloa, niin en keksi kuinka se voisi välttää valon nopeuden ylittymistä muuten kuin hidastamalla aikaa. Jo kävelevä ihminen voi valokellolla piirtää yli c:n siksakkia (mutta koska c:tä ei voi ylittää, jalankulkijan aika on hitaampi). Jos vaikka sauvakävelysauvan sisään laittaa 2 peiliä ja valo kulkee siellä niin siksakki on olemassa, hypotenuusa on hiukan pidempi kuin kolmion pitkä sivu. Siis kaikki liikenteessä olijat "havaitsee" hypotenuusan (jos liikkuvat eri nopeudella). Vai osaako luonto huijata ihmistä niin ovelasti ettei ihminen vaan tajua jotain avainasiaa.
Otsikko: Vs: Epäselvyys suhteellisuusteorian aika ja etäisyys käsityksissä
Kirjoitti: Eusa - 25.09.2023, 23:58:09
Lainaus käyttäjältä: mistral - 25.09.2023, 22:22:07
Ajattelen että K ja K' kumoavat toistensa dilaation mutta onko Lorenz-muunnos fysikaalinen vai ei-fysikaalinen vai kolmantena vaihtoehtona molempia. Voihan olla että se muuttuu fysikaaliseksi kun kumoaminen on alle 100%, siis jos esim 10% ei kumoudu niin sen verran muuttuu fysikaaliseksi. Toki kyse on aina fysiikasta mutta ainakaan minun järki tavoita 100% kumoutumisen mekanismia. Ärsyttäväähän se on kun asetelma ei ole monimutkainen.
Kun olen miettinyt valokelloa, niin en keksi kuinka se voisi välttää valon nopeuden ylittymistä muuten kuin hidastamalla aikaa. Jo kävelevä ihminen voi valokellolla piirtää yli c:n siksakkia (mutta koska c:tä ei voi ylittää, jalankulkijan aika on hitaampi). Jos vaikka sauvakävelysauvan sisään laittaa 2 peiliä ja valo kulkee siellä niin siksakki on olemassa, hypotenuusa on hiukan pidempi kuin kolmion pitkä sivu. Siis kaikki liikenteessä olijat "havaitsee" hypotenuusan (jos liikkuvat eri nopeudella). Vai osaako luonto huijata ihmistä niin ovelasti ettei ihminen vaan tajua jotain avainasiaa.
Eiköhän kyse ole siitä onko ihminen perehtynyt logiikkaan ja sisäistänyt - siihen se ihmettely loppuu.
Otsikko: Vs: Epäselvyys suhteellisuusteorian aika ja etäisyys käsityksissä
Kirjoitti: mistral - 26.09.2023, 11:41:12
Lainaus käyttäjältä: Eusa - 25.09.2023, 23:58:09
Eiköhän kyse ole siitä onko ihminen perehtynyt logiikkaan ja sisäistänyt - siihen se ihmettely loppuu.

Mutta onko loogista käyttää perspektiivikuvausta? Nimittäin ei siinä ole ristiriitaa ensinkään, sehän kuvaa arkitodellisuutta. Tarvitaan aito ristiriita kuten kello 1 ja 2. 1 sanoo 2'lle, sinä käyt hitaammin kuin minä. Siihen 2, eikun sinähän se käyt hitaammin kuin minä.

Tässä on todellinen ristiriita, Eusa on hyvä ja keksii paremman kuvauksen.
Otsikko: Vs: Epäselvyys suhteellisuusteorian aika ja etäisyys käsityksissä
Kirjoitti: Eusa - 26.09.2023, 11:57:22
Lainaus käyttäjältä: mistral - 26.09.2023, 11:41:12
Mutta onko loogista käyttää perspektiivikuvausta? Nimittäin ei siinä ole ristiriitaa ensinkään, sehän kuvaa arkitodellisuutta. Tarvitaan aito ristiriita kuten kello 1 ja 2. 1 sanoo 2'lle, sinä käyt hitaammin kuin minä. Siihen 2, eikun sinähän se käyt hitaammin kuin minä.

Tässä on todellinen ristiriita, Eusa on hyvä ja keksii paremman kuvauksen.
Miksi haluat ristiriitaa asiaan, jossa sellaista ei ole? Haiskahtaa pseudotieteilyltä, valitettavasti.

Ethän intä ristiriidaksi sitä, että molemmat morjestajat ovat sitä mieltä, että oma kämmen täyttää kuva-alaa enemmän kuin kaverin käpälä; tai kuva-alan kämmenestä vapaa alue päinvastoin.

Lotentz-muunnos on samanlainen: molempien kaksosten havaintopäättelykentässä eri vauhtitilassa olevan nopeuden suuntaiset pituusmitat ovat pienemmät kuin omat lepomitat ja ajallisuus laajentunut. Kaverin kannalta päinvastoin, koska ei se oma paikallinen fysiikka muutu vaan vain sen projektiivinen vaikutus toiseen liiketilaan.
Otsikko: Vs: Epäselvyys suhteellisuusteorian aika ja etäisyys käsityksissä
Kirjoitti: velihopea - 26.09.2023, 16:22:02
Lainaus käyttäjältä: mistral - 25.09.2023, 00:04:10
Myonin matka yläilmakehästä maan pinnalle on yksi todiste pituuskontraktiosta, se on mahdotonta jos kontraktiota ei ole.

Tuo mahdottomuusväite on väärä. Tapaus selittyy mainiosti ilman pituuskontraktiota! Esimerkiksi Kari Enqvist kirjassa "Johdatus suhteellisuusteoriaan", ursa 2021, s. 42-43, tekee niin. Koska selitys on maukas, ja kun minäkin nyt luulen ymmärtäväni sen, kerronpa sen kaikille, lukijoiden huvitukseksi ja sivistykseksi. Samalla altistan sen kommenteille.

Maata pommittaa koko ajan n. 0.98c-vauhtisia hiukkasia. Kun sellainen törmää n. 10 km korkeudessa tihenevään ilmakehään, voi syntyä myoni. Jos sellaisen nappaisi vauhdista pinseteillä kiinni ja näyttäisi asiantuntijalle, hän sanoisi: kas, myonihan se siinä; se on epästabiili ja hajoaa 50% tod.näk 2.2x10^-6 s:ssa; mutta jos ei hajoa, niin se hajoaa 50% tod.näk seuraavan 2.2x10^-6 s aikana; mutta jos ei siinäkään, niin seuraavassa; jne.  2.2x10^-6 s:ssa myoni liikkuu 647 m. Koetilanteelle oli laskettu arvio, että mittausalalta ja mittausaikana ylhäällä syntyi 10 milj myonia. Kun tuo myonipopulaatio on kokenut 15,5 puoliintumista (x 647 m = n. 10 km), myoneista on vain muutama jäljellä, jotka selviävät maahan asti.

Mutta myonitunnistimissa oli n. 450.000 osumaa. Miten voi olla noin paljon? KE antaa selityksen pelkästään aikadilataatioon nojaten, seuraavasti:

Koska myonin nopeus on 0.98c, sen sisäinen kello on AE-kaavan mukaan hidastunut n. 1/5-osaan. Kellona toimii myonin kvanttifysikaalinen ominaisvärähtely, joka ohjaa milloin hajoamisen pitäisi keskimäärin tapahtua. Siis hieman sama kuvio kuin avaruusrakettiesimerkissä, jossa nopeus (ja gravitaatio) vaikuttavat mukana olevan matkustajan elintoimintojen vilkkauteen ja kellon käyntiin.

Koska myoneja ohjaavat kellot olivat hidastuneet 1/5-osaan, myonit pääsevätkin kulkemaan keskimäärin 5x647 m = 3.235 m ennen kun ensimmäinen populaation puoliintuminen tulee täyteen. Kaksi tuollaista puoliintumista lisää ja päästään n. 10 km:iin. Ja iso lauma myoneja pääsee maahan asti. Havaittujen myonien määrä vastasi odotettua.

Selitys ei tarvitse pituuskontraktiota! Mutta KE jatkaa toteamalla että ST:ssä aikadilataatio ja pituuskontraktio ovat kuin kolikon kaksi puolta.PituuskKontraktioon nojaten KE antaa selityksen, seuraavasti:

Myoni tuntee olonsa aivan normaaliksi (niinkuin edellisessäkin selityksessä). Myoni elelee kellonsa mukaisesti ja uskoo, että 2.2x10^-6 s:ssä matkaa tulee syötyä 647 m. Sitten myoni päättelee, tai muuten vaan huomaa, että 0.98c vauhdissa 10 km matka onkin lyhentynyt 1/5-osaan eli 2 km:iin. Joten sen onnekas myoni selvittää kolmella populaation puoliintumisella. Niinkuin koetulos näyttikin.

Huomio: aikadilataatio ja pituuskontraktio ovat matemaattisessa mielessä koplautuneet kolikon eri puoliksi. Vaikka matemaattisesti voidaan käyttää kumpaa tahansa kolikon puolta, se ei oikeuta päättelemään että molemmat tapahtumaskenaariot olisivat mahdollisia todellisuudessa. Aikadilataatio-skenaariota tapahtuu koko ajan. Vaikka jostain maahan asti selvinnestä myonista kuinka tuntuisi, että eihän se ilmakehä ollut kuin 2 km, tuntuminen ei auta, niin ei ole konkreettisesti ollut asia. Yksi naula lisää pituuskontraktion arkkuun?
Otsikko: Vs: Epäselvyys suhteellisuusteorian aika ja etäisyys käsityksissä
Kirjoitti: mistral - 26.09.2023, 20:06:24
Lainaus käyttäjältä: Eusa - 26.09.2023, 11:57:22
Miksi haluat ristiriitaa asiaan, jossa sellaista ei ole? Haiskahtaa pseudotieteilyltä, valitettavasti.

Ethän intä ristiriidaksi sitä, että molemmat morjestajat ovat sitä mieltä, että oma kämmen täyttää kuva-alaa enemmän kuin kaverin käpälä; tai kuva-alan kämmenestä vapaa alue päinvastoin.

Lotentz-muunnos on samanlainen: molempien kaksosten havaintopäättelykentässä eri vauhtitilassa olevan nopeuden suuntaiset pituusmitat ovat pienemmät kuin omat lepomitat ja ajallisuus laajentunut. Kaverin kannalta päinvastoin, koska ei se oma paikallinen fysiikka muutu vaan vain sen projektiivinen vaikutus toiseen liiketilaan.

Se on ristiriita jos raketti kuvaa Big Benin 12.30 ja kadulla olija kuvaa sen 13.30 samalla hetkellä.
Otsikko: Vs: Epäselvyys suhteellisuusteorian aika ja etäisyys käsityksissä
Kirjoitti: Eusa - 26.09.2023, 20:11:40
Lainaus käyttäjältä: velihopea - 26.09.2023, 16:22:02
Tuo mahdottomuusväite on väärä. Tapaus selittyy mainiosti ilman pituuskontraktiota! Esimerkiksi Kari Enqvist kirjassa "Johdatus suhteellisuusteoriaan", ursa 2021, s. 42-43, tekee niin. Koska selitys on maukas, ja kun minäkin nyt luulen ymmärtäväni sen, kerronpa sen kaikille, lukijoiden huvitukseksi ja sivistykseksi. Samalla altistan sen kommenteille.

Maata pommittaa koko ajan n. 0.98c-vauhtisia hiukkasia. Kun sellainen törmää n. 10 km korkeudessa tihenevään ilmakehään, voi syntyä myoni. Jos sellaisen nappaisi vauhdista pinseteillä kiinni ja näyttäisi asiantuntijalle, hän sanoisi: kas, myonihan se siinä; se on epästabiili ja hajoaa 50% tod.näk 2.2x10^-6 s:ssa; mutta jos ei hajoa, niin se hajoaa 50% tod.näk seuraavan 2.2x10^-6 s aikana; mutta jos ei siinäkään, niin seuraavassa; jne.  2.2x10^-6 s:ssa myoni liikkuu 647 m. Koetilanteelle oli laskettu arvio, että mittausalalta ja mittausaikana ylhäällä syntyi 10 milj myonia. Kun tuo myonipopulaatio on kokenut 15,5 puoliintumista (x 647 m = n. 10 km), myoneista on vain muutama jäljellä, jotka selviävät maahan asti.

Mutta myonitunnistimissa oli n. 450.000 osumaa. Miten voi olla noin paljon? KE antaa selityksen pelkästään aikadilataatioon nojaten, seuraavasti:

Koska myonin nopeus on 0.98c, sen sisäinen kello on AE-kaavan mukaan hidastunut n. 1/5-osaan. Kellona toimii myonin kvanttifysikaalinen ominaisvärähtely, joka ohjaa milloin hajoamisen pitäisi keskimäärin tapahtua. Siis hieman sama kuvio kuin avaruusrakettiesimerkissä, jossa nopeus (ja gravitaatio) vaikuttavat mukana olevan matkustajan elintoimintojen vilkkauteen ja kellon käyntiin.

Koska myoneja ohjaavat kellot olivat hidastuneet 1/5-osaan, myonit pääsevätkin kulkemaan keskimäärin 5x647 m = 3.235 m ennen kun ensimmäinen populaation puoliintuminen tulee täyteen. Kaksi tuollaista puoliintumista lisää ja päästään n. 10 km:iin. Ja iso lauma myoneja pääsee maahan asti. Havaittujen myonien määrä vastasi odotettua.

Selitys ei tarvitse pituuskontraktiota! Mutta KE jatkaa toteamalla että ST:ssä aikadilataatio ja pituuskontraktio ovat kuin kolikon kaksi puolta.PituuskKontraktioon nojaten KE antaa selityksen, seuraavasti:

Myoni tuntee olonsa aivan normaaliksi (niinkuin edellisessäkin selityksessä). Myoni elelee kellonsa mukaisesti ja uskoo, että 2.2x10^-6 s:ssä matkaa tulee syötyä 647 m. Sitten myoni päättelee, tai muuten vaan huomaa, että 0.98c vauhdissa 10 km matka onkin lyhentynyt 1/5-osaan eli 2 km:iin. Joten sen onnekas myoni selvittää kolmella populaation puoliintumisella. Niinkuin koetulos näyttikin.

Huomio: aikadilataatio ja pituuskontraktio ovat matemaattisessa mielessä koplautuneet kolikon eri puoliksi. Vaikka matemaattisesti voidaan käyttää kumpaa tahansa kolikon puolta, se ei oikeuta päättelemään että molemmat tapahtumaskenaariot olisivat mahdollisia todellisuudessa. Aikadilataatio-skenaariota tapahtuu koko ajan. Vaikka jostain maahan asti selvinnestä myonista kuinka tuntuisi, että eihän se ilmakehä ollut kuin 2 km, tuntuminen ei auta, niin ei ole konkreettisesti ollut asia. Yksi naula lisää pituuskontraktion arkkuun?
Pituus on vain lukuarvo, joka saadaan valovauhdin c perusteella. Koska nopealle myonille valovauhti on 1/5 valovauhdista, joka on Maalle isotropia, ovat Maan mitat Maan liikesuunnassa myonia vastaan litistyneet viidesosaansa. Mitään fysikaalista muutosta ei tapahdu, nopeus on vain huomioitava.
Otsikko: Vs: Epäselvyys suhteellisuusteorian aika ja etäisyys käsityksissä
Kirjoitti: mistral - 26.09.2023, 20:43:19
Lainaus käyttäjältä: velihopea - 26.09.2023, 16:22:02

Mutta myonitunnistimissa oli n. 450.000 osumaa. Miten voi olla noin paljon? KE antaa selityksen pelkästään aikadilataatioon nojaten, seuraavasti:

Koska myonin nopeus on 0.98c, sen sisäinen kello on AE-kaavan mukaan hidastunut n. 1/5-osaan. Kellona toimii myonin kvanttifysikaalinen ominaisvärähtely, joka ohjaa milloin hajoamisen pitäisi keskimäärin tapahtua. Siis hieman sama kuvio kuin avaruusrakettiesimerkissä, jossa nopeus (ja gravitaatio) vaikuttavat mukana olevan matkustajan elintoimintojen vilkkauteen ja kellon käyntiin.

Koska myoneja ohjaavat kellot olivat hidastuneet 1/5-osaan, myonit pääsevätkin kulkemaan keskimäärin 5x647 m = 3.235 m ennen kun ensimmäinen populaation puoliintuminen tulee täyteen. Kaksi tuollaista puoliintumista lisää ja päästään n. 10 km:iin. Ja iso lauma myoneja pääsee maahan asti. Havaittujen myonien määrä vastasi odotettua.

Selitys ei tarvitse pituuskontraktiota! Mutta KE jatkaa toteamalla että ST:ssä aikadilataatio ja pituuskontraktio ovat kuin kolikon kaksi puolta.PituuskKontraktioon nojaten KE antaa selityksen, seuraavasti:

Myoni tuntee olonsa aivan normaaliksi (niinkuin edellisessäkin selityksessä). Myoni elelee kellonsa mukaisesti ja uskoo, että 2.2x10^-6 s:ssä matkaa tulee syötyä 647 m. Sitten myoni päättelee, tai muuten vaan huomaa, että 0.98c vauhdissa 10 km matka onkin lyhentynyt 1/5-osaan eli 2 km:iin. Joten sen onnekas myoni selvittää kolmella populaation puoliintumisella. Niinkuin koetulos näyttikin

Jos ostaa dilaation, tulee kauppaan aina kontraktio mukaan. Ymmärtääkseni kontraktio menee näin:
100km/h ---> newtonilainen ympäristö...
100 000km/h ---> newtonilainen...
100 000km/s ---> newtonilainen sekä relativistinen...
295 000km/s ---> newtonilainen mutta hallitsevasti relativistinen..................................................................................

Raketissa nähdään kiihdytyksen aikana kuinka loppuvaiheessa ei niinkään vauhti muutu vaan vastaantulevien taivaankappaleiden litistyminen kertoo missä mennään. Eli vauhti tuntuu suunnilleen samalta mutta litteys kertoo muutoksesta.

Jos haluaa eroon kontraktiosta, pitäisi dilaatio jotenkin poistaa. Mutta miten tehdä tyhjäksi valokellon hypotenuusa?
Otsikko: Vs: Epäselvyys suhteellisuusteorian aika ja etäisyys käsityksissä
Kirjoitti: mistral - 28.09.2023, 10:41:22
Lainaus käyttäjältä: Eusa - 26.09.2023, 20:11:40
Pituus on vain lukuarvo, joka saadaan valovauhdin c perusteella. Koska nopealle myonille valovauhti on 1/5 valovauhdista, joka on Maalle isotropia, ovat Maan mitat Maan liikesuunnassa myonia vastaan litistyneet viidesosaansa. Mitään fysikaalista muutosta ei tapahdu, nopeus on vain huomioitava.

Mitäköhän tämä tarkoittaa "joka on Maalle isotropia"?

Otsikko: Vs: Epäselvyys suhteellisuusteorian aika ja etäisyys käsityksissä
Kirjoitti: Eusa - 28.09.2023, 12:34:27
Lainaus käyttäjältä: mistral - 28.09.2023, 10:41:22
Mitäköhän tämä tarkoittaa "joka on Maalle isotropia"?
Tarkoittaa sitä lähtökohtaa, että jokainen mittaaja mittaa itsensä ympäri pyörivät sädemitat samoina. Toisen kappaleen, joka liikkuu, pyörivä sädemitan voi mitata vaihtelevan sen mukaan onko sen pyörimisvaihe nopeuden suunnassa.

Kannattaa muistaa jäsentää kaksi selvästi eri asiaa:
- suora havainto, jossa on vain perille pääsyt projektiotieto
- mitattu tieto tai sitä ennustava teoria, jolloin tehdään välillisiä johtopäätöksiä valitun mittausmääritteen yleiseksi noudattamiseksi
Otsikko: Vs: Epäselvyys suhteellisuusteorian aika ja etäisyys käsityksissä
Kirjoitti: mistral - 28.09.2023, 12:44:29
Lainaus käyttäjältä: Eusa - 28.09.2023, 12:34:27
Tarkoittaa sitä lähtökohtaa, että jokainen mittaaja mittaa itsensä ympäri pyörivät sädemitat samoina. Toisen kappaleen, joka liikkuu, pyörivä sädemitan voi mitata vaihtelevan sen mukaan onko sen pyörimisvaihe nopeuden suunnassa.

Kannattaa muistaa jäsentää kaksi selvästi eri asiaa:
- suora havainto, jossa on vain perille pääsyt projektiotieto
- mitattu tieto tai sitä ennustava teoria, jolloin tehdään välillisiä johtopäätöksiä valitun mittausmääritteen yleiseksi noudattamiseksi

Ahaa, miksei voi sanoa selkokielellä, K:n mielestä K' on litistynyt ja K':n mielestä K on litistynyt?
Otsikko: Vs: Epäselvyys suhteellisuusteorian aika ja etäisyys käsityksissä
Kirjoitti: Eusa - 28.09.2023, 18:24:30
Lainaus käyttäjältä: mistral - 28.09.2023, 12:44:29
Ahaa, miksei voi sanoa selkokielellä, K:n mielestä K' on litistynyt ja K':n mielestä K on litistynyt?
Koska asialle on tuollainen tehokas matemaattinen termi "isotropia". Olisit voinut googlata.
Otsikko: Vs: Epäselvyys suhteellisuusteorian aika ja etäisyys käsityksissä
Kirjoitti: mistral - 28.09.2023, 19:23:53
Lainaus käyttäjältä: Eusa - 26.09.2023, 20:11:40
Pituus on vain lukuarvo, joka saadaan valovauhdin c perusteella. Koska nopealle myonille valovauhti on 1/5 valovauhdista, joka on Maalle isotropia, ovat Maan mitat Maan liikesuunnassa myonia vastaan litistyneet viidesosaansa. Mitään fysikaalista muutosta ei tapahdu, nopeus on vain huomioitava.

Ok otetaan uusiksi. Jos mitään fysikaalista muutosta ei tapahdu niin kun ajan kuluminen hidastuu 2x niin silloin myoni ylittää valon nopeuden. Siis kun sen sekunti vastaa 2 sekuntia niin se etenee 2 sekunnin matkan. Mikä on lähes 600 000km. Joten jos kontraktio halutaan mitätöidä, pitää se sievistää aritmeettisesti. Jotta se puolestaan onnistuisi, pitäisi myös dilaatio mitätöidä koska se on mitalin toinen puoli. Olepa hyvä ja mitöi dilaatio, ehkä se onnistuu.
Otsikko: Vs: Epäselvyys suhteellisuusteorian aika ja etäisyys käsityksissä
Kirjoitti: Riqis - 29.09.2023, 10:37:28
Tästä ketjusta on tullut ilmiantoja. Ylläpito ystävällisesti muistuttaa, että keskustelu foorumilla edellyttää ystävällistä tai neutraalia viestintää.

Käytännössä on tässä aihepiirissä ok epäillä tai väittää, että jokin voi mennä tai menee pseudotieteen puolelle, mutta mikään ylimääräinen vinoilu ei liene tarpeen. 
Otsikko: Vs: Epäselvyys suhteellisuusteorian aika ja etäisyys käsityksissä
Kirjoitti: Eusa - 29.09.2023, 12:25:21
Lainaus käyttäjältä: mistral - 26.09.2023, 20:06:24
Se on ristiriita jos raketti kuvaa Big Benin 12.30 ja kadulla olija kuvaa sen 13.30 samalla hetkellä.
Tuollainen "sama hetki" on erikoinen ehdotus. Raketin saama Big Benin 12:30 kertoo raketille myös kadulla vieressä tarkkailevan kellolle 12:30, jos hän on synkronoinut kellonsa Big Benin kanssa.

Suhteellisuus on sitä, että vain samassa liiketilassa keskenään olevat voivat jakaa yhteisiä "samoja hetkiä", jotka ovat siis ko kehyksen pelkästään avaruudellisia erillisyyksiä.

Kun raketilla ja Maalla on keskinäistä nopeutta, niillä ei ole yhteistä "samaa hetkeä" vaan samanaikaisuuden perusteet, pelkkä avaruudellinen erillisyys, ovat molemmille toisiaan tarkastellessa vastakkaisessa järjestyksessä. Sen sijaan kolmas liikekehys, joka kehittyy siten, että sen suhteen Maalla ja raketilla on vauhtia yhtä paljon, kokee Big Benin ja raketin kellon kulkevan tasatahtia.

Taisin viitata pseudotieteen vaaraan - kyse on sen tunnistamisesta, että vajavaisin tiedoin minulla on mahdollisuus erehtyä ontuviin tulkintoihin; tutustun saamaani vastaukseen huolellisesti ja selvitän asiaa opiskelemalla tarkemmin...
Otsikko: Vs: Epäselvyys suhteellisuusteorian aika ja etäisyys käsityksissä
Kirjoitti: mistral - 29.09.2023, 21:16:42
Lainaus käyttäjältä: Eusa - 29.09.2023, 12:25:21
Tuollainen "sama hetki" on erikoinen ehdotus. Raketin saama Big Benin 12:30 kertoo raketille myös kadulla vieressä tarkkailevan kellolle 12:30, jos hän on synkronoinut kellonsa Big Benin kanssa.

Suhteellisuus on sitä, että vain samassa liiketilassa keskenään olevat voivat jakaa yhteisiä "samoja hetkiä", jotka ovat siis ko kehyksen pelkästään avaruudellisia erillisyyksiä.

Kun raketilla ja Maalla on keskinäistä nopeutta, niillä ei ole yhteistä "samaa hetkeä" vaan samanaikaisuuden perusteet, pelkkä avaruudellinen erillisyys, ovat molemmille toisiaan tarkastellessa vastakkaisessa järjestyksessä. Sen sijaan kolmas liikekehys, joka kehittyy siten, että sen suhteen Maalla ja raketilla on vauhtia yhtä paljon, kokee Big Benin ja raketin kellon kulkevan tasatahtia.

Taisin viitata pseudotieteen vaaraan - kyse on sen tunnistamisesta, että vajavaisin tiedoin minulla on mahdollisuus erehtyä ontuviin tulkintoihin; tutustun saamaani vastaukseen huolellisesti ja selvitän asiaa opiskelemalla tarkemmin...

Aiemmin mainitsin että jo kävelyvauhdissa ilmenee valokellon hypotenuusa tosin sen pituus kolmion pitkään sivuun verrattuna  on 0,000.... pitempi, siis ero on pyöreästi nolla. Mutta suuremmissa nopeuksissa hypotenuusa alkaa erottua paremmin. Tämä on todellinen asia joten sitä ei voi hylätä teoreettisesti vaikka K ja K' kumoavatkin toistensa hypotenuusan.
Tänään joku lamppu syttyi päässä kun mietin kumoutumisen mekanismia. Mainitsin jo viestissä 124 siitä:

1. ristiindilaatio ja sen kumoutuminen on fyysinen asia?
2. ristiindilaatio ja sen kumoutuminen on pinnallinen taso ja sillä on myös syvempi, todellinen taso?


Päädyin 2:een. Idea menee näin: koska molemmat koordinaatistot on dilatoituneet, ne ei huomaa toisessa eroa. Siis K ei huomaa dilaatiota K':ssa ja päinvastoin.

1. kohdassa taas "lasketaan" ensin dilaatio ja sitten kumotaan se, huomataan että tehdään saman suuruinen "nosto ja heti perään saman suuruinen talletus pankkiin". Tulos on nolla, pankissa ei olisi tarvinnut ensinkään käydä.

2. kohdassa taas luonto ohittaa pankin ja kaikki on hyvin?

Näin maa <---> raketti koordinaatistot voisivat toimia jos jätetään huomioimatta viiveet. Viiveet toki lasketaan aina kun halutaan yksittäisestä tilanteesta oikea tulos mutta nyt käsittelen vain periaatetta.

Mutta heti alkajaisiksi jos mennään 2. kohdan mukaan, pitäisi testata toimiiko luonto näin. Ja luultavasti on testattukin ja jos hylky tulee niin tulee. Kuitenkin nyt sain puettua sanoiksi "syvemmän tason".
Otsikko: Vs: Epäselvyys suhteellisuusteorian aika ja etäisyys käsityksissä
Kirjoitti: Eusa - 30.09.2023, 02:09:09
Hyvä Mistral, kumoutumiset ja fysikaalisuutta syvemmät tasot ovat filosofiaa, jota kutsutaan metafysiikaksi.

Riittää, että otetaan kuva ja todetaan siitä perspektiivi.

Kamera ei muuta laajasti todellisuutta, jäsentää vain valonsäteet hyödylliseksi kuvaksi. Samoin Lorentz-muunnos jäsentää infon ajasta ja etäisyydestä hyödylliseksi kuvaksi, jonka perusteella on hyvä ennustaa.
Otsikko: Vs: Epäselvyys suhteellisuusteorian aika ja etäisyys käsityksissä
Kirjoitti: velihopea - 02.10.2023, 22:56:38
Olen koettanut ymmärtää Eusan eri yhteyksissä esittämiä selityksiä ST:n aika ja kuljettu matka asioihin. Että kyse on näkökulmaprojektiosta, mikä on perspektiivivalokuvauksen kaltainen asia.

Niissä kirjoissa mitä olen ST:stä lukenut, aikadilataatiota ja pituuskontraktiota ei ole selitetty valokuvauksen ja perspektiiviin avulla, tai niihin jotenkin verrannollisina. Vaikka kuinka olen niitä yrittänyt saada päähäni, sorry, en noita vertailuja ymmärrä. Mutta jos sinulla ja muillakin, jotka ovat mukisematta lukeneet kommenttisi, on tuo ymmärrys, niin hyvä.

Mutta oletan että sen ymmärryksen pohjalla on myös matematiikka.

Olen kiinnostunut mitä tuo matematiikka sanoo kellon lukemiksi eri liiketiloissa. Siispä palaan vanhaan esimerkkiini kahdesta raketista (postaus #108, 05-09-2023). Siinä ajatuskokeessa maasta lähtenyt 0.8c-vauhtinen raketti 1 sai 60 min:ssa 0.4c-vauhtisen raketin 2 kiinni, joka oli saanut 30 min etumatkan.

Kysyn mitä tuon käytetyn matematiikan mukaan, sillä hetkellä kun R1 tavoitti R2:n ja meni 10 m etäisyydeltä ohi:
1. R1:n oma kello näytti?
2. R1:n kuski näki, että R2:n kello näytti?
3. minkä kellolukeman R1 oli vastaanottanut viimeksi maasta radioviestillä?
4. R2:n oma kello näytti?
5. R2:n kuski näki, että R1:n kello näytti?
6. minkä kellolukeman R2 oli vastaanottanut viimeksi maasta radioviestillä?
Minuutin tarkkuus riittää vastauksessa.
Otsikko: Vs: Epäselvyys suhteellisuusteorian aika ja etäisyys käsityksissä
Kirjoitti: mistral - 03.10.2023, 22:59:16
Lainaus käyttäjältä: velihopea - 02.10.2023, 22:56:38
Olen koettanut ymmärtää Eusan eri yhteyksissä esittämiä selityksiä ST:n aika ja kuljettu matka asioihin. Että kyse on näkökulmaprojektiosta, mikä on perspektiivivalokuvauksen kaltainen asia.

Niissä kirjoissa mitä olen ST:stä lukenut, aikadilataatiota ja pituuskontraktiota ei ole selitetty valokuvauksen ja perspektiiviin avulla, tai niihin jotenkin verrannollisina. Vaikka kuinka olen niitä yrittänyt saada päähäni, sorry, en noita vertailuja ymmärrä. Mutta jos sinulla ja muillakin, jotka ovat mukisematta lukeneet kommenttisi, on tuo ymmärrys, niin hyvä.

Mutta oletan että sen ymmärryksen pohjalla on myös matematiikka.

Olen kiinnostunut mitä tuo matematiikka sanoo kellon lukemiksi eri liiketiloissa. Siispä palaan vanhaan esimerkkiini kahdesta raketista (postaus #108, 05-09-2023). Siinä ajatuskokeessa maasta lähtenyt 0.8c-vauhtinen raketti 1 sai 60 min:ssa 0.4c-vauhtisen raketin 2 kiinni, joka oli saanut 30 min etumatkan.

Kysyn mitä tuon käytetyn matematiikan mukaan, sillä hetkellä kun R1 tavoitti R2:n ja meni 10 m etäisyydeltä ohi:
1. R1:n oma kello näytti?
2. R1:n kuski näki, että R2:n kello näytti?
3. minkä kellolukeman R1 oli vastaanottanut viimeksi maasta radioviestillä?
4. R2:n oma kello näytti?
5. R2:n kuski näki, että R1:n kello näytti?
6. minkä kellolukeman R2 oli vastaanottanut viimeksi maasta radioviestillä?
Minuutin tarkkuus riittää vastauksessa.

Löysin Jyväskylän yliopiston Youtuben jossa on jotain kaavoja. Pituuskontraktion tube
https://www.youtube.com/watch?v=29LAzwUHd4I
ja sitten dilaation tube
https://www.youtube.com/watch?v=vSksu_J1IOY
(tämä tube seuraa heti kontraktiotuben perään)

Aikamoinen merkkien määrä, kontraktiotubessa on jo K, K', v, l, lnolla, deltat, deltat', delta tau, gamma mutta kun ne kirjaa A-4 paperille, alkaa pikkuhiljaa selviämään.

Yritän jossain vaiheessa itsekin laskea jotain kunhan ehdin.
Otsikko: Vs: Epäselvyys suhteellisuusteorian aika ja etäisyys käsityksissä
Kirjoitti: mistral - 04.10.2023, 11:25:12
Lainaus käyttäjältä: velihopea - 05.09.2023, 11:54:55
Tässä pyrin ajatuskokeella osoittamaan vääräksi väitteen, että kahdesta liiketilasta katsomalla ristiin näkisi aina sen toisen kellon jätättäneen, niinkuin esim. TA-lehden 5/2023 yleisökysymyksen vastauksessa esitetään.

Otetaan kaksi rakettia. Raketti 1 (R1) kulkee tasaista 0.8c vauhtia. Yhdessä tunnissa se kulkee ~864'' km. Raketti 2 (R2) kulkee tasaista 0.4c vauhtia. Tunnissa R2 kulkee puolet R1:n matkasta ~432'' km.

Valolta (tai radioviestiltä) kuluu 864'' km matkan taittamiseen 48 min ja puoleen väliin 24 min.

Maassa ja molemmissa raketeissa on tarkat kellot.

AE:n kaavan mukaan 0.8c vauhtinen R1:n kello hidastuu maan kelloon nähden ~0.6-osaan (maa 60 min, R1 ~36 min). Vastaavasti 0.4c vauhtinen R2:n kello hidastuu ~0.92 osaan (maa 60 min, R2 ~55 min).

Ensin lähtee R2. Se ajelee kaikessa rauhassa puoleen väliin 432'' km päähän ja pysäköi. Kun R2 on pysähtynyt, se on samassa liiketilassa kuin maa ja sen kello käy samaan tahtiin maan kellon kanssa. R2 lähettää maahan suuntaan viestin "start". R2 odottaa vielä paikallaan 24 min, ja kun maa-asema ja odottava R1 ovat juuri saaneet start-viestin, tapahtuu:
- R2 nollaa kellonsa ja säntää 0.4c vauhdilla eteenpäin
- R1 nollaa kellonsa ja säntää 0.8c vauhdilla R2:n perään
- maa-asema nollaa kellonsa ja alkaa lähettämään minuutin välein kellonaikaansa ja kuuntelee mitä raketit lähettävät
- raketit kuuntelevevat mitä maa-asema lähettää ja lähettävät maa-asemalle minuutin välein kellonsa ajan

On saatu järjestely, jossa voidaan "katsoa" liiketiloista toiseen mitä kello siellä on, informaation välitysaika huomioiden.

Kun (maan kellon mukaan) 60 min on kulunut R1 saavuttaa R2:n. Ohitushetkellä kuskit moikkaavat toisiaan ja kysyvät minkä kellolukeman sait maasta. Kysymys on siitä saivatko molemmat saman lukeman.

Nimittäin TA-lehden vastauksessa, ja muissakin yhteyksissä, esitetään myös valiidina tulkintamallina, että raketti on paikallaan ja maa liikkuu. Joten maan kello jätättää raketin kelloon nähden. Kuinka paljon? Tietysti AE:n kaavan mukaan. R1:n tapauksessa se tarkoittaa, että maan kello on hidastunut 0.6-osaan mitä R1:n kello käy. R2:n tapauksessa maan kello olisi hidastunut 0.92-osaan mitä R2:n kello käy.

Näin ei ollut. Maan kello ei ollut valikoidusti hidastunut erikseen R1:een nähden ja R2:een nähden. Yksi viesti ei voi valikoida arvoaan sen mukaan kuka sen vastaanottaa. Itse asiassa olikohan maan kello hidastunut ollenkaan?

Tästä päättelen, että esitetty tulkintamalli "aina toisen kello jätättää" ei voi olla oikea, koska se johtaa ristiriitaan.

Mitä jos yllä olevan viestin muuttaisi niin päin että startti on kaukaisuudessa ja R1 ja R2 kohtaavat maassa? Silloin maan kello näkyisi molemmille ja samanaikaisesti. Ja jos kellona olisi aurinkokello, sen piirtämä varjo olisi vakuuttava todiste kellonajasta. Oli kello mikä oli, R1:n ja R2:n näkemys maan dilaatiosta eroaa toisistaan ja sen perusteella aurinkokello näyttäisi eri aikaa mikä on ristiriitaista.
Otsikko: Vs: Epäselvyys suhteellisuusteorian aika ja etäisyys käsityksissä
Kirjoitti: velihopea - 04.10.2023, 20:17:48
Lainaus käyttäjältä: mistral - 03.10.2023, 22:59:16
....Yritän jossain vaiheessa itsekin laskea jotain kunhan ehdin.

Aloin jo kiittelemään ja elämään toivossa, että voisin saada edes joltakin selvän vastauksen...

Lainaus käyttäjältä: mistral - 04.10.2023, 11:25:12
Mitä jos yllä olevan viestin muuttaisi niin päin että startti on kaukaisuudessa ja R1 ja R2 kohtaavat maassa? Silloin maan kello näkyisi molemmille ja samanaikaisesti. Ja jos kellona olisi aurinkokello, sen piirtämä varjo olisi vakuuttava todiste kellonajasta. Oli kello mikä oli, R1:n ja R2:n näkemys maan dilaatiosta eroaa toisistaan ja sen perusteella aurinkokello näyttäisi eri aikaa mikä on ristiriitaista.

En osaa mieltää esimerkin (postaukseni #108, 05-09-2023) muuttamista tähän muotoon. Alkuperäinen esimerkki on minusta ajateltavissa oleva ja siinä kulminoituu ne asiat mistä olen kiinnostunut. Mistral, tai Eusa, tai kuka tahansa, onko mahdollista tehdä laskuharjoitus ja kertoa mitkä 6 minuuttilukemaa tuli tulokseksi.

Otsikko: Vs: Epäselvyys suhteellisuusteorian aika ja etäisyys käsityksissä
Kirjoitti: mistral - 04.10.2023, 21:37:48
Lainaus käyttäjältä: velihopea - 04.10.2023, 20:17:48
Aloin jo kiittelemään ja elämään toivossa, että voisin saada edes joltakin selvän vastauksen...

En osaa mieltää esimerkin (postaukseni #108, 05-09-2023) muuttamista tähän muotoon. Alkuperäinen esimerkki on minusta ajateltavissa oleva ja siinä kulminoituu ne asiat mistä olen kiinnostunut. Mistral, tai Eusa, tai kuka tahansa, onko mahdollista tehdä laskuharjoitus ja kertoa mitkä 6 minuuttilukemaa tuli tulokseksi.

Niin mikä oli R2:n starttietäisyys maasta?
Otsikko: Vs: Epäselvyys suhteellisuusteorian aika ja etäisyys käsityksissä
Kirjoitti: velihopea - 04.10.2023, 23:24:27
Lainaus käyttäjältä: mistral - 04.10.2023, 21:37:48
Niin mikä oli R2:n starttietäisyys maasta?

R2 lähti levosta 432 milj km etäisyydeltä maasta suuntana maasta suoraan poispäin. Se on matka jonka 0.4c-vauhtinen R2-raketti kulkee maa-koordinaatiston yhdessä tunnissa. 0.8c-vauhtinen R1 taas lähti maasta samaan suuntaan kuin R2 about samaan aikaan kuin R2 lähti.
Otsikko: Vs: Epäselvyys suhteellisuusteorian aika ja etäisyys käsityksissä
Kirjoitti: mistral - 06.10.2023, 00:53:38
Lainaus käyttäjältä: velihopea - 02.10.2023, 22:56:38

Kysyn mitä tuon käytetyn matematiikan mukaan, sillä hetkellä kun R1 tavoitti R2:n ja meni 10 m etäisyydeltä ohi:
1. R1:n oma kello näytti?
2. R1:n kuski näki, että R2:n kello näytti?
3. minkä kellolukeman R1 oli vastaanottanut viimeksi maasta radioviestillä?
4. R2:n oma kello näytti?
5. R2:n kuski näki, että R1:n kello näytti?
6. minkä kellolukeman R2 oli vastaanottanut viimeksi maasta radioviestillä?
Minuutin tarkkuus riittää vastauksessa.

Tässä väliaikatietoa, en saanut näitä niinkään Jyväskylän yliopiston kaavoista vaan operoin pelkän Lorentzin kertoimen kanssa.

1. R1 oma kello 36min
2. R1 näki R2:n kellon näyttävän 50,38min
3. tarkoitatko ylivalonnopeudella reaaliaikaista?
4. R2:n oma kello 55min
5. R2 näki että R1:n kello näytti 50,38min
6. sama kuin 3. kohdassa

Tässä ei oteta huomioon kiihdytyksiä, nopeudet on tasaisia.
Otsikko: Vs: Epäselvyys suhteellisuusteorian aika ja etäisyys käsityksissä
Kirjoitti: Eusa - 06.10.2023, 04:23:19
Vain kohtaamisessa rinnakkain voidaan kellot synkronoida. Mikäli siinä tilanteessa molemmat eivät ole samaa mieltä kellojensa näyttämistä, kyse ei ole tästä todellisuudesta.
Otsikko: Vs: Epäselvyys suhteellisuusteorian aika ja etäisyys käsityksissä
Kirjoitti: velihopea - 06.10.2023, 10:00:11
Kiitos mistral laskuharjoituksen tekemisestä.

Jotta voit tehdä sen loppuun, kohdilla
3. minkä kellolukeman R1 oli vastaanottanut viimeksi maasta radioviestillä? ja
6. minkä kellolukeman R2 oli vastaanottanut viimeksi maasta radioviestillä?

tarkoitan sitä kun maa-asema lähettää omaa kelloaikaansa radioviestillä koko ajan, sanokamme joka sekunnin välein mitä minuuttilukemaa maan kello näyttää. Molemmat raketit vastaanottavat niitä viestejä. Kysyn siis mikä on ohitushetkellä R1:n saaman ja R2:n saaman viimeisen maa-viestin lukema (joka on saavuttanut raketit normaalille radioviestin nopeudella eli valon nopeudella).

Ja kyllä, kiihdytyksiä ei oteta huomioon ja nopeudet on tasaisia.

Lainaus käyttäjältä: Eusa - 06.10.2023, 04:23:19
Vain kohtaamisessa rinnakkain voidaan kellot synkronoida. Mikäli siinä tilanteessa molemmat eivät ole samaa mieltä kellojensa näyttämistä, kyse ei ole tästä todellisuudesta.

En tiedä miten "kohtaaminen" yleisesti pitäisi määritellä. Tässä ajatuskokeessa pidän "kohtaamisena" sitä kun todellisuudessa R1 saa jossain kohtaa ja jollain hetkellä R2:n kiinni ja menee ohi. Pienen hetken R1 ja R2 ovat rinnakkain ihan vaikka 10 m näköetäisyydellä ja kuskit voivat nähdä ja kirjata mitä toisen raketin kello näytti ja mitä oma kello näytti. Ei tarvi pysähtyä palaveria pitämään ja mielipiteitä vaihtamaan.

Otsikko: Vs: Epäselvyys suhteellisuusteorian aika ja etäisyys käsityksissä
Kirjoitti: mistral - 06.10.2023, 15:10:38
Sain maan signaalin lasketuksi mutta on 95% varmuudella väärin :)

1. R1 oma kello 36min
2. R1 näki R2:n kellon näyttävän 50,38min
3. R1 sai maan signaaliksi 7,2min
4. R2:n oma kello 55min
5. R2 näki että R1:n kello näytti 50,38min
6. R2 sai maan signaaliksi 11min

Tämä oli tosi vaikea laskettava kun kaavatkaan ei ole hallussa.
Otsikko: Vs: Epäselvyys suhteellisuusteorian aika ja etäisyys käsityksissä
Kirjoitti: Eusa - 06.10.2023, 15:32:13
Lainaus käyttäjältä: velihopea - 04.10.2023, 23:24:27
R2 lähti levosta 432 milj km etäisyydeltä maasta suuntana maasta suoraan poispäin. Se on matka jonka 0.4c-vauhtinen R2-raketti kulkee maa-koordinaatiston yhdessä tunnissa. 0.8c-vauhtinen R1 taas lähti maasta samaan suuntaan kuin R2 about samaan aikaan kuin R2 lähti.
Eli lähtevät Maasta kiihdyttäen rinnakkain mutta R1 kiihdyttää kaksinkertaiseen nopeuteen?

Ei niiden kellojen vertailulla ole erityistä merkitystä sen jälkeen, kun ovat erillään ja erkaantuvat toidistaan Maan koordinaatistossa vauhdilla 0.4c.

Vasta, jos tapahtuu jarrutuksia tai kiihdytyksiä ja kohtaavat uudelleen, voidaan kohtaamistilanteen ikäero ennustaa.
Otsikko: Vs: Epäselvyys suhteellisuusteorian aika ja etäisyys käsityksissä
Kirjoitti: velihopea - 06.10.2023, 19:51:00
Ensiksi mistral, kiitos laskuharjoituksen loppuunsaattamisesta.

Mutta Eusa olet ymmärtänyt väärin koejärjestelyn lähtötilanteen. Se on kuvattu postauksessani #108, 05-09-2023. Ja tänään tein vielä järjestelyihin tarkennoksen: "maa-asema lähettää omaa kelloaikaansa radioviestillä koko ajan, sanokamme joka sekunnin välein mitä minuuttilukemaa maan kello näyttää." 

Eusa: "Eli lähtevät Maasta kiihdyttäen rinnakkain mutta R1 kiihdyttää kaksinkertaiseen nopeuteen?"

Ei noin. Vaan R2 oli ennen kokeen alkua itsekseen ajellut "puolimatkaan" eli 432'' km päähän ja pysähtynyt. Sieltä R2 lähettää radioviestin maahan (sen tulo maahan kestää 24 min), että lähden 24 min päästä 0.4c-vauhdilla, jolloin myös nollaan kelloni. Kun maa ja siellä oleva R1 saavat viestin, ne nollaavat kellonsa ja R1 lähtee R2:n suuntaan 0.8c-vauhdilla ja maa alkaa radioviestittämään kelloaikaansa.

Eusa: "Ei niiden kellojen vertailulla ole erityistä merkitystä sen jälkeen, kun ovat erillään ja erkaantuvat toidistaan Maan koordinaatistossa vauhdilla 0.4c."

Tietenkin. (Tarkennoksena: R1 pakenee maata 0.8c-vauhdilla ja R2 pakenee maata 0.4c-vahdilla)

Eusa: "Vasta, jos tapahtuu jarrutuksia tai kiihdytyksiä ja kohtaavat uudelleen, voidaan kohtaamistilanteen ikäero ennustaa."

Tietenkin, jos lähtö olisi kuin luulit.
Otsikko: Vs: Epäselvyys suhteellisuusteorian aika ja etäisyys käsityksissä
Kirjoitti: Eusa - 07.10.2023, 00:21:41
Lainaus käyttäjältä: velihopea - 06.10.2023, 19:51:00
Ensiksi mistral, kiitos laskuharjoituksen loppuunsaattamisesta.

Mutta Eusa olet ymmärtänyt väärin koejärjestelyn lähtötilanteen. Se on kuvattu postauksessani #108, 05-09-2023. Ja tänään tein vielä järjestelyihin tarkennoksen: "maa-asema lähettää omaa kelloaikaansa radioviestillä koko ajan, sanokamme joka sekunnin välein mitä minuuttilukemaa maan kello näyttää." 

Eusa: "Eli lähtevät Maasta kiihdyttäen rinnakkain mutta R1 kiihdyttää kaksinkertaiseen nopeuteen?"

Ei noin. Vaan R2 oli ennen kokeen alkua itsekseen ajellut "puolimatkaan" eli 432'' km päähän ja pysähtynyt. Sieltä R2 lähettää radioviestin maahan (sen tulo maahan kestää 24 min), että lähden 24 min päästä 0.4c-vauhdilla, jolloin myös nollaan kelloni. Kun maa ja siellä oleva R1 saavat viestin, ne nollaavat kellonsa ja R1 lähtee R2:n suuntaan 0.8c-vauhdilla ja maa alkaa radioviestittämään kelloaikaansa.

Eusa: "Ei niiden kellojen vertailulla ole erityistä merkitystä sen jälkeen, kun ovat erillään ja erkaantuvat toidistaan Maan koordinaatistossa vauhdilla 0.4c."

Tietenkin. (Tarkennoksena: R1 pakenee maata 0.8c-vauhdilla ja R2 pakenee maata 0.4c-vahdilla)

Eusa: "Vasta, jos tapahtuu jarrutuksia tai kiihdytyksiä ja kohtaavat uudelleen, voidaan kohtaamistilanteen ikäero ennustaa."

Tietenkin, jos lähtö olisi kuin luulit.
Eli aluksi R1 ja R2 ovat 24 min etäisyydellä toisistaan. Sitten R1 ottaa sen R2:n kiinni. Laskelmiesi perusteella, kun alukset pysähtyvät päätepisteessä, R1:n kello näyttää  19 min vähemmän kuin R2:n kello (55-36).

R2:n 24 min odottelu tarkoittaa, että raketit lähtevät Maan samanaikaisuustasossa samanaikaisesti (Einstein-synkronointi). Samanpaikkaisuuteen niillä nopeuseron johdosta on tuo 19 min ero.

Käytännön kellosynkronointi tarkoittaisi, että Maasta olisi ensin lähetetty signaali vakioetäisyydessä odottelevalle R2:lle, joka välittömästi toimittaa vastaussignaalin.

R1:n ja R2:n välinen keskinäinen vauhti jomman kumman koordinaatistossa ei ole 0.4c vaan Lorentzin muunnoksen mukainen vauhti (n. 0.59c).
Otsikko: Vs: Epäselvyys suhteellisuusteorian aika ja etäisyys käsityksissä
Kirjoitti: mistral - 07.10.2023, 12:47:34
Lainaus käyttäjältä: Eusa - 07.10.2023, 00:21:41
R1:n ja R2:n välinen keskinäinen vauhti jomman kumman koordinaatistossa ei ole 0.4c vaan Lorentzin muunnoksen mukainen vauhti (n. 0.59c).

Hyvä huomio. Jos ottaa kärjistetyn esimerkin, R1 menee 0,96c ja R2 0,95c, niin niiden välinen nopeusero ei ole 0,01c vaan paljon suurempi. Mutta miten lasket sen?
Otsikko: Vs: Epäselvyys suhteellisuusteorian aika ja etäisyys käsityksissä
Kirjoitti: velihopea - 07.10.2023, 13:41:17
Lainaus käyttäjältä: Eusa - 07.10.2023, 00:21:41
Käytännön kellosynkronointi tarkoittaisi, että Maasta olisi ensin lähetetty signaali vakioetäisyydessä odottelevalle R2:lle, joka välittömästi toimittaa vastaussignaalin.

R1:n ja R2:n välinen keskinäinen vauhti jomman kumman koordinaatistossa ei ole 0.4c vaan Lorentzin muunnoksen mukainen vauhti (n. 0.59c).

Tässä ajatuskokeessa kellosynkronointi ei tapahdu noin. Vaan siten että R2 on ensin omia aikojaan ajellut tai se on hinattu 432'' km päähän maasta. R2 on siellä paikallaan, samassa liiketilassa kuin maa. Siinä tilassa R2:n kello tikittää yhtä pitkiä minuutteja kuin maankin kello.

Aloitteen tekijä on R2. Se lähettää maahan radioviestin "lähdetään" (jonka viestin kulku maahan kestää 24 min), ja odottelee vielä paikallaan R2-kellon mukaan 24 min. Tuon 24 min kuluttua, myös maa ja R1 heti kun ovat saanet viestin, kaikki nollaavat kellonsa ja R1 ja R2 aloittavat nopean lentonsa.

Eusa, olisiko koejärjestely nyt selvä, että voisit laskea minuuttiarvot

... sillä hetkellä kun R1 tavoitti R2:n ja meni 10 m etäisyydeltä ohi:
        1. mitä R1:n oma kello näytti?
        2. mitä R1:n kuski näki, että R2:n kello näytti R2:n kyljen kellonäytöstä?
        3. minkä kellolukeman R1 oli vastaanottanut viimeksi maasta radioviestillä?
        4. mitä R2:n oma kello näytti?
        5. mitä R2:n kuski näki, että R1:n kello näytti R1:n kyljen kellonäytöstä?
        6. minkä kellolukeman R2 oli vastaanottanut viimeksi maasta radioviestillä?
        Minuutin tarkkuus riittää vastauksessa.

Kunhan saan sinunkin vastauksen, lupaan antaa oman vastauksenikin, ja siitä ehkä selviää missä kohtaa ajattelen/lasken eri lailla -- tai väärin, jos niikseen on.

Tässä ajatuskokeessa en kysy R1 ja R2 keskinäistä nopeuseroa.
Otsikko: Vs: Epäselvyys suhteellisuusteorian aika ja etäisyys käsityksissä
Kirjoitti: Eusa - 07.10.2023, 16:20:08
Lainaus käyttäjältä: velihopea - 07.10.2023, 13:41:17
Tässä ajatuskokeessa kellosynkronointi ei tapahdu noin. Vaan siten että R2 on ensin omia aikojaan ajellut tai se on hinattu 432'' km päähän maasta. R2 on siellä paikallaan, samassa liiketilassa kuin maa. Siinä tilassa R2:n kello tikittää yhtä pitkiä minuutteja kuin maankin kello.
Ei sitä oikeaa etäisyyskohtaa yleisesti saa muuten selville kuin signaaleilla. Erityisesti toki voi mukana kuljetettavan kellon ja kiihtyvyysanturin avulla hinautua oikeaan kohtaan

Mutta ei noilla pohdinnoilla ole koeasetelmaan merkitystä. Todetaan Einstein-synkronointi ja sen mukaan tulokset.

Alusten keskinäisen aikadilataation laskemista ei tarvita, mutta jos haluaa sen selvittää, tulee käyttää niiden keskinäistä nopeutta 0,588...c. Laskukaava löytyy googlettelemalla "relativistinen nopeuksien yhteenlasku".
Otsikko: Vs: Epäselvyys suhteellisuusteorian aika ja etäisyys käsityksissä
Kirjoitti: mistral - 07.10.2023, 20:14:21
Onko tämä rel.nopeuksien yhteenlasku?

u = (v+w)/(1+vw)/c^2)

Löytyy täältä: https://users.aalto.fi/~thunebe1/courses/monjst.pdf

Laskin 0,4c + 0,4c

u = (0,4 + 0,4)/ (1 + 0,16)/1  =  0,689c

Sitten Eusan tarjoama 0,588c ................siis 0,4 + 0,588 = 0,8

u = (0,4 + 0,588) / (1 + 0,2352)/ 1 = 0,8c

Kyllä Eusa oli oikeassa. Nyt joudun taas korjaamaan laskujani.
Otsikko: Vs: Epäselvyys suhteellisuusteorian aika ja etäisyys käsityksissä
Kirjoitti: mistral - 07.10.2023, 21:06:11
Lainaus käyttäjältä: mistral - 06.10.2023, 15:10:38
Sain maan signaalin lasketuksi mutta on 95% varmuudella väärin :)

1. R1 oma kello 36min
2. R1 näki R2:n kellon näyttävän 50,38min
3. R1 sai maan signaaliksi 7,2min
4. R2:n oma kello 55min
5. R2 näki että R1:n kello näytti 50,38min
6. R2 sai maan signaaliksi 11min

Tämä oli tosi vaikea laskettava kun kaavatkaan ei ole hallussa.

Korjauslaskelma:

1. R1 oma kello 36min
2. R1 näki R2:n kellon näyttävän 33min
3. R1 sai maan signaaliksi 7,2min
4. R2:n oma kello 55min
5. R2 näki että R1:n kello näytti 33min
6. R2 sai maan signaaliksi 11min

Vielä sanon että on hapuilua :)
Otsikko: Vs: Epäselvyys suhteellisuusteorian aika ja etäisyys käsityksissä
Kirjoitti: Eusa - 08.10.2023, 04:11:36
Eikös Maan signaali kulu sen koordinaatistossa tunnin 0.8c vauhdilla matkanneelle 48 min? Tunnin päähän asti ehtinyt valo on läpäissyt tuon kohdan siis 12 min verran. Saman signaalin vastaanottaa samassa kohtauspaikassa tietysti myös R2. Miten tuollaisiin erilaisiin 7 tai 11 min tuloksiin voi päätyä?

10 m ero ei aiheuta mitään.

1. R1:n oma kello näytti 36 min
2. R1 näki R2:n kellon näyttävän 55 min
3. R1 sai maan signaalista 12 min
4. R2:n oma kello näytti 55 min
5. R2 näki että R1:n kello näytti 36 min
6. R2 sai maan signaalista 12 min

Jos ajatellaan, että Einstein-synkronointi olisi tehty jo taannoin ja Maa olisi lähettänyt signaalia aikaisemmin, R1-lähdön nollauksen suhteen, niin kun R2 lähti, Maan signaalista laskettu kellotieto sille oli -24 min. R2 kuitenkin kiihdytti.

Jotta yksinkertaistettua tulosta voi tulkita järkevästi, pitää mielestäni ajatella R1:n käyneen kiihdyttämässä ensin nopeuteen 0.8c ja synkronoi kellon ajoitetusti Maan ohituksessa. Silloin R1:n ja R2:n fysikaalinen ikäero löytyy helposti pelkästä R2:n kiihtymisjaksosta, nopeuskehyksen vaihtamisesta.

Nimittäin, suhteellisuuden perusteella Maan ja R2:n kellot olivat synkronoidut nollaan tasan lähtöhetkellä, mutta R2:n kiihdyttäessä poispäin Maasta, Maan ikääntyminen hidastui ja synkronointi hajosi.

Jos simuloitaisiin ajoitus, jossa R2 myös olisi ollut jo pitempään vauhdissa 0.4c ja ohittanut Maan ennen R1:stä, päättelisivät R1 ja R2 kohdatessaan, ettei heille ole muodostunut keskinäistä ikäeroa Maa-majakkaan jatkuvasti suhteuttaessa. Ikäero suppeassa suhteellisuudessa voi rakentua vain kiihdytysjaksoissa eli kehyksenvaihdoissa.
Otsikko: Vs: Epäselvyys suhteellisuusteorian aika ja etäisyys käsityksissä
Kirjoitti: velihopea - 08.10.2023, 11:09:08
Lainaus käyttäjältä: Eusa - 08.10.2023, 04:11:36
1. R1:n oma kello näytti 36 min
2. R1 näki R2:n kellon näyttävän 55 min
3. R1 sai maan signaalista 12 min
4. R2:n oma kello näytti 55 min
5. R2 näki että R1:n kello näytti 36 min
6. R2 sai maan signaalista 12 min

Kiitos Eusa. Minun vastauksessa on täsmälleen samat minuuttilukemat.

Huomio noista rakettien lähdöistä ja kiihdytyksistä: Tässä ajatuskokeessa olen ajatellut, että kiihdytykset tapahtuvat "silmänräpäyksessä" eli kiihdytysaikaista erilaista kellon käyntiä ei tarvitse ottaa laskuissa huomioon. R1 ja R2 lähtevät levosta eikä ole tarvetta tehdä "lentävä lähtö" -järjestelyjä.

Toinen huomio: gravitaatiotakaan ei oteta huomioon. Oikeastaan sen huomiotta jättäminen on vastoin todellisuutta. Sillä ajattelen, että kokeen kolme osallista kelloa on samakäyntisiä maan pinnalla. Mutta kun R2 on hinattu alkutilanteessa "puolimatkaan" eli jo käytännössä maan gravitaation ulkopuolelle (R2:n kello on nopeutunut) ja kun R1 pääsee lähtemään ja maan piiristä ulos (R1:nkin kelloon tulee nopeutus-lisä). Siis, vastoin todellisuutta, gravitaatio-vaikutusta ei ole laskettu noissa minuuttiajoissa.
Otsikko: Vs: Epäselvyys suhteellisuusteorian aika ja etäisyys käsityksissä
Kirjoitti: Eusa - 08.10.2023, 12:06:58
Tarkoitin gravitaation huomioimisella juurikin aluksen kiihdytystä eli antigravitaatiota, jota vastaan syttyy näennäinen vastasuuntainen liike, gravitaatio.

Jos yleinen suhteellisuus olisi alunperin ymmärretty oikein, olisi maanpinnan tuki eli kiihtyvyys ylöspäin nimetty gravitaatioksi ja vapaa putoaminen antigravitaatioksi, mutta nyt on tyydyttävä käsitteisiin niin päin kuin ne historiallisesti ovat jääneet/jäämässä käyttöön.
Otsikko: Vs: Epäselvyys suhteellisuusteorian aika ja etäisyys käsityksissä
Kirjoitti: Eusa - 08.10.2023, 12:32:27
Lainaus käyttäjältä: velihopea - 08.10.2023, 11:09:08
Kiitos Eusa. Minun vastauksessa on täsmälleen samat minuuttilukemat.

Huomio noista rakettien lähdöistä ja kiihdytyksistä: Tässä ajatuskokeessa olen ajatellut, että kiihdytykset tapahtuvat "silmänräpäyksessä" eli kiihdytysaikaista erilaista kellon käyntiä ei tarvitse ottaa laskuissa huomioon. R1 ja R2 lähtevät levosta eikä ole tarvetta tehdä "lentävä lähtö" -järjestelyjä.

Toinen huomio: gravitaatiotakaan ei oteta huomioon. Oikeastaan sen huomiotta jättäminen on vastoin todellisuutta. Sillä ajattelen, että kokeen kolme osallista kelloa on samakäyntisiä maan pinnalla. Mutta kun R2 on hinattu alkutilanteessa "puolimatkaan" eli jo käytännössä maan gravitaation ulkopuolelle (R2:n kello on nopeutunut) ja kun R1 pääsee lähtemään ja maan piiristä ulos (R1:nkin kelloon tulee nopeutus-lisä). Siis, vastoin todellisuutta, gravitaatio-vaikutusta ei ole laskettu noissa minuuttiajoissa.
Mikä oli ideasi tässä esimerkissä, jossa lopulta ei ole mitään merkitystä keskinäisellä "ristiindilaatiolla", josta kuitenkin keskusteltiin?

Tosin, ristiindilaatiollahan ei ole koskaan fysikaalista merkitystä - kuten ei kuvaperspektiivikään oikeasti aiheuta etäisyydessä pienemmältä vaikuttavien kohteiden todellista pienenemistä.

Kiihtyvyyden aiheuttamaa ikääntymiseroa voisi analogisoida ! :) heliumpallolla, joka suurena kohoaa taivaalle ja pienenee perspektiivisesti etääntyessään. Sen todellinen koko kuitenkin säilyy. Mutta sitten, kun palloon tulee reikä ja helium poistuu, pallo pienenee oikeasti ja putoaa lähettäjälleen, joka voi todeta pienenemisen.

Samoin kaksosen kiihdyttäessä takaisin se luovuttaa pois ajoainetta, kokee fysikaalisen muutoksen ja kuluttaa vähemmän aikaa suhteessa toiseen kaksoseen, joka voi asian todeta jälleennäkemisessä.
Otsikko: Vs: Epäselvyys suhteellisuusteorian aika ja etäisyys käsityksissä
Kirjoitti: mistral - 08.10.2023, 15:26:58
Lainaus käyttäjältä: Eusa - 08.10.2023, 04:11:36
Eikös Maan signaali kulu sen koordinaatistossa tunnin 0.8c vauhdilla matkanneelle 48 min? Tunnin päähän asti ehtinyt valo on läpäissyt tuon kohdan siis 12 min verran. Saman signaalin vastaanottaa samassa kohtauspaikassa tietysti myös R2. Miten tuollaisiin erilaisiin 7 tai 11 min tuloksiin voi päätyä?

Järkeilin näin:

........................... oma lukema........................kontr. välimatka..................................erotus.....................

R1 ..........................36min.............................28,8min c nopeudella.......................36min-28,8min=7,2min

R2..........................55min.............................44min c nopeudella...........................55min-44min=11min

Voi hyvinkin olla pielessä.
Otsikko: Vs: Epäselvyys suhteellisuusteorian aika ja etäisyys käsityksissä
Kirjoitti: Eusa - 08.10.2023, 17:42:25
Lainaus käyttäjältä: mistral - 08.10.2023, 15:26:58
Järkeilin näin:

........................... oma lukema........................kontr. välimatka..................................erotus.....................

R1 ..........................36min.............................28,8min c nopeudella.......................36min-28,8min=7,2min

R2..........................55min.............................44min c nopeudella...........................55min-44min=11min

Voi hyvinkin olla pielessä.
Joo. Kontraktio kuuluu mittaajan paikallisuuten - signaali tiedottaa kuitenkin Maan paikallisuudesta.
Otsikko: Vs: Epäselvyys suhteellisuusteorian aika ja etäisyys käsityksissä
Kirjoitti: mistral - 09.10.2023, 00:12:50
Olen ennenkin postannut Richard Mullerin kirjasta erikoisen asian Bellasta, Berkeley Lab Laser Accelerator, vain 9cm pitkä mutta kiihdyttää sillä matkalla elektronin 0,999 999 27 x valon nopeuteen.
Suunnataan Bella kohti 8,6 valovuoden päässä olevaa Siriusta. Bellaan tulevan elektronin lepokoordinaatistossa tämä todella on Siriuksen etäisyys. Muutama sekunnin miljardisosa myöhemmin liikkuvan elektronin gamma = 8317. Sen nopeus on 0,99999927 kertaa valon nopeus. Elektronin lepokoordinaatistossa Sirius on 8317  lähempänä, vain 0,001 valovuoden päässä. Siriuksen ja elektronin etäisyys  elektronin lepokoordinaatistossa mitattuna on pienentynyt lähes 8,6 valovuotta noin sekunnin miljardisosassa. Etäisyyden muutosnopeus on yli 8,6 miljardia kertaa valon nopeus.
Esimerkki osoittaa, että kiihtyvissä koordinaatistoissa mitatut etäisyydet voivat muuttua mielivaltaisen suurella nopeudella.....


Tämä juttu koskee R2:ta, kun se lähtee kulkemaan nollasta 0,4c nopeutta, sen takana oleva avaruus kontraktoituu. On kaksi vaihtoehtoa. Se kontraktoituu niinkuin Muller sanoi - välittömästi, tai sitten se kontraktoituu c-nopeudella. Jompi kumpi. No, on kolmaskin vaihtoehto, kontraktio ei ole todellinen asia. Mutta itse pitäisin c-nopeudella vaihtoehtoa mahdollisena. Silloin R2 mittaisi jotain 25% maan signaalista ei-kontraktoituneessa avaruudessa ja 75% kontraktoituneessa. Tämäkin sekoittaa "pakkaa".
Otsikko: Vs: Epäselvyys suhteellisuusteorian aika ja etäisyys käsityksissä
Kirjoitti: Eusa - 09.10.2023, 05:22:37
Mistral, turhaan mystifioit. Kun nostat katseen maisemaan, ei perspektiivi lähde mitenkään muodostumaan vaan on se projektio mitä valitset katsoa.

Samoin suhteellisuuden kontraktio on sen suhteen mihin vertaat, sen nopeuteen. On jatkuvasti valittavana loputon määrä kontraktioita, vertailukehyksiä.

Vuorovaikutuskiihtyvyys on sitten toinen asia. Se on absoluuttinen fysikaalinen muutos toisin kuin suhteellinen nopeusvalinta. Silloin se kappaleen osa, joka luovuttaa itsestään osan pois toisena kappaleena, määrittää kiihtymisen suuntansa mukaan uutta ajallista asemaansa muihin kappaleisiin nähden, itseisaikasuhdemuutostaan eli ikääntymiseroa eli maailmanviivan kaareutumistaan. Kiihtymisen etupuolella olevien ikäkehitys vilkastuu ja taakse jäävien hidastuu.
Otsikko: Vs: Epäselvyys suhteellisuusteorian aika ja etäisyys käsityksissä
Kirjoitti: velihopea - 09.10.2023, 13:14:28
Lainaus käyttäjältä: Eusa - 08.10.2023, 12:32:27
Mikä oli ideasi tässä esimerkissä, jossa lopulta ei ole mitään merkitystä keskinäisellä "ristiindilaatiolla", josta kuitenkin keskusteltiin?

Kiitos Eusa kun kysyit. Säästit minut aasinsillan keksimiseltä miten kai vielä haluan jatkaa tätä asiaa, mikä jatko tuntui joku päivä sitten ihan mahdottomalta.

Ensiksi määrittelen, mitä minä ymmärrän "ristiindilataatiolla"? TA-lehden 5/2023 lukijakysymyksen vastaksessa oli rakettiesimerkki. Siinä raketti lähti maasta. Henkilö A jäi maahan, B lähti raketin matkaan. Molemmilla oli kellot. Vastauksen antoi Kari Enqvist, jota pidän hyvin ST-asiat tuntevana. Seuraavia vastauksen kohtia pidän "ristiindilaationa": "A:n mielestä B:n kello jätättää" ja "B:n mielestä A:n kello jätättää" ja "itse asiassa kaikkien mielestä kaikkien muiden [vh jotka liikkuvat eri nopeudella kuin minä vh] kellot jätättävät".

Konstruoimassani esimerkissä on kolme liiketilaa: maa, R1 ja R2. Halusin järjestelyn, jossa kuskit R1:ssä ja R2:ssa näkevät ohitushetkellä omilla silmillään toisen liiketilan kellon ilman signaaliviivettä. Ts. poissuljen "minun mielestä" ajattelutavan. Maan kellosta R1 ja R2 saavat tietoa signaaliviiveen takaa, josta voimme päätellä miten maa-R1 ja maa-R2 liiketilapareissa maan kello mahdollisesti "on tuntunut jätättävän" a) suhteessa R1:een ja b) suhteessa R2:een.

Jos laskemamme minuuttilukemat ovat oikein, niinkuin uskon, esim. R1:n kuski tunnusti että hänen kellonsa oli jätättänyt R2:n kelloa. Ja R2:n kuski tunnusti, että hänen kellonsa oli edistänyt R1:n kelloa. ST ei opeta näin vaan, että aina sen toisen kello on jätättänyt. Siis tämä esimerkki falsifioi tuon ST-opetuksen!

Sana "epäselvyys" keskustelumme otsikossa on minusta erinomaisen paikallaan. Nimittäin vallitseva ST totuuskäsitys sanoo, että aikadilaatio on todellinen, ja siihen elimellisesti liittyvä pituuskontraktio myös. Siispä kuljetut matkat ovatkin oikeasti Lorentz-kaavalla lyhentyneitä! Hartaana ST-uskovaisena mistral ilmeisesti koetti saada pituuskontraktion mukaan kuvioon. Minäkin yritin laskea se huomioiden, mutta en osannut, ainakaan en saanut ollenkaan järkeenkäypää tulosta.

Olisipa lukijakunnasta osallistuttu laajemminkin tähän harjoitukseen. Olisi ollut mielenkiintoista nähdä millaisia tuloksia eri ihmiset olisivat saaneetkaan.
Otsikko: Vs: Epäselvyys suhteellisuusteorian aika ja etäisyys käsityksissä
Kirjoitti: Eusa - 09.10.2023, 13:24:24
Velihopea,

Ajatuskokeessasi ei esiinny mitenkään aikadilaatio R1:n ja R2:n kesken. Sen voi kumpi vain missä tahansa vaiheessa toki mallintaa ja se on "ristiindilaatio", koska projisoituu kuten analogiassani perspektiivit kahden morjestajan välillä.
Otsikko: Vs: Epäselvyys suhteellisuusteorian aika ja etäisyys käsityksissä
Kirjoitti: mistral - 10.10.2023, 00:56:49
Lainaus käyttäjältä: Eusa - 09.10.2023, 13:24:24
Velihopea,

Ajatuskokeessasi ei esiinny mitenkään aikadilaatio R1:n ja R2:n kesken. Sen voi kumpi vain missä tahansa vaiheessa toki mallintaa ja se on "ristiindilaatio", koska projisoituu kuten analogiassani perspektiivit kahden morjestajan välillä.

Eikö ristiindilaatio ole jo velihopean 6 kysymyksessä?

Tässä lainaus:
... sillä hetkellä kun R1 tavoitti R2:n ja meni 10 m etäisyydeltä ohi:
        1. mitä R1:n oma kello näytti?
        2. mitä R1:n kuski näki, että R2:n kello näytti R2:n kyljen kellonäytöstä?
        3. minkä kellolukeman R1 oli vastaanottanut viimeksi maasta radioviestillä?
        4. mitä R2:n oma kello näytti?
        5. mitä R2:n kuski näki, että R1:n kello näytti R1:n kyljen kellonäytöstä?
        6. minkä kellolukeman R2 oli vastaanottanut viimeksi maasta radioviestillä?
        Minuutin tarkkuus riittää vastauksessa.

Siis kodassa 2 ja 5 oli ristiindilaatio. Siis siinä molemmat näkee toisensa hitaampana. Se lukema on sama, on ihan teorian mukaista. R1:n ja R2:n välinen nopeusero ei ole 0,4c vaan 0,588c. Kun sillä laskee Lorentz-kertoimen, näkee mikä on oman kellon lukema, ja se on kertoimen verran suurempi.
Otsikko: Vs: Epäselvyys suhteellisuusteorian aika ja etäisyys käsityksissä
Kirjoitti: Eusa - 10.10.2023, 03:53:36
Lainaus käyttäjältä: mistral - 10.10.2023, 00:56:49
Eikö ristiindilaatio ole jo velihopean 6 kysymyksessä?

Tässä lainaus:
... sillä hetkellä kun R1 tavoitti R2:n ja meni 10 m etäisyydeltä ohi:
        1. mitä R1:n oma kello näytti?
        2. mitä R1:n kuski näki, että R2:n kello näytti R2:n kyljen kellonäytöstä?
        3. minkä kellolukeman R1 oli vastaanottanut viimeksi maasta radioviestillä?
        4. mitä R2:n oma kello näytti?
        5. mitä R2:n kuski näki, että R1:n kello näytti R1:n kyljen kellonäytöstä?
        6. minkä kellolukeman R2 oli vastaanottanut viimeksi maasta radioviestillä?
        Minuutin tarkkuus riittää vastauksessa.

Siis kodassa 2 ja 5 oli ristiindilaatio. Siis siinä molemmat näkee toisensa hitaampana. Se lukema on sama, on ihan teorian mukaista. R1:n ja R2:n välinen nopeusero ei ole 0,4c vaan 0,588c. Kun sillä laskee Lorentz-kertoimen, näkee mikä on oman kellon lukema, ja se on kertoimen verran suurempi.
Ei siinä kysytä toisen kellon käynnin ennustamisesta oman paikallisen fysiikan mittakaavaan vaan mitä kello 10 m päässä näyttää minuutin tarkkuudella.

Jos olisi kysytty havaitun valon Doppler-siirtymästä ohituksen yhteydessä, olisi päädytty siihen, että molempien kannalta samoin aluksi taajuus on korkeampi ja vaihtuu ohitettaessa matalaksi - kun suodatetaan pois signaalien Galilei-suhteellisuus olettaen valonnopeus vakioiseksi mittakaavamäärittäjäksi molemmille arvoon c eikä c+v, todetaan relativistinen ristiindilaatio.
Otsikko: Vs: Epäselvyys suhteellisuusteorian aika ja etäisyys käsityksissä
Kirjoitti: mistral - 16.10.2023, 21:15:14
Löysin erikoisen podcastin Yle areenasta:
https://areena.yle.fi/podcastit/1-4435769

Siinä millenium palkittu Tuomo Suntola avaa omia ajatuksiaan DU teoriasta. Hyvin mielenkiintoinen ja erikoinen teoria, sen pätevyydestä en tiedä mutta kovasti hän haluaa testata ennusteita mikä on hyvä asia.
Otsikko: Vs: Epäselvyys suhteellisuusteorian aika ja etäisyys käsityksissä
Kirjoitti: mistral - 22.11.2023, 22:14:54
Olen miettinyt gravitaation dilaatiota. Joku hiljainen hälytyskello on soittanut. Ongelma on intuitio, olen pitänyt itsestäänselvänä aaltojen pidentymisen tuottamaa taajuuden alenemista (sama kuin doppler-ilmiö kun paloauto menee ohi niin ääni madaltuu). Tämähän on niin helppoa siirtää gravitaatiopunasiirtymään että sama pätee siinäkin. Siis kun paloauton sykli  on auton lähestyessä 1,5 sek niin loitotessa se on 2sek. Mutta voiko sen siirtää?

Ehkä ei voi. Nimittäin kumpi ratkaisee dilaation, aallonpituus vai aaltofunktion romahdus pisteessä? Näissä on iso ero, aalto voi olla vaikka 1000km pitkä mutta sen romahdus on vain pieni piste. Kun ajatellaan neutronitähteä, sen pinnalta nousee aalto joka pitenee matkalla mutta valon nopeus (jos alkuoletuksena aika ei hidastukaan) pysyy samana, eli c-nopeus kuljettaa pisteen alhaalta ylös vakionopeudella. Siis jos pinnalta lähetetään puhetta radiolähettimellä niin pisteet eli fotonit säilyttävät toistensa välisen etäisyyden ylös asti. Otan esimerkin fotoneista:

fotoni no...........................etäisyys edelliseen fotoniin

1..............................................0mm
2..............................................2mm
3..............................................5mm
4..............................................1mm
5..............................................3mm

Tämä on mielivaltainen kuvaus, olennaista on vain että valon nopeus tuo tämän 5 ryppään samanlaisena alhaalta ylös. Vaikka aallonpituus muuttuisi niin nopeus ei muutu ja siksi rypäs vastaanotetaan samalla taajuudella kuin lähetettiinkin.
Myönnän että idea on raakile mutta voiko sen falsifioida?
Otsikko: Vs: Epäselvyys suhteellisuusteorian aika ja etäisyys käsityksissä
Kirjoitti: Eusa - 15.12.2023, 04:45:24
Lainaus käyttäjältä: mistral - 22.11.2023, 22:14:54Olen miettinyt gravitaation dilaatiota. Joku hiljainen hälytyskello on soittanut. Ongelma on intuitio, olen pitänyt itsestäänselvänä aaltojen pidentymisen tuottamaa taajuuden alenemista (sama kuin doppler-ilmiö kun paloauto menee ohi niin ääni madaltuu). Tämähän on niin helppoa siirtää gravitaatiopunasiirtymään että sama pätee siinäkin. Siis kun paloauton sykli  on auton lähestyessä 1,5 sek niin loitotessa se on 2sek. Mutta voiko sen siirtää?

Ehkä ei voi. Nimittäin kumpi ratkaisee dilaation, aallonpituus vai aaltofunktion romahdus pisteessä? Näissä on iso ero, aalto voi olla vaikka 1000km pitkä mutta sen romahdus on vain pieni piste. Kun ajatellaan neutronitähteä, sen pinnalta nousee aalto joka pitenee matkalla mutta valon nopeus (jos alkuoletuksena aika ei hidastukaan) pysyy samana, eli c-nopeus kuljettaa pisteen alhaalta ylös vakionopeudella. Siis jos pinnalta lähetetään puhetta radiolähettimellä niin pisteet eli fotonit säilyttävät toistensa välisen etäisyyden ylös asti. Otan esimerkin fotoneista:

fotoni no...........................etäisyys edelliseen fotoniin

1..............................................0mm
2..............................................2mm
3..............................................5mm
4..............................................1mm
5..............................................3mm

Tämä on mielivaltainen kuvaus, olennaista on vain että valon nopeus tuo tämän 5 ryppään samanlaisena alhaalta ylös. Vaikka aallonpituus muuttuisi niin nopeus ei muutu ja siksi rypäs vastaanotetaan samalla taajuudella kuin lähetettiinkin.
Myönnän että idea on raakile mutta voiko sen falsifioida?
Vaikuttaa siltä, ettet ole sisäistänyt kuinka vauhti c on vakioarvo, joka vaihtelee tarkastelukehyksen mukaan.

Jos mietitään millaiset nopeudet eri liikekehykset kokevat c:nä niin, että niille paikallinen isotropia tuottaa universaalit ominaisuudet ainerakenteelle, niin noilla nopeuksilla on eri vauhdit, jotka ovat pienempiä kuin c miettijän kehyksessä.

Jos miettijä vaihtaa nopeutensa samaksi jonkin kehyksen kanssa, silloin heillä on yhteisesti se suurin valovauhti c ja muilla skaalautuvat hitaammiksi.
Otsikko: Vs: Epäselvyys suhteellisuusteorian aika ja etäisyys käsityksissä
Kirjoitti: mistral - 15.12.2023, 09:05:01
Lainaus käyttäjältä: Eusa - 15.12.2023, 04:45:24Vaikuttaa siltä, ettet ole sisäistänyt kuinka vauhti c on vakioarvo, joka vaihtelee tarkastelukehyksen mukaan.

Jos mietitään millaiset nopeudet eri liikekehykset kokevat c:nä niin, että niille paikallinen isotropia tuottaa universaalit ominaisuudet ainerakenteelle, niin noilla nopeuksilla on eri vauhdit, jotka ovat pienempiä kuin c miettijän kehyksessä.

Jos miettijä vaihtaa nopeutensa samaksi jonkin kehyksen kanssa, silloin heillä on yhteisesti se suurin valovauhti c ja muilla skaalautuvat hitaammiksi.

Sori en maininnut yhtä tärkeää olettamusta: aika ei muuttuisi gravikaivossa. Tämä on teoreettinen olettamus. Olettamus lähtee siitä ettei dilaatiolle olisi mekanismia. Juuri kuvaus fotoniryppäästä olisi perusteena sille ettei tapahtumien taajuus muutu. Aallonpituus kyllä muuttuu muttei tapahtumien taajuus, tämä on intuitionvastainen tilanne. Intuitio sanoo että pidempi aalto ---> hitaampi taajuus ---> hitaampi aika vaaditaan alhaalla jotta ylä- ja alatason historia kulkisi rinnakkain. Nyt tässä intuitio romutetaan ja tarkastellaankin fotoneja, miten ne nousevat muuttumattomina ylös koska niiden on pakko noudattaa valon nopeutta.
Otsikko: Vs: Epäselvyys suhteellisuusteorian aika ja etäisyys käsityksissä
Kirjoitti: Eusa - 15.12.2023, 23:36:12
Lainaus käyttäjältä: mistral - 15.12.2023, 09:05:01Sori en maininnut yhtä tärkeää olettamusta: aika ei muuttuisi gravikaivossa. Tämä on teoreettinen olettamus. Olettamus lähtee siitä ettei dilaatiolle olisi mekanismia. Juuri kuvaus fotoniryppäästä olisi perusteena sille ettei tapahtumien taajuus muutu. Aallonpituus kyllä muuttuu muttei tapahtumien taajuus, tämä on intuitionvastainen tilanne. Intuitio sanoo että pidempi aalto ---> hitaampi taajuus ---> hitaampi aika vaaditaan alhaalla jotta ylä- ja alatason historia kulkisi rinnakkain. Nyt tässä intuitio romutetaan ja tarkastellaankin fotoneja, miten ne nousevat muuttumattomina ylös koska niiden on pakko noudattaa valon nopeutta.
Eikö ole todellisuudelle vierasta kehittää mittausten vastaisia "olettamuksia"?
Otsikko: Vs: Epäselvyys suhteellisuusteorian aika ja etäisyys käsityksissä
Kirjoitti: mistral - 16.12.2023, 10:27:06
Lainaus käyttäjältä: Eusa - 15.12.2023, 23:36:12Eikö ole todellisuudelle vierasta kehittää mittausten vastaisia "olettamuksia"?

On selvää että mittauksissa saadaan dilaatio gravitaatiokaivosta. Ongelma on falsifioida ryppään muoto
1..............................................0mm
2..............................................2mm
3..............................................5mm
4..............................................1mm
5..............................................3mm

Jos muoto säilyy samana alhaalta ylös, silloin kellon käynti ei ole muuttunut. Jos muoto muuttuu, silloin se falsifioituu.
Kun fotonirypäs lähtee, sen eri fotonien pitäisi poiketa eri määriä c-nopeudesta jotta muoto muuttuisi. Jos muoto säilyy ja rypäs kuljettaa vaikka puhetta, sen taajuuden on pakko säilyä samana. Kun puheen taajuus säilyy, eikö se todista että aikakin säilyy?
Otsikko: Vs: Epäselvyys suhteellisuusteorian aika ja etäisyys käsityksissä
Kirjoitti: Eusa - 17.12.2023, 08:20:46
Lainaus käyttäjältä: mistral - 16.12.2023, 10:27:06On selvää että mittauksissa saadaan dilaatio gravitaatiokaivosta. Ongelma on falsifioida ryppään muoto
1..............................................0mm
2..............................................2mm
3..............................................5mm
4..............................................1mm
5..............................................3mm

Jos muoto säilyy samana alhaalta ylös, silloin kellon käynti ei ole muuttunut. Jos muoto muuttuu, silloin se falsifioituu.
Kun fotonirypäs lähtee, sen eri fotonien pitäisi poiketa eri määriä c-nopeudesta jotta muoto muuttuisi. Jos muoto säilyy ja rypäs kuljettaa vaikka puhetta, sen taajuuden on pakko säilyä samana. Kun puheen taajuus säilyy, eikö se todista että aikakin säilyy?

Valovauhti c on jokaiselle havaitsijalle sovittu yhteinen lukuarvo, mutta havaitsijoiden kesken valovauhdin sisältämien etäisyyden ja ajan arvot poikkeavat toisistaan. Esimerkiksi saman emission ja absorption välin valon vauhti on toiselle 10c/10 ja toiselle 3c/3, missä luvut 10 ja 3 edustavat Lorentz-muuntuneiden etäisyyksien ja aikojen skaalaa.

Ryppääsi voi siis kokonaisuutena olla lyhentynyt tai pidentynyt ilman, että sillä on rajavauhdin c kanssa mitään tekemistä.
Otsikko: Vs: Epäselvyys suhteellisuusteorian aika ja etäisyys käsityksissä
Kirjoitti: mistral - 17.12.2023, 13:00:53
Tarkoitatko että ES:llä todistat YS:n? ES:n ja YS:n dilaatiomekanismit on on aivan eri maailmoista, ES:n on valokellon siksakkikuvion kolmion hypotenuusan pituus kun taas YS:n on käsittääkseni aallonpituuden muutos eli puna-/sinisiirtymä.
Otsikko: Vs: Epäselvyys suhteellisuusteorian aika ja etäisyys käsityksissä
Kirjoitti: Eusa - 17.12.2023, 17:21:36
Lainaus käyttäjältä: mistral - 17.12.2023, 13:00:53Tarkoitatko että ES:llä todistat YS:n? ES:n ja YS:n dilaatiomekanismit on on aivan eri maailmoista, ES:n on valokellon siksakkikuvion kolmion hypotenuusan pituus kun taas YS:n on käsittääkseni aallonpituuden muutos eli puna-/sinisiirtymä.
YS ei tuo Lorentzin muunnoksiin mitään uutta. Vaikka englanninkielessä on paljon YS:n kiihtyvyystilanteissa käytetty dilaatiosanaa, pitäisi käyttää ilmaisua "differential aging". Projektoitu aikadilaatio ja ikääntymisero ovat aivan eri asioita. Aikadilaatio liittyy koordinaatistovalintoihin ja ikääntymisero kappalevalinnan muuttumiseen ulkoisessa vuorovaikutuksessa eli läheisen massakeskittymän tuottamassa kelluvuudessa, nosteessa. Nosteen täsmällinen näennäisvastakohta on gravitaatio vapaana putoamisena.

Tässäkin ketjussa taisi olla lainausta Einsteinilta, missä kuvattiin kuinka inertiaalikoordinaatistojen vertailu on symmetristä, mutta koordinaatistokehyksen vaihtuessa epäinertiaalisesti toiseksi aiheutuu ikääntymiseroa.

Esimerkissäsi lähetetään signaalirypäs gravitaatiokaivosta ulos. Ryppääseen kohdistuu fysikaalinen noste ja se kokee inertiaalikoordinaatistoon (vapaasti putoava havaitsija) verrattuna punasiirtymää sitä enemmän kuta pitempään kulkee kiihtyvyyskentässä - valo ikään kuin kantaa inertiaalisuutta mukanaan ja emissiossa ikäännytään hitaammin (ryppääseen kohdistuu suurempi noste) kuin absorptiossa (pienempi noste/gravitaatio).
Otsikko: Vs: Epäselvyys suhteellisuusteorian aika ja etäisyys käsityksissä
Kirjoitti: mistral - 17.12.2023, 20:35:23
Viittaat vissiin entisen tiedefoorumin keskusteluun jossa käsiteltiin Princeton-papereita, olen edelleen samaa mieltä papereista kuin silloinkin.
Mutta kun esitin hämmentävän idean fotoniryppäästä, ajattelen että sen voisi falsifioida YS:n avulla jos falsifiointi ylipäätään onnistuu. Vielä dilaatiomekanismista, näen sen siis ongelmana joka ratkaistaan sovittamalla punasiirtymä omaan vapaan avaruuden koordinaatistoon tai päinvastoin sinisiirtymä omaan kaivokoordinaatistoon.
avaruuskoord. punasiirtymä viittaa kaivon aikaan ==> hitaampi kertoimella x
kaivokoord. sinisiirtymä viittaa avaruuden aikaan ==> nopeampi kertoimella x

 Tämä mahdollistaa rinnakkaiset historiat, siis historiathan ei saa vaeltaa toisistaan erilleen niin että kaivosta avaruuteen matkustettaisiin ajassa vinoa aikajanaa pitkin vaikkapa tuhat vuotta, siis janan pitää olla suora yhteys kaivon ja avaruuden välillä.
  Nyt kun suora yhteys on saatu, hinta siitä on siis dilaatio. On pakko hyväksyä ero ajan nopeudessa, eli aika on pantu muuttujaksi.
 
Siirryn ideaan, kuten sanoin aallonpituuden muutoksen ollaan ajateltu korreloivan ajan muutoksen kanssa mutta onko se vain intuition petos? Mitä jos dilaatiota ei tulekaan valoaallon vuorovaikutuksessa vaikka antennin kanssa eikä pitempi aallonpituus myöhästytä vuorovaikutusta? Eikö tämä olekin oikea kriteeri? Havainnollistus, kun tunnin ohjelma lähetetään kaivosta, fotonit tulee samalla taajuudella antenniin kuin ne lähetettiin koska c-nopeus pakottaa ne tulemaan. Tässä siis dilaatiota ei ole, ainoa viive on signaaliviive.
 
 
Otsikko: Vs: Epäselvyys suhteellisuusteorian aika ja etäisyys käsityksissä
Kirjoitti: Eusa - 18.12.2023, 00:26:41
Lainaus käyttäjältä: mistral - 17.12.2023, 20:35:23Viittaat vissiin entisen tiedefoorumin keskusteluun jossa käsiteltiin Princeton-papereita...
Näyttää tämän ketjun sivulla 3 Velihopea esitelleen Einsteinin papereita...
Otsikko: Vs: Epäselvyys suhteellisuusteorian aika ja etäisyys käsityksissä
Kirjoitti: mistral - 18.12.2023, 23:17:54
Lainaus käyttäjältä: Eusa - 17.12.2023, 17:21:36Esimerkissäsi lähetetään signaalirypäs gravitaatiokaivosta ulos. Ryppääseen kohdistuu fysikaalinen noste ja se kokee inertiaalikoordinaatistoon (vapaasti putoava havaitsija) verrattuna punasiirtymää sitä enemmän kuta pitempään kulkee kiihtyvyyskentässä - valo ikään kuin kantaa inertiaalisuutta mukanaan ja emissiossa ikäännytään hitaammin (ryppääseen kohdistuu suurempi noste) kuin absorptiossa (pienempi noste/gravitaatio).

Mitäköhän noste tarkoittaa? Ymmärrän niin että gravitaatio painaa aallon massaa alaspäin.Ja aalto reagoi, ei hidastamalla vauhtia, vaan "laihduttamalla", sen massa pienenee. Mitään ei saa ilmaiseksi kun mennään vasten gravitaatiota. Kun massa pienenee, se on aallonpituus kasvaa, niin tässä on se intuitiopetos: oletetaan intuisiivisesti että pidetynyt aalto vaatii enemmän aikaa. Itse olen elänyt tässä luulossa vissiin 15 vuotta mutta sitten tajusin ettei ajan kulu välttämättä muutukaan. Atomikello kyllä uskollisesti todistaa ajankulun muuttuvan mutta DU-teoria antaa sille eri syyn, olisiko ollut se että tyhjiön permittiivisyys muuttuu eri gravipotentiaaleilla.
Otsikko: Vs: Epäselvyys suhteellisuusteorian aika ja etäisyys käsityksissä
Kirjoitti: Eusa - 19.12.2023, 05:58:30
Noste on gravitaation vastakohta mutta oleellisempi, koska siinä tapahtuu varsinaiset paikallisen fysiikan paineiset vuorovaikutukset - gravitaatio on koordinaatistogeometrinen näennäisyys suhteessa nosteeseen.

Gravitaation kaareva kenttä kuitenkin antaa myös paikallisia vaikutuksiaan vuorovesivoimina ja ohjaa kappaleita törmäyksiin tai sidotuilla kiertoradoilla törmäyksen ohi. Saman voisi kuvata nosteen puolelta niin, että ainevuorovaikutukset pitävät kappaleita erillään sen minkä pystyvät.
Otsikko: Vs: Epäselvyys suhteellisuusteorian aika ja etäisyys käsityksissä
Kirjoitti: mistral - 19.12.2023, 12:05:12
Lainaus käyttäjältä: Eusa - 19.12.2023, 05:58:30Noste on gravitaation vastakohta

Onko sinulla viitettä nosteeseen? Nostehan on nesteessä mutta ei tyhjiössä. Voitko antaa linkin jollekin sivulle?
Otsikko: Vs: Epäselvyys suhteellisuusteorian aika ja etäisyys käsityksissä
Kirjoitti: Eusa - 21.12.2023, 22:19:03
Lainaus käyttäjältä: mistral - 19.12.2023, 12:05:12Onko sinulla viitettä nosteeseen? Nostehan on nesteessä mutta ei tyhjiössä. Voitko antaa linkin jollekin sivulle?
Tarkoitan sitä nostetta, joka näkyy yleisen suhteellisuusteorian matematiikassa. Esimerkiksi planeetta kiertoradalla varaa tilaa ja aikaa toisin kuin keskustähti - se on liikemääränsä ansiosta keskimääräisesti avaruudelliselta tiheydeltään kevyempi ja "kelluu" aine-energiana korkeammalla; esim. Maa on keskimääräiseltä ainetiheydeltään kuitenkin n. 4 kertaa Aurinkoa tiheämpi.

Kokonaistiheys aika-avaruudessa vallitsevan nosteen kannalta on yhdistelmä aikatiheyttä (aine) ja avaruustiheyttä (liike). Neliulotteisessa tiheystarkastelussa pallogeometrialla on erityisasema eikä samanlaista keskipistettä (ei siis kiertoratakeskiö, vaikka liike huomioidaankin) ole kuin normaalinosteen kanssa vaan tarkastellaan kappaleen ajallisuuslinjan eli maailmanviivan asemaa.
Otsikko: Vs: Epäselvyys suhteellisuusteorian aika ja etäisyys käsityksissä
Kirjoitti: mistral - 27.12.2023, 17:30:01
Lainaus käyttäjältä: Eusa - 21.12.2023, 22:19:03Tarkoitan sitä nostetta, joka näkyy yleisen suhteellisuusteorian matematiikassa.

Einsteinin perintö kirjassa ei muistaakseni ollut mitään nosteesta. Toiseksi jos aika ei muutu, niin se on alkuoletus. Alkuoletuksen mukaan pitäisi ajan ongelma ratkaista, jos ratkaistaan YS:n avulla, siitä tulee kehäpäätelmä. Eli jos YS on alkuoletuksena, tulee ratkaisuksi YS:n mukainen tulos, eihän tulosta voi etukäteen päättää. Siksipä joudutaan pureutumaan ajan hidastumisen mekanismiin ihan sellaisenaan. Esitän mekanismille havainnollistusta, 2 tikapuut. Alhaalla gravikaivossa on tikapuut maassa siis vaakatasossa, samoin ylhäällä kaivon reunalla, vaakatasossa. Yksi puolaväli kuvaa minuuttia. Tikkaat liikkuu vaikka länteen yhden puolan vauhtia minuutissa (sekä alhaalla että ylhäällä). Nyt kun valo punasiirtyy niin ylhäällä intuitio sanoo että alatikkaat menee hitaammin kuin ylätikkaat. Tässä nyt on se mekanismi, kelpaako sinulle? Vai mikä olisi oikea ymmärrys? Jos tämä on hyvä niin seuraavaksi pitäisi avata mekanismia, onko se "vedenpitävä".
Otsikko: Vs: Epäselvyys suhteellisuusteorian aika ja etäisyys käsityksissä
Kirjoitti: velihopea - 07.01.2024, 12:42:04
Päivää pitkästä aikaa, ja hyvää uutta vuotta 2024.

Lainaus käyttäjältä: Eusa - 18.12.2023, 00:26:41Näyttää tämän ketjun sivulla 3 Velihopea esitelleen Einsteinin papereita...

Paperi on itse asiassa mistralin löydös, seuraava: "Dialog about relatitivity theory" sivut 66-75. https://einsteinpapers.press.princeton.edu/vol7-trans/82. Teksti on Einsteinin 1918 kirjoittama. En osaa yhdistää sitä mistralin ja Eusan joulukuussa käymään keskusteluun. Minusta paperi on ikäänkuin oppi-isän (AE) neuvontaa miten ST:n relatiivisuusperiaatetta ei pidä tulkita väärin, mikä väärintulkinta on yhä voimissaan nykyisessä ST-opetuksessa.

Paperissa esitetään ajatuskoe, ensin ikäänkuin kaksosparadoksin perustapaus. Siinä kello U1 pysyy paikallaan ja kello U2 liikkuu edes-takaisen lenkin ja palaa U1:n viereen. U2:n kello todettiin jääneen jälkeen U1:sta. Sitten opetuslapsi kysyy ~"Eikö relatiivisuusperiaatteen mukaisesti U2 voikin ajatella olevansa paikallaan ja U1 tekee vastaavaan edes-takaisen liikkeen toiseen suuntaan, ja nyt U1-kellon pitäisi olla U2:ta jäljessä". Olen nimennyt tämän symmetriatapaukseksi.

Tähän AE vastaa ~"Periaateessa kyllä, mutta nuo tapaukset eivät ole samanlaisia ja siten ne eivät ole vertailukelpoisia, koska niissa tapahtuvat erilaiset prosessit". AE:n kuvauksen perustapaukselle olen selostanut postauksessa #31 ja symmetriatapaukselle #32 (25.07.2023). Perustapauksessa U2 liikkui ulkoisen voiman avaulla, kenties avaruusraketilla. Mutta symmetriatapauksen kuvaus oli mitä ihmeellisin, minusta mahdoton. Sen tapahtumakulussa jostain nousee hetkellinen (pseudo)gravitaatiovoima, jolla saadaa U1 liikkeelle, kääntymään ääripäässä ja taas pysähtymään U2:n viereen. Ja U2:lle ilmaantuu jostain vastaava hetkellinen paikallaanpitovoima.

Pointiksi nostan: AE minusta opettaa, että ei riitä, että U2 kuvittelee olevansa paikallaan ja se onkin U1 joka liikkuu. Kuvittelun voimasta tuo toinen erilainen tapahtumakulku ei aktivoidu. Vaan oikeasti pitää tapahtua, että U2 on paikallaan ja U1 liikkuu!

Jos katsoo nykyisiä ST-opetusvideoita niissä kaksosparadoksin symmetriatapaus kuvataan niin, että raketti on paikoillaan ja maa ja koko muu kosmos vilistää raketin ikkunassa. Ikäänkuin sellainen oikeasti voisi tapahtua! Haloo! Tuollaisen "tosta-vaan tulkinnan" AE:kin kieltää tuossa paperissaan!

Toinen yleinen opetus, minusta vääränä ST-tulkintana, on, että esim. kun 0.8c-raketissa kelloon tulee 60 min, raketin kuski voi luulla ja pitää totena, että paikalleen jääneessä maan kellossa olisi vasta 36 min. Mutta näin voi olla vain, jos oikeasti raketti on jäänyt paikalleen ja maa on alkanut etääntymään raketista 0.8c-nopeudella. Esittämäni ajatuskoe, jossa oli maa ja raketit R1 ja R2 (postaus #108,   05.09.2023 ja #153 2.10.2023), minusta osoitti, että tuollainen väärintulkinta ei voi pitää paikkansa.
Otsikko: Vs: Epäselvyys suhteellisuusteorian aika ja etäisyys käsityksissä
Kirjoitti: mistral - 07.01.2024, 20:18:25
Samoin hyvää uuttavuotta.
Einsteinin selitys on kyllä sinänsä looginen mutta pseudogravitaatiokenttä ei vaan ole todellinen ja siihen se kaatuu. Miksi kaatuu, siksi koska 1) epätodellisesta kentästä puuttuu punasiirtävä mekanismi. Mutta on vielä toinenkin syy 2) punasiirtokaan ei välttämättä hidasta aikaa. Tämä on oma teoriani jota kuvasin edeltävissä viesteissä. Jos teoria toimii, fysiikka yksinkertaistuu melkoisesti, esim tapahtumahorisontin aika ei muuttuisikaan.
Otsikko: Vs: Epäselvyys suhteellisuusteorian aika ja etäisyys käsityksissä
Kirjoitti: Eusa - 09.01.2024, 21:40:29
Lainaus käyttäjältä: mistral - 07.01.2024, 20:18:25Samoin hyvää uuttavuotta.
Einsteinin selitys on kyllä sinänsä looginen mutta pseudogravitaatiokenttä ei vaan ole todellinen ja siihen se kaatuu. Miksi kaatuu, siksi koska 1) epätodellisesta kentästä puuttuu punasiirtävä mekanismi. Mutta on vielä toinenkin syy 2) punasiirtokaan ei välttämättä hidasta aikaa. Tämä on oma teoriani jota kuvasin edeltävissä viesteissä. Jos teoria toimii, fysiikka yksinkertaistuu melkoisesti, esim tapahtumahorisontin aika ei muuttuisikaan.
Oletko ottanut huomioon vaihtoehtoa, että gravitaatio kaikissa muodoissaan olisikin näennäinen eli pseudoilmiö? Varmuudella todellista on vain paikallistivien ainekappaleiden energeettinen rakenne, säilyvä fysikaalinen virta ja muodostuvat tiheydet sekä vuorovaikutuskiihtyvyydet.

Aika-avaruus kaarevana olisi vain välttämätön käsite erillisyyksien etäisyyksille ja korreloiva tila aineen rakenteelliselle ikääntymiselle; muutoksille?...
Otsikko: Vs: Epäselvyys suhteellisuusteorian aika ja etäisyys käsityksissä
Kirjoitti: mistral - 10.01.2024, 14:09:00
Lainaus käyttäjältä: Eusa - 09.01.2024, 21:40:29Oletko ottanut huomioon vaihtoehtoa, että gravitaatio kaikissa muodoissaan olisikin näennäinen eli pseudoilmiö? Varmuudella todellista on vain paikallistivien ainekappaleiden energeettinen rakenne, säilyvä fysikaalinen virta ja muodostuvat tiheydet sekä vuorovaikutuskiihtyvyydet.

Aika-avaruus kaarevana olisi vain välttämätön käsite erillisyyksien etäisyyksille ja korreloiva tila aineen rakenteelliselle ikääntymiselle; muutoksille?...

Tässä ollaan ongelman luonteessa: mikä on ys:n dilaatiomekanismi? Kun se varmistuu (siis ys:n mukaan), voidaan nähdä kuinka se voi toimia Einsteinin pseudogravitaatiossa. Eli ensin dilaatiomekanismi ja sitten sen soveltaminen pseudogravitaatiokenttään.

Itse olen ymmärtänyt mekanismin yksinkertaisesti aallonpituuden kasvuksi (tai lyhenemiseksi). Esimerkkinä Schwarzschildin säde, siinä aallonpituus kasvaa äärettömäksi kun aalto lähetetään horisontista suoraan ylös ja koska äärettömän pitkä aalto per aallon lähtöpituus = ääretön ==> ääretön dilaatio. Mikä tarkoittaa ajan pysähtymistä horisontissa.

Jos edellä kuvattu on oikea ymmärrys, ei pseudogravitaatio pysty tuottamaan dilaatiota koska se kenttä ei oikeasti muutu miksikään gravitaatiopotentiaaliltaan. Aidossa mustan aukon kentässä gravitaatiopotentiaali muuttuu mutta pseudokentässä ei ole mitään massaa joka muuttaisi sitä.

Näin mekanismi jolla aallonpituus lisääntyy, puuttuu. Jos keksit jonkun muun mekanismin joka selittää dilaation, se olisi kiinnostavaa.
Otsikko: Vs: Epäselvyys suhteellisuusteorian aika ja etäisyys käsityksissä
Kirjoitti: Eusa - 10.01.2024, 14:58:11
Lainaus käyttäjältä: mistral - 10.01.2024, 14:09:00Tässä ollaan ongelman luonteessa: mikä on ys:n dilaatiomekanismi? Kun se varmistuu (siis ys:n mukaan), voidaan nähdä kuinka se voi toimia Einsteinin pseudogravitaatiossa. Eli ensin dilaatiomekanismi ja sitten sen soveltaminen pseudogravitaatiokenttään.

Itse olen ymmärtänyt mekanismin yksinkertaisesti aallonpituuden kasvuksi (tai lyhenemiseksi). Esimerkkinä Schwarzschildin säde, siinä aallonpituus kasvaa äärettömäksi kun aalto lähetetään horisontista suoraan ylös ja koska äärettömän pitkä aalto per aallon lähtöpituus = ääretön ==> ääretön dilaatio. Mikä tarkoittaa ajan pysähtymistä horisontissa.

Jos edellä kuvattu on oikea ymmärrys, ei pseudogravitaatio pysty tuottamaan dilaatiota koska se kenttä ei oikeasti muutu miksikään gravitaatiopotentiaaliltaan. Aidossa mustan aukon kentässä gravitaatiopotentiaali muuttuu mutta pseudokentässä ei ole mitään massaa joka muuttaisi sitä.

Näin mekanismi jolla aallonpituus lisääntyy, puuttuu. Jos keksit jonkun muun mekanismin joka selittää dilaation, se olisi kiinnostavaa.
Aikadilaatio olisi syytä kiinnittää vain tasaisen nopeuden havaintoprojektiiviseen perspektiivitilanteeseen.

Kun kappale vuorovaikuttaa ulkoisesti ja kiihtyy, silloin siirtyy energiaa ulkoisen vuorovaikuttajan kanssa ja se toinen kiihtyy vastasuuntaan.

Koska yhteiskappaleen liiketila ei muutu, se on aika-avaruuden intervallien suhteen invariantti. Sen sijaan kumpikin osakappale siirtyy uuteen asemaan yhteisen massakeskipisteen suhteen - kiihtyvyys on absoluuttista liikettä. Siksi molempien vuorovaikuttavien kappaleiden on oltava oikeassa siinä, että toisen aika hidastuu, MUTTA lisäksi oleellisesti: MOLEMPIEN ikääntyminen nopeutuu yhteisen massakeskipisteen suhteen; sillä kiihtyjällä voimakkaammin kumpi kiihdyttää enemmän ja kumpi on kauempana massakeskiöstä. Noin ollen ikääntymisnopeus on herkkä kappaleen massalle - kevyen ikämuutokset tapahtuvat helpommin, vähemmällä energian käytöllä.

Toisaalta, mikäli kappaleet voisivat vetää toisiaan vastaavalla voimalla välittömästi takaisin yhteen ilman ajelehtimista, tapahtuu päinvastainen ja erityisesti massakeskipisteen suhteen tilanne nollautuu ja toistensakin suhteen kappaleiden iät ovat kehittyneet symmetrisesti.

Minkä tahansa inertiaalisen "majakan" suhteen: kappaleen kiihtyessä kohti sen ikääntyminen hidastuu majakan suhteen ja poispäin kiihdyttäessä ikääntyminen nopeutuu.

Koska muutos riippuu myös etäisyydestä, kerran toisistaan pois poukonneet eri massaiset kappaleet ikääntyvät eri tahtia yhteisen massakeskipisteen suhteen. Sen sijaan yhteisen optisen keskipisteen suhteen ne ikääntyvät samaa tahtia.

Ajatellaan seuraavaksi aineen sidoksia. Niin kauan kun rakenteessa sidosvoimat kiihdyttävät hiukkasia, niillä on yhteinen ikääntyminen. Mutta ulkopuolella olevien kappaleiden suhteen muodostuu kenttä, jossa eri etäisyyksillä olevien ikääntymiset ovat tämän kappaleen kiihtyvien hiukkasten suhteen erilaisia.

Kun nuo ikäkehitykset tulkitaan ikääntymisen kentäksi, saadaan kaareutunut ikääntymisympäristö. Tämä johtaa gravitaatioilmiöön.

Perimmäinen syy on ainekertymän paineinen nostekiihtyvyys ja koskee kaikkia irtolaisia siltä osin, kun ne eivät ole samassa jatkuvassa sähkö-, heikko-ja ydinvoimien riepottelussa keskusmassan kanssa vaan vapaassa pudotuksessa.
Otsikko: Vs: Epäselvyys suhteellisuusteorian aika ja etäisyys käsityksissä
Kirjoitti: mistral - 10.01.2024, 21:57:48
Näköjään ikääntymismekanismi voidaan ymmärtää niin eri tavalla. Kuitenkin YS on tiettyjen ehtojen rajoittama ratkaisu ja ikääntyminen jotenkin nousee olisko kenttäyhtälöistä. Eli kenttäyhtälöihin on kätketty tieto kuinka ikääntyminen ilmenee, mutta yhtälöiden takana on kuitenkin melko yksinkertainen fyysinen maailma. Jos kuka tavoittaa tämän maailman, hänellä on oikotie ymmärtää sen toimintaa. DU-teoriassa on 4. metrinen ulottuvuus kun suhteellisuusteoriassa on 3 metristä ulottuvuutta. Yhteistä kuitenkin on avaruuden kaareutuminen eli kun sanotaan että avaruus kaareutuu, se tarkoittaa DU:ssa 4. ulottuvuuteen kun taas suhteellisuudessa se tarkoittaa eri asiaa. Mutta kumpikin puhuu avaruuden kaareutumisesta, tämä on yhteistä. Mutta sitten aika kysymys menee eri tavalla, DU:ssa aika ei muutu ja siksi se etsii selitystä atomikellojen tuloksille siitä että avaruuden dynamiikka muuttaisi valon nopeutta joka puolestaan muuttaa atomikellojen käyntinopeutta. Eli on yritystä yksinkertaistaa fysiikkaa, tosin itse en ymmärrä DU-teoriaa sen enempää.
Joka tapauksessa oletan että Einstein on ensin saanut oivalluksen avaruuden fysiikasta ja vasta sitten ollaan kehitetty kenttäyhtälöt jotka täyttää Einsteinin "avaruuskuvan". Tai epäilen että se olisi toisinpäin: ensin kenttäyhtälöt ja sitten katsotaan millainen fyysinen avaruus siitä muodostuu. Joten se Einsteinin "avaruuskuva" on ollut yksinkertaisempi kuin sen ympärille rakennettu matikka. Olisi kiva tietää kuinka Einstein sai ymmärryksen aika-avaruuden ikääntymismekanismista, sehän on ollut siinä "avaruuskuvassa" aivan olennainen osa.
Otsikko: Vs: Epäselvyys suhteellisuusteorian aika ja etäisyys käsityksissä
Kirjoitti: Eusa - 13.01.2024, 11:06:22
Suhteellisuusteorioissa on 4 metristä ulottuvuutta - tämä korjattakoon. Dimensio suureelle ct on m/s×s; sekunnit supistuvat pois ja jää metri.